Download as docx, pdf, or txt
Download as docx, pdf, or txt
You are on page 1of 217

ENT MCQs CORRECT

EAR :
1. The best treatment of traumatic perforation of tympanic membrane is: (HBS)
 Mastoidectomy
 Wait & see*
 Myringoplasty
 Antibiotic ear drops
2. A 50-year-old diabetic has presented to the ENT OPD with severe pain in the right ear for the last
2 days. The pain increased during chewing & eating meals. There is no complaint of ear
discharge or itching. On palpation there is tenderness over the right tragus. The most likely
diagnosis is: (HBS)
 Malignant otitis externa
 Impacted wax
 Acute suppurative otitis media
 otomycosis
 Boil ear * ( furuncles )
3. A 30-year-old female has presented to the ENT Outpatients with severe right sided earache for
the last two days and right sided facial palsy for the last 1 day. There is no history of ear
discharge or trauma to the ears. Currently there are no complaints of fever, ear, or nasal
discharge. On examination there are vesicles on the tympanic membrane, deep meatus, concha,
and retro auricular sulcus. The appropriate management in this condition is: (HBS)
 Corticosteroids*
 bed rest and reassurance
 antibiotics
 nerve decompression
 a+ntiviral therapy lij
4. A 35-year-old female patient with history of right chronic ear discharge. developed severe pain in
right side of head, fever, and neck stiffness two days back. When presented in emergency
department, she had an altered conscious level. Most appropriate diagnosis in this case will be:
(HBS)
 Meningitis*
 Subdural abscess
 Otogenic brain abscess
 Lateral sinus thrombophlebitic
5. A 40-year-old factory worker has presented with progressively decreased hearing in both the ears
for the last 3 years. His pure tone audiometry was done which revealed sensorineural hearing
loss in both the ears with dip in air & bone conduction at 4k. this type of graph is typically seen
in: (HBS)
 Presbycusis
 Acoustic tumour
 Noise trauma*
 Otosclerosis
 Meniere's disease
6. Which statement is true regarding external auditory meatus? (HBS)
 Straight in adults
 Its outer one third is bony
 It is 2.5 cm long*
 Its deeper part contains hair
 It is 2.5 inches long
7. A 24 years old male soldier by profession has reported with history of deafness and intermittent
discharge from left ear for the last 03 years. At present he has medium sized central perforation
in left ear, dry for the last 03 months. Audiogram shows 30 dB conductive hearing loss in left
ear. What should be the choice of treatment? (HBS/RIHS)
 Conservative treatment
 Radical Mastoidectomy with tympanoplasty
 Myringoplasty*
 Hearing aid
8. A 22-year-old female has presented to the ENT OPD with intense itching & pain in the left ear for
the last 1 week along with watery discharge. On otoscopy there are blackish spores in the left
ear. The 1st step in management would be: (HBS)
 Topical steroids
 Oral antifungal
 Topical antibiotics
 Aural toilet*
 Topical antifungal
9. The best treatment for traumatic perforation of drum is (HBS)
 Myringoplasty
 Put a piece of gel foam on perforation
 Suction toilet and local antibiotic drops
 Observe for spontaneous healing and consider myringoplasty in case of failure*
 Local antibiotic drops
10. A 45-year-old female presented with the complaint of attacks of severe vertigo at night
especially when she turned to the left side in bed for last four days. It lasted for 1 to 2 minutes &
was associated with nausea, vomiting & sweating. There is unidirectional nystagmus. This was
not accompanied by any hearing loss or tinnitus. The most likely diagnosis in this patient is:
(HBS)
 Meniere's disease
 Benign paroxysmal positional vertigo*
 Vestibular neuronitis
 Basilar migraine
 Labyrinthitis
11. BELL'S palsy is characterized by (HBS)
 ipsilateral lower motor neuron lesion with sparing of forehead
 contralateral upper motor neuron lesion with sparing of forehead
 ipsilateral upper motor neuron lesion with involvement of forehead
 ipsilateral upper motor neuron lesion with sparing of forehead
 ipsilateral lower motor neuron lesion with involvement of forehead*
12. A 58-year-old man who has 30 years of Diabetes Mellitus developed sudden pain in the right ear
with fever. On examination there was severe inflammation with small red swelling in the right
external auditory canal. The tympanic membrane was intact. He is most likely suffering from.
(HBS)
 CSOM with Cholesteatoma
 Squamous cell carcinoma of the ear
 Boil ear*
 Malignant otitis externa
13. A patient presents with severe pain in the right ear. On examination shows vesicles on the right
pinna along with tenderness of right pinna and right facial palsy. What is the diagnosis? (HBS)
 Myringitis bullosa
 Ramsay Hunt syndrome*
 Bell's palsy
 Malignant otitis externa
14. The patient complains of reduced hearing. The tuning fork test show positive Rinne test on right
side and Weber test lateralizing to opposite side. This means: (HBS)
 Conductive deafness on right side
 Perceptive deafness on right side
 Cochlear hydrops on right side
 Sensorineural deafness on right side*
15. Middle ear cleft consists of: (HBS)
 Eustachian tube, internal auditory and external auditory meatus
 Eustachian tube and tympanic cavity
 Eustachian Tube
 Eustachian tube, middle ear cavity and mastoid air cell system*
 Eustachian tube and internal auditory meatus
16. Carhart's notch is dip in bone conduction seen in patients with otosclerosis. It is usually present
at: (HBS)
 3 KH2
 1 KH2
 6 KH2
 2 KH2*
 5 KH2
17. Benign paroxysmal positional vertigo is vertigo provoked by change in head position and is
relieved after some seconds or minutes. Which clinical test helps its diagnosis? (HBS)
 Dix Hallpike Test*
 HINTS test
 Fistula test
 Caloric test
18. A known case of CSOM presented to the emergency with the complaint of headache, nausea &
vomiting for the last 1 day. Her attendant told that the ear discharge used to be scanty & foul
smelling and sometimes blood stained. The most appropriate investigation in this case would
be: (HBS)
 Pure tone audiogram
 Otomicroscopy
 X ray mastoid
 CT scan*
 Culture and sensitivity of ear discharge
19. A 25-year-old lady presents in ENT OPD with progressive bilateral hearing loss accompanied with
attacks of tinnitus. On otoscopy bilateral tympanic membranes are normal looking. Family
history is positive. Audiogram shows a dip at 2 KHz. What is the best possible diagnosis? (HBS)
 Meniere's disease
 Acoustic neuroma
 Vestibular neuronitis
 Otosclerosis*
20. Furunculosis is caused by. (HBS)
 Proteus
 Pneumococcus
 Pseudomonas
 Staphylococcus*
 E. coli
21. An old man is having long standing ear discharge. Recently has started having severe pain the
same ear. On examination there is granulation tissue along with ear discharge in the external
auditory meatus and middle ear cavity. The investigation of choice in this case is (HBS)
 Biopsy of granulations
 X-ray mastoids
 ESR
 CT scan*
 MRI
22. A 30-year-old male smoker has presented to the ENT OPD with episodic attacks of vertigo for the
last 2 months, fluctuating hearing loss & tinnitus. On otoscopy both the tympanic membranes
are normal. The patient's pure tone audiogram shows sensorineural hearing loss mainly in the
low frequencies. The most likely diagnosis is: (HBS)
 Benign paroxysmal positional vertigo
 Vestibular neuronitis
 Meniere's disease*
 Acoustic neuroma
 Labyrinthitis
23. A 55-year-old lady presented with deafness. Her pure tone audiogram showed average
threshold of 67dB. You will classify her hearing loss into which category? (HBS)
 Moderately severe*
 Mild
 Moderate
 Severe
24. A 20-year-old boy presented in OPD with swelling behind the right ear for five days. The swelling
was preceded by discharge from the same ear for 3 weeks along with occasional pain behind the
ear. ON examination there was erythema behind Right pinna with protrusion of right pinna. The
likely diagnosis in this patient is: (HBS)
 Bezold Abscess
 mastoid Abscess*
 Pharyngeal Abscess
 Boil in the ear canal
25. Patient came to ENT emergency with severe pain in right ear and hearing loss when he came
from Karachi by air. On otoscopic examination tympanic membrane is congested and patient is
afebrile. What is diagnosis (HBS)
 Barotrauma*
 Haemotympanum
 Dislocation of ossicles
 Otitis media with effusion
26. A young girl was being syringed for removal of wax. Suddenly she developed severe vertigo. The
reason for her vertigo is (HBS)
 Too cold water has been used for syringing*
 There was healed perforation in the ear drum
 Deep auricular branch of vagus has been stimulated
 There was perforation in the ear drum
 The patient was already suffering from labyrinthitis
27. A 22-year-old female has presented to the ENT outdoor with the complaint of right sided
earache for the last 4 days. There is no history of ear discharge, fever, or hearing loss. On
examination the tympanic membrane is normal. However, she has impacted carious right lower
third molar. The most likely cause of earache in this patient is. (HBS)
 Referred pain via the 10'th cranial nerve
 Referred pain via the 9'th cranial nerve
 Idiopathic
 Referred pain via the 5'th cranial nerve*
 Referred pain via the cervical spinal nerves C2 & C3
28. A 60-year-old diabetic has presented to the ENT OPD with severe right sided earache & facial
paralysis for the last two days. On otoscopy granulations are present in the external auditory
canal. The appropriate drug of choice in such patient would be: (HBS)
 Metronidazole
 Corticosteroids
 Penicillin
 Clotrimazole
 Ciprofloxacin*
29. A 5-year-old child is brought to the ENT OPD with the complaint of decreased hearing. At home,
the child tends to increase the volume of TV and does not respond when called from behind. He
also snores a lot during sleep. His tympanometry is done. Which curve is expected? (HBS)
 Type A
 Type C
 Type As
 Type B*
 Type Ad
30. A fifty-year-old lady sustained head injury and had bleeding from her left ear. She also had
perforation in the ear drum which healed in two weeks’ time. However, she kept complaining of
hearing loss in the same ear. Audiogram showed 60dB conductive hearing loss with high
compliance depicted on tympanogram. The diagnosis is (HBS)
 Scarred ear drum
 Stapedius tendon disruption
 Fracture temporal bone
 Ossicular chain disruption*
31. A 35-year-old female presented to the ENT OPD with the complaint of severe dizziness on lying
down. especially when she turns to the left side in bed. The dizziness is associated with nausea.
vomiting & sweating. This is not accompanied by hearing loss or tinnitus. The most important
step in the management of this patient is (HBS)
 Antiemetics
 Bedrest
 Particle repositioning maneuver/Epleys maneuver*
 Reassurance
32. In a patient with severe itching and pain in right ear in hot humid weather, otoscopic
examination revealed bloating paper like material in right ear. The best treatment is (HBS)
 Analgesics and soda glycerin drops
 Oral antifungal drugs
 Suction and antifungal drops*
 Antifungal drops
33. Bezold's abscess is collection of pus. (HBS)
 Over zygoma
 Along stylopharyngeus
 Along posterior belly of digastric*
 In sheath of sternomastoid muscle
34. A 45-year-old male patient has presented with the complaint of progressively decreasing hearing
in the left ear for the last one year. There are no complaints of ear pain, blocking sensation &
discharge from ears. On otoscopy both the tympanic membranes are normal. On tuning fork
tests. Rinne's is +ve on the right side and -ve on the left. Weber is lateralized towards the left
ear. The diagnosis on the basis of tuning fork tests is (HBS)
 Right sensorineural hearing loss
 Left sensorineural hearing loss
 Left conductive hearing loss*
 Mixed hearing loss in both ear
 Right conductive hearing loss
35. A 45-year-old male patient has presented with the complaint of progressively decreasing hearing
in the left ear for the last one year. There are no complaints of ear pain, blocking sensation &
discharge from ears. On otoscopy both the tympanic membranes are normal. On tuning fork
tests. Rinne's is +ve on the right side and -ve on the left. Weber is lateralized towards the Right
ear. The diagnosis on the basis of tuning fork tests is (HBS)
 Right sensorineural hearing loss*
 Left sensorineural hearing loss
 Left conductive hearing loss
 Mixed hearing loss in both ear
 Right conductive hearing loss
36. The posterior meatal wall is left infact in. (HBS)
 Mastoidectomy
 Fenestration Operation
 Modified Radical Mastoidectomy
 Cortical Mastoidectomy*
 Radical Mastoidectomy
37. A 20 year old male patient has presented to the ENT OPD with bilateral ear discharge for the last
3 years. The discharge is profuse and not foul smelling or blood stained. On otoscopy there is a
central perforation in the pars tensa in both ears. The best treatment option in this patient is
(HBS)
 Topical ear drops*
 Myringotomy with Gromet insertion
 Mastoid exploration
 Anti allergics & nasal decongestants
 Oral antibiotics
38. A patient presented with severe pain in the ear and facial nerve palsy. Examination shows
vesicles on pinna, retro auricular sulcus and in the ear canal. Which of the following cranial
nerve is involving in this condition: (HBS)
 VII cranial nerve*
 IX cranial nerve
 XI cranial nerve
 X cranial nerve
39. A boy is suffering from Acute Otitis Media. His symptoms are not settling in spite of adequate
medical treatment, T.M is bulging. Treatment is: (HBS)
 Myringotomy*
 Aspiration
 Analgesics
 Mastoidectomy
 Topical lignocaine drops
40. A 22-year-old male presented to the ENT outdoor with sudden onset of right sided earache for
the last 4 days. Earache was preceded by upper respiratory tract infection. On examination the
right tympanic membrane is congested with leash of blood vessels along the handle of the
malleus. Tuning fork tests reveal conductive hearing loss in the right ear. The organism most
commonly implicated in this condition is (HBS)
 Staphylococcus aureus
 Pseudomonas aeroginosa
 Bacteroides fragilis
 E soli
 Streptococcus pneumoniae*
41. A 20-year-old male patient presented to the ENT OPD with the complaints of ear discharge &
deafness in the right ear for the last 12 years. On inquiring further about the complaints. he told
that the discharge is persistent. scanty, foul smelling & occasionally blood stained. On otoscopy,
there is pearly white keratin collection in the pars flaccida. The best treatment option in this
patient is (HBS)
 Tympanoplasty
 Myringotomy
 Oral antibiotics
 Mastoid exploration*
 Topical antibiotic drops
42. A 50-year-old male presented with unilateral progressive hearing loss that is sensory neural
type. He also complains tinnitus and facial numbness. CT scan brain shows a homogenous mass
on Cerebello-pontine angle. What is your most likely diagnosis? (HBS)
 Vestibular schawanoma*
 Meningioma
 Brain cyst
 Cavernous sinus thrombosis
43. A young boy was involved in road traffic accident. He developed vertigo when he tried to move
about. On examination he had blue ear drum. Audiogram done two weeks later confirmed
sensorineural hearing loss. The likely diagnosis is (HBS)
 Longitudinal fracture of temporal bone
 Transverse fracture of temporal bone*
 Ossicural chain disruption
 Disruption of spiral ganglion
44. A 2-year-old child was referred by pediatrician to ENT department for hearing evaluation, with
history of neonatal meningitis. Most appropriate audiological investigation in this case will be
(HBS)
 Auditory brain stem evoked response Audiometry*
 Tympanometry
 Speech reflex test
 Pure tone Audiometry
45. A 14-year-old girl had her right ear pierced 3 weeks ago which was followed by severe earache &
swelling. She is not responding well to the usual treatment. If this condition is not treated
aggressively which of the following cosmetic deformity is likely to develop. (HBS)
 Microtia
 Cauliflower ear
 Keloid*
 Bat ear
 Macrota
46. A 65-year-old patient presented to the ENT OPD with the complaints of progressively decreased
hearing in both the ears for the last 5 years. He often has to ask others to repeat what they have
said. He often complains that he can hear the sound but cannot understand what is being said.
The most appropriate investigation in this patient would be. (HBS)
 Pure tone audiometry
 Brainstem evoked response audiometry
 Bekesy audiometry
 Otoacoustic emissions
 Tympanometry*
47. Bony facial canal is narrowest at which segment. (HBS)
 Labyrinthine*
 Mastoid
 Tympanic
 Q Meatal
48. A 23-year-old female has developed dizziness after a mild head injury. The spinning sensation
during dizziness lasts for two to five seconds. The best treatment in her case will be: (HBS)
 Dix hallpike test
 Reassurance*
 Epley's maneuver
 Anti emetic drugs
49. A 22-year-old female has presented to the ENT OPD with blockage and decreased hearing in the
right ear for the last two days. On otoscopy. there is hard impacted wax in the right ear. The
first step in the management of this patient is (HBS)
 Wax softening agents*
 Antibiotic drops
 Suction
 Corticosteroid ear drops
 Syringing
50. Otitis Media with effusion (OME) is common problem encountered in children. Which one is the
objective test that helps in diagnosis? (HBS)
 Brainstem evoked response audiometry
 Pure tone audiometry
 Otoacoustic emission
 Otoscopy
 Tympanometry*
51. A 30-year-old male presented with headache and intermittent high-grade fever with rigors. He
also gives history of right ear discharge for last 15 years. Tenderness can be elicited on palpation
of right side of upper neck. He is markedly pale, and HB is 7.8g/dl. The most likely diagnosis is
 Bezold's abscess
 Mastoiditis
 Lateral sinus thrombosis*
 Meningitis
52. A 40-year-old diabetic patient is complaining of severe itching for 10 days. He also has
conductive hearing loss. Examination with Otoscope showed black-headed Hyphae. The most
likely diagnosis is (IMDC)
 Foreign body in ear
 Aspergillus Niger*
 Aspergillus fumigatus
 Candida infection
 Mucormycosis
53. Tympanometry showed decrease compliance, Audiometry showed mixed hearing loss with
Carhart’s notch seen on (IMDC/FMDC)
 In bone conduction
 In air conduction
 In Bone conduction at 1000Hz
 In Bone conduction at 2000Hz*
 Air conduction at 2000 Hz
54. A 25 years' male presented red swollen pinna, tender to touch. What is the treatment? (IMDC)
 Analgesic and ciprofloxacin
 Co-amoxiclave is drug of choice
 Analgesic and Co-amoxiclave
 Analgesic and I&D
 Third generation cephalosporin’s and I&D *
55. A 30 years' male got ear trauma, post traumatic profound hearing loss occurs in (IMDC)
 Longitudinal fractures of temporal bone
 Le forte type III fracture
 Rupture of tympanic membrane
 Transverse fracture of temporal bone*
 Fracture squamous temporal bone.
56. A 35 years' male presented with combination of fluctuating SNHL, roaring tinnitus-, episodic
vertigo and fullness of the ear is most likely to be associated. (IMDC)
 Otosclerosis
 Chronic SOM
 Meniere's Disease*
 Acoustic Neuroma
 Perilymph fistula
57. Severe headache, vomiting, dysphasia & visual field defects in a patient with Cholesteatoma
indicate: (IMDC)
 Cerebellar Abscess
 Otogenic facial paralysis
 Mastoid abscess
 Temporal lobe abscess*
 Meningitis
58. A patient suffered from right sided hearing loss after having bath. He is also complaining of aural
fullness. Otoscopic examination shows external auditory canal full of brownish material. What is
the best treatment? (IMDC)
 Syringing
 Dry mopping
 Soda glycerin drops followed by syringing
 Soda glycine drops followed by suctioning*
 suction
59. A 40-year-old patient is complaining of severe earache for 2 days. He also has conductive hearing
loss. Examination with Otoscope was painful showed swollen external auditory meatus.
Commonest organism responsible for it is? (IMDC)
 Staph aureus *
 Beta hemolytic streptococcus
 Candida
 Pseudomonas
 Haemophilus influenza
60. During middle ear surgery, medial wall of middle ear was examined. It includes all except (IMDC)
 Oval window
 Processus cochleariformis
 Round window
 Pyramid*
 First genu of facial nerve
61. Myringotomy with grommet insertion is the ideal surgical treatment for (IMDC)
 Acute otitis media
 Serous otitis media*
 Adhesive otitis media
 Retracted tympanic membrane
 Cholesteatoma
62. Audiogram showed hearing loss both in AC and BC with no air bone gap. Following are the
causes except (IMDC)
 Ototoxic drugs
 Torch Infections*
 Noise
 Otosclerosis
 Meniere’s disease
63. A 35 years' male presented with vertigo lasts for few seconds by putting head on pillow, while
Tuning Fork tests are normal. The Diagnosis is? (IMDC)
 Meniere's disease
 Acoustic Neuroma
 Benign paroxysmal positional vertigo *
 Labyrinthitis
 Vestibular neuronitis
64. Diagnosis in above case is made by: (IMDC)
 Fistula test
 Romberg test
 Caloric test
 Hall pike’s maneuver*
 Epley's maneuver
65. Commonest indication for Myringotomy is (IMDC)
 Myringitis bullosa
 Serous otitis media*
 Middle ear defect
 Mastoiditis
 Otosclerosis
66. Flat Tympanogram is a feature of (IMDC)
 Ossicular discontinuity
 Serous otitis media *
 Perforation of eardrum
 Otosclerosis
 CSOM
67. Treatment of choice in deafness associated with Attico antral perforation is (IMDC)
 Simple mastoidectomy
 Modified radical mastoidectomy*
 Watch and wait
 Instillation of antibiotic drops
 Myringoplasty
68. In a classic case of Meniere’s disease which one of the following statements is TRUE? (IMDC)
 Carhart’s Notch is a characteristic feature in pure tone audiogram
 Schwartz’s sign is usually present in the Tympanic membrane
 Low frequency sensory neural deafness is often seen in pure tone audiogram *
 Decompression of Fallopian Canal is the treatment of choice.
 Antibiotics and steroids play an important role in the recovery
69. The normal length of external auditory meatus is (IMDC)
 7mm
 10mm
 24mm *
 36mm
 46mm
70. The commonest extra cranial complication following mastoidectomy includes (IMDC)
 Facial nerve palsy *
 Dislocation of incus
 Post-operative hematoma
 Cochlear injury
 Trigeminal nerve injury
71. A 3years old child presents with fever and (Rt) earache from last 48 Hrs, which is not relieving
with medications. On otoscopic examination, there is congested tympanic membrane with slight
bulge with” cartwheel appearance”. What is the treatment of choice in this patient? (IMDC)
 Myringotomy with broad spectrum antibiotics *
 Myringotomy with grommet with antibiotics
 Only antibiotics and analgesics
 Wait and observe the patient.
 Self-limited. No treatment required
72. Common cause of Eustachian tube disease is due to (IMDC)
 Adenoids
 Sinusitis
 Otitis media
 Pharyngitis*
 Tonsillitis
73. A patient of CSOM with Cholesteatoma present with acute onset of vertigo treatment is (IMDC)
 Immediate exploration *
 Antibiotics + Steroids
 Labyrinthine sedatives + Bed Rest
 Labyrinthine sedatives only
 Conservative treatment / No surgical intervention required
74. Condition where a pulsatile tumour is found in external auditory meatus which bleeds to touch
is (IMDC)
 Cholesteatoma
 Polyp
 Glomus tumour *
 Malignancy middle ear
 Acoustic neuroma
75. All of the following are Ototoxic drugs except (IMDC)
 Kanamycin
 Streptomycin
 Gentamicin
 Ampicillin
 Metronidazole *
76. Hypoesthesia of the posterior aspect of the external auditory canal may be an early sign of
(IMDC)
 Trigeminal neuralgia
 Facial paralysis
 Lateral sinus thrombosis
 Multiple sclerosis
 Acoustic neuroma*
77. A 20 years old male patient presents with history of left ear blockage and reduced hearing from
some weeks. On examination, brownish material was found in the external auditory canal. The
best solution for softening the wax is (IMDC)
 Gentacin ear drops
 Normal saline
 Warm oil
 Liquid paraffin
 5 % soda glycerin*
78. A patient comes to ENT OPD with severe pain in right ear and hearing loss from 2dayst. On
exam, blood and air bubbles were seen behind tympanic membrane. Tympanic membrane was
intact. He also has history of travelling by air. The Probable diagnosis is (IMDC)
 ASOM
 CSOM
 Secretory OM
 Barotrauma*
 Ossicular Chain disruption
79. The most common tumor of external auditory canal following long standing blood stained
mucopurulant discharge with severe earache is (IMDC)
 Squamous cell carcinoma *
 Basal cell adenoma
 Adenocarcinoma
 Malignant otitis externa
 Malignant melanoma
80. A 6 years old child brought to ENT OPD for the evaluation of hearing problem. There is suspicion
of Otitis media with effusion. What is the best investigation to confirm the diagnosis? (IMDC)
 Pure tone audiometry
 Tympanometry *
 Tuning fork tests
 Speech audiometry
 Brain stem evoked response audiometry
81. A young female patient came to ENT OPD with complaints of ringing of ears from last some
months. she also gave history of CSOM for which using gentamycin ear drops. Most probable
cause is (IMDC)
 BPPV
 Ototoxicity *
 Salicylates level
 Complication of Cholesteatoma
 Fracture petrous temporal bone
82. A28 years male patient gives history of chronic suppurative otitis media from last 3 years. What
can be the Most common complication if untreated? (IMDC)
 Meningitis*
 Extra Dural abscess
 Petrositis
 Acute Mastoiditis
 Facial paralysis
83. Best treatment for Attic antral type CSOM is (IMDC)
 Cortical mastoidectomy
 Canal wall down procedure *
 Atticotomy
 Myringotomy
 Tympanoplasty
84. A Patient presented with mucoid, odorless, profuse ear discharge from last 1 year. What
otoscopic findings are expected in this case? (IMDC)
 Attic perforation
 Retraction pockets
 Marginal perforation
 Congested tympanic membrane
 Central perforation *
85. Which of the following is a cause of sensorineural hearing loss? (IMDC)
 Presbycusis *
 Tympanosclerosis.
 Otosclerosis
 Longitudinal fracture of the temporal bone.
 Cholesteatoma.
86. A child aged 3 years presented with severe sensorineural deafness, he was prescribed hearing
aids but showed no improvement. What is the next line of management? (IMDC/YMDC)
 Myringotomy
 Stapedectomy
 Cochlear implant*
 Conservative. No treatment required
 Mastoidectomy.
87. A 50 years old diabetic male patient has developed severer itching in his ears. On Otoscopy,
there is debris with black specks in external auditory canal. The treatment of choice in this case
is:- (IMDC)
 Otosporin ear drops
 Gentamicin ear drops.
 Salicylic acid in spirit drops.
 Repeated suction toilet with antifungal drops *
 Oral anti-fungal drugs.
88. A 2 years old baby has put a plastic bead in his ear. On Otoscopy, it is present in deeper portion
of meatus and totally occluding the meatus. The best way to remove this bead is: (IMDC)
 Remove it with foreign body hook.
 Remove it with crocodile forceps.
 Remove it with suction
 Remove it with ear syringing.
 Remove it under general anesthesia using microscope *
89. A complication of common cold:
 Otitis externa
 Quinsy.
 Facial palsy.
 Maxillary sinusitis*
 Inverted papilloma.
90. Cartilaginous part of Eustachian tube is (IMDC)
 1/3
 2/3 *
 1/2
 3/4
 None of the above
91. A16 year’s old boy presents with acute pain in his right ear and a little bleeding from the ear. He
had been in a boxing competition and had sustained blow to the ear. There is a little blood in
the auditory canal and small perforation of the eardrum. What should be treatment plan?
(IMDC/RIHS)
 Leave it alone and prophylactic antibiotics*
 Ear drops
 Tympanoplasty
 Fracture petrous bone
 Oral antibiotics
OR
 Precautions to keep ear dry*
 Ear drops
 Tympanoplasty
 Suction of blood in ear canal
 Topical antibiotics
92. 55-year-old female Known diabetic suffering from severe right sided ear ache with right sided
facial palsy. Ear examination showed granulation tissue in the floor of external auditory canal
(right). The most likely diagnosis is (RIHS/FMDC)
 Bell’s palsy
 Squamous cell carcinoma
 Malignant otitis externa*
 CSOM
 Histiocytosis multiplex
93. A 5-Year-old boy complains of sudden pain in the ear high grade fever. The drum is so congested
Choice of TM (RIHS)
 Gentamycin
 Acyclovir
 Mastoid exploration
 Amoxicillin oral and analgesics *
 Myringotomy
94. A 45 years old man presents with deafness in his right ear which came on after swimming. He
has no previous history of ear problems and examination shows no evidence of inflammation
but some wax. (RIHS/FMDC)
 Surgical intervention
 5% sodaglycerine and suction*
 Syringing
 Suction
 Removal with hook
95. You are suspecting otitis media with effusion in a child of 3years of age. What is the best
investigation to confirm the diagnosis? (IMDC/RIHS/FMDC)
 Brain stem evoked response audiometery
 Pure tone audiometry
 Tuning fork tests
 Tympanometry*
 Speech audiometry
96. Patient on gentamycin comes with ringing of ears (RIHS)
 Ototoxicity*
 BPPV
 Salicylates level
 Fracture petrous temporal bone
 Complications of colesteotoma
97. Meniere’s disease is characterized by all except (RIHS/FMDC)
 Fluctuating hearing loss
 Increase perilymphatic pressure *
 Attacks of vertigo
 SNHL
98. In Acoustic Neuroma all are true except (RIHS/FMDC)
 Is commonest tumor of cerebellopontine angle
 There is always bilateral sensory neural hearing loss*.
 Unilateral tinnitus
 May cause facial paralysis.
 There is usually unilateral sensory neural hearing loss
99. Patient after travelling from America suddenly developed severe pain in right ear and hearing
loss on examination, blood and air bubbles were seen behind tympanic membrane, Tympanic
membrane was intact, patient is afebrile (IMDC/RIHS/FMDC)
 Barotrauma*
 Acute suppurative Otitis media
 Secretary Otitis media
 ossicular chain disruption
 Fracture petrous bone
100. Best solution used for softening wax is (RIHS/FMDC)
 Hydrogen per oxide
 Water
 Oil
 2 to 5% soda glycerin*
 Liquid paraffin
101. Most common intra cranial complication of chronic Suppurative Otitis media (RIHS/FMDC)
 Meningitis *
 Extra Dural abscess
 Subdural abscess
 Raised intra cranial pressure
 Cerebellar abscess
102. A patient with history of CSOM developed vertigo and severe hearing loss, diagnosis? (RIHS)
 Menier’s disease
 Vestibular neuronitis
 Acoustic neuroma
 Labyrinthitis*
 Meningitis
103. A 40 year’ male has developed marked hearing loss in both ears following viral infection. He is
a busy rich businessman. With medical treat What is the best choice for communication? (IMDC)
 Lip reading
 Hearing aids*
 Sign Language
 Cochlear implant
 Bone Anchored hearing Aid
104. A 60 year’ man with diabetes has returned from a holiday from West Indies. He has severe pain
in both ears with swelling and redness of pinna, and pain on pulling the pinna. What is the
treatment? (IMDC)
 Ciprofloxacin, control of diabetes, analgesics*
 Acyclovir
 Mastoid exploration, control of diabetes
 Amoxicillin oral and analgesics
 Control of diabetes, analgesics, antifungal medicines
105. An 18 year’ boy with previous history of chronic Suppurative Otitis media developed vertigo
and Severe hearing loss. Diagnosis is? (IMDC)
 Menier’s disease
 Vestibular neuronitis
 Acoustic Neuroma
 Labyrinthitis*
 Meningitis
106. A 5 year’ boy complains of sudden pain in the ear high grade fever. The drum is congested.
Choice of TM is (IMDC)
 Gentamycin
 Acyclovir
 Mastoid exploration
 Amoxicillin oral and analgesics*
 Myringotomy
107. A 21 year’ male presented with history of hearing loss in right ear of 09 months duration. Right
Ear drum is intact but mobility is restricted. Left Ear drum is normal looking Rhinne’s test is
negative on right side and positive on left side. Weber is lateralized towards right side. Mention
the type of deafness in right ear. (IMDC)
 Conductive deafness*
 Sensory deafness
 Neural deafness
 Mixed deafness
 Psychogenic deafness
108. A 44 year’ known diabetic female suffering from severe right sided earache with right side
facial palsy. Ear examination showed granulation tissue in the floor of external auditory canal
(right). The most likely treatment is (IMDC)
 Control of diabetes
 Control of diabetes, Ciprofloxacillin, Debridement*
 Ciprofloxacillin for six weeks
 Cortical mastoidectomy
 Removal of granulations and control of diabetes
109. Speech frequency is between (IMDC/FMDC)
 20 to 20000Hz*
 400 to 20000Hz
 20 to 10000Hz
 1000 to 20000Hz
 500 to 8000Hz
110. A 32 year male presented with dryness of eyes, dry mouth, facial paralysis of right side. His
hearing is normal. Stepedial reflex is absent. Lesion is at what level. (IMDC)
 Parotid gland
 Vertical part of facial nerve
 Facial nerve nucleus in Pons
 Geniculate ganglion*
 CP angle
111. Most common bacteria causing chronic supperative Otitis media (IMDC)
 Pseudomonas and streptococcus
 Haemophilis influenzae and staph aureus
 Pseudomonas and staph aureus*
 Streptococcus pneumonia and anaerobes
 Anaerobes and streptococcus
112. During hot humid weather, a 25 year’ male developed pain and severe itching in his right ear.
On otoscopy, white curdy discharge is seen in right External Auditory Canal (IMDC/RIHS)
 Otomycosis with Candida infection*
 Aspergillosis
 Exostosis
 Otitis media with effusion
 Acute Otitis externa
113. Best indication of cochlear implant is (IMDC)
 Bilateral sensory neural hearing loss
 Bilateral Conductive hearing loss
 Bilateral profound hearing loss in children*
 Bilateral hearing loss in children
 Bilateral mixed hearing loss in children
114. A 23 years old patient presented in ENT OPD with complaint of pulsatile tinnitus. Otoscopy
shows “Rising sun” appearance. What is the most likely diagnosis? (IMDC)
 Otosclerosis
 Vestibular schwannoma
 Glomus Tumour *
 Acute SOM
 Meniere’s disease
115. What is the average length of external auditory canal in an adult?
 15 mm
 25 mm*
 35 mm
 40 mm
 45 cm
116. Which of the following perforation is typically associated with Attico-antral type of chronic
suppurative otitis media
 Anteroinferior central perforation
 Central Kidney shaped perforation
 Pin hole perforation in pars tensa
 Posterosuperior marginal perforation*
 Posterior central perforation
117. Preauricular sinus results from failure of complete fusion between branchial arches: which of
the branchial arches are responsible for this condition (YMDC/FMDC)
 1st & 2nd branchial arches*
 2nd & 3rd branchial arches
 3rd & 4th branchial arches
 1st & 3rd branchial arches
 3rd & 6th branchial arches
118. A5 Yr girl was diagnosed as a case of Preauricular cyst. What is the cause of this disease
 Infection of the hair follicle
 Congenital anomaly*
 Sebaceous cyst
 Lymhadenitis
 Neoplasm
119. A 30 year female came with H/o insect in her Rt ear She is disturbed with its movements ,
which Ear drops should be used to make the insect unconscious before removal (YMDC)
 Chloroform ear drops*
 Halothane drops
 Ketamine drops
 Xylocaine drops
 Lignocaine drops
120. What is the usual location of boil in the ear (YMDC)
 Outer half of the external auditory canal
 Outer 1/3 of the external auditory canal*
 Outer 2/3 of the external auditory canal
 Inner 1/3 of the external auditory canal
 Any part of the external auditory canal
121. A 5 yr boy was brought with severe Rt earache & fever since last night On examination his Rt
tympanic membrane was bulging & congested What are the common organisms responsible for
this condition (YMDC)
 Staph, E coli & Proteus
 Staph, Morexella & E coli
 Strep pneumonae, H influenzae & M catarrhalis*
 Streptococci, Staph & Pseudomonas
 Staph, Bacteroides & Fungus
122. Which of the following investigation is used to confirm OME (otitis media with effusion)
(YMDC)
 Tuning fork test
 Pure tone audiometry
 Tympanometry *
 Caloric test
 Evoked response audiometry
123. Mother of a 5 Yr child gives H/o impaired hearing in her child for about a month There was no
pain or ear discharge but there was H/o mouth breathing O/E tympanic membranes of both ears
were dull & show air bubbles What is the cause of hearing loss in this child
 CSOM
 Acute Otitis media
 OME (otitis media with effusion) *
 Wax in the ears
 Otosclerosis
124. A young student traveled from Karachi to Islamabad by air while suffering from Upper
respiratory infection. He felt severe earache during landing & blockage of Rt ear What is the
cause of his symptoms
 Acute suppurative otitis media
 OME (otitis media with effusion)
 Aero otitis*
 Herpes Zoster
 Otitis Externa
125. A 19 yr boxer with H/o blunt trauma was diagnosed as a case of Hematoma Left Pinna. What is
the usual location of blood collection?
 Between the perichondrium & cartilage*
 Between the skin & perichondrium
 Between the skin & subcutaneous tissue
 Within the layers of the skin
 Within the cartilage
126. A 4 Yr boy was brought with H/o round & smooth foreign body insertion Lt ear What is the
most suitable instrument to remove this foreign body
 Croccodile force
 Ring probe
 Tilley’s forceps
 Syringing *
 Suction
127. Why the boil (furuncle) ear is very painful
 Hair follicles are close to each other
 Skin of the external ear is more vascular
 Skin of the external ear is thin & delicate
 Skin of the external ear is tightly adherent to underlying cartilage*
 Nerve supply of external ear is mainly pain fibers
128. A young lady was diagnosed as a case of Boil ear What is the most common site to elicit
tenderness
 Mastoid process
 Mastoid tip
 Parotid region
 Tragus*
 Angle of mandible
129. What is the most common Fungus responsible for Otomycosis?
 Aspergillus*
 Candida
 Mucormycosis
 Rhizopus
 Alternaria
130. A 22 Yr lady was diagnosed as a case of Wax Rt ear She also gives H/o perforation Rt ear drum
What is the best method to remove the wax
 Syringing
 Ring probe / Jobson horne probe *
 Croccodile forceps
 Normal saline at room temperature
 Suction clearance
131. A patient was diagnosed as a case of Otomycosis What is the most common symptom by which
patient commonly presents
 Blood-stained ear discharge
 Itching*
 Purulent ear discharge
 Severe earache
 Tinnitus
132. Bullous Myringitis is also known as
 Herpes Zoster oticus
 Malignant otitis externa
 Otitis externa Hemorrhagica*
 Diffuse otitis externa
 Furunculosis
133. Syringing was advised in a 25 yr male patient Which of the following material will be used for
this purpose
 Distilled water at room temperature
 Normal saline at body temperature*
 Tap water at room teprature
 Alcohol
 Methylated spirit
134. What is the most common finding on clinical examination of the ear in a patient of
Otomycosis?
 Soggy blotting paper like material filling the external auditory canal*
 Ear is dry with external ear canal filled with spores
 Inflammed external ear canal with flakes of epithelium sticking to its walls
 Squamous debris filling the external ear canal
 External ear filled with wax
135. In which of the following patient’s Malignant otitis externa is most commonly seen
 Patients who have completed Anti-tuberculosis therapy
 Patients with Osteomyelitis of the mandible
 Patients with poorly controlled Diabetes mellitus*
 Patients with Squamous cell carcinoma of external auditory canal
 Patient with chronic ear discharge
136. A 18 Yr female patient came in OPD with hard & impacted wax in her both ears Which of the
following agents is used to soften the wax (YMDC)
 2% Carbolic acid in glycerin
 2% Potassium chloride in glycerin
 2% Sodium bicarbonate in glycerin*
 2% Sodium chloride in glycerin
 2% Silver nitrate in glycerin
137. Syringing was advised in a 20 Yr female patient for removal of wax What is the least likely
complication
 Facial nerve paralysis*
 Otitis Externa
 Perforation of the tympanic membrane
 Trauma to External ear canal
 Vertigo
138. A 8 Yr boy suffering from Acute otitis media presented with bulging tympanic membrane
despite receiving adequate medical therapy What is the most appropriate treatment option at
this stage
 Continue antibiotics for another 5 days
 Perform immediate cortical mastoidectomy
 Perform myringotomy*
 Perform radical mastoidectomy
 Perform tympanoplasty
139. Following is Not the sign of retracted Tympanic membrane
 Dull Tympanic membrane
 Loss of landmarks
 Loss of cone of light
 Pearly grey shiny tympanic membrane*
 Prominent lateral process of malleus
140. What is the most common age group for Acute suppurative otitis media
 Newborn
 Early childhood *
 Teen aged persons
 Old aged people
 Middle aged people
141. In diagnosis of Tubotympanic type of CSOM What is the main symptom in these patients
 Blood-stained ear discharge
 Mucopurulent ear discharge*
 Scanty ear discharge
 Foul smelling ear discharge
 Vertigo
142. Which of the following diseases if not treated properly can lead to Tubotympanic type of
CSOM?
 Acute otitis media*
 Cholesteatoma
 Mastoiditis
 Otitis externa
 Myringitis bullosa
143. A 6 years girl was diagnosed as a case of Acute Otitis media What is the common early
symptom with which patient comes to the doctor
 Hearing loss
 Ear discharge
 Pain*
 Tinnitus
 Vertigo
144. What is the commonest route of infection in Acute otitis media
 Arterial spread
 Venous spread
 Eustachian tube*
 Grommets (ventilation tubes)
 Lymphatic spread
145. Average length of Eustachian tube in an adult is
 45 mm
 36mm*
 20 mm
 10 mm
 50 mm
146. Cartilaginous part of the Eustachian tube in an adult is:
 ½ of the total length
 1/3 of the total length
 2/3 0f the total length*
 ¼ of the total length
 1/5 of the total length
147. Blockage of the Eustachian tube will lead to (YMDC)
 Positive middle ear pressure
 Negative middle ear pressure*
 No change in middle ear pressure
 Sensorineural hearing loss
 Bleeding from the ear
148. On examination of a 30 Yr female patient showed Attic perforation & granulations in her Rt ear
Which type of discharge is expected in this patient
 Mucopurulent & blood stained
 Mucopurulent & non foul smelling
 Profuse & blood stained
 Scanty, foul smelling & blood stained*
 No ear discharge at all
149. A CT scan of a patient suffering from CSOM showed extensive cholesteatoma & erosion of the
ossicular chain Which surgery is required in this patient. (HBS)
 Atticotomy
 Radical or Modified radical Mastoidectomy*
 Cortical Mastoidectomy
 Tympanoplasty
 Ossiculoplasty
150. Myringotomy was planned in a 7 Yr boy suffering from OME (otitis media with effusion) What is
the best site on ear drum to give incision
 Antroinferior quadrant*
 Antrosuperior quadrant
 Postrosuperior quadrant
 Postroinferior quadrant
 Pars flaccida
151. Which of the following Ear disease complications are most common
 Acute otitis media
 Tubotympanic type of CSOM
 Atticoantral type of CSOM*
 OME (otitis media with effusion)
 Otosclerosis
152. A 10 Yr child was diagnosed as a case of Bezold’s Abscess What is the location of pus collection
in this child
 Mastoid tip
 Petrous apex
 Posterior apex
 Sternoceidomastoid muscle*
 Digastric muscle
153. What is the most common Intracranial complication in CSOM
 Brain abscess
 Extradural abscess
 Meningitis *
 Subdural abscess
 Lateral sinus thrombosis
154. A 25 Yr male has foul smelling ear discharge Rt ear for 4 Yrs He developed severe headache, Lt
sided body weakness, hemianopia & aphasia What is the most likely reason for this
presentation
 Acute Labyrinthitis
 Cerebellar abscess
 Temporal lobe Abscess*
 Extradural abscess
 Meningitis
155. Kerning’s sign is positive in following complication of CSOM
 Cerebellar Abscess
 Lateral sinus thrombosis
 Meningitis*
 Mastoiditis
 Extradural abscess
156. In following ear diseases myringotomy is indicated
 CSOM
 Otitis media with effusion*
 Tympanosclerosis
 Otitis externa
 Otosclerosis
157. A 40 Yr male underwent ear surgery for extensive cholesteatoma middle ear & mastoid What
type of surgery you think was done in this patient
 Myringoplasty
 Myringotomy
 Radical mastoidectomy*
 Cortical mastoidectomy
 Stapedotomy
158. Cortical Mastoidectomy is also called as (IMDC)
 Canal wall up mastoidectomy*
 Canal wall down mastoidectomy
 Atticotomy
 Modified radical mastoidectomy
 Ossiculoplasty
159. In tubotympanic type of CSOM which has become dry with conservative management the
surgical treatment of choice is (RIHS)
 Cortical mastoidectomy
 Myringotomy
 Modified radical mastoidectomy
 Stapedotomy
 Myringoplasty*
160. A 25 Yr female C/O progressive hearing loss both ears for about 10 Yrs No H/O ear disharge or
earache No H/O trauma either O/ E Ear drum is intact both ears, Rinne’s negative both ears
Tympanogram is also within normal limits both ears What is most likely diagnosis in this lady
 Otitis media with effusion
 CSOM
 Ossicular disruption
 Otosclerosis*
 Adhessive otitis media
161. Which part of the ear is diseased in otosclerosis
 Membranous labyrinth
 Otic capsule*
 Saccule
 Utricle
 Semicircular canals
162. A lady who is a diagnosed case of Otosclerosis got pregnant. What will be the effect of
pregnancy on her ear symptoms?
 Symptoms will remain the same
 Symptoms will reduce
 Symptoms will disappear altogether
 Symptoms will increase*
 Symptoms will increase on one side
163. A male patient met a road traffic accident 6 months back He C/O hearing loss in his left ear O/E
both tympanic membranes are intact Rinne’s negative on Lt side & Weber’s lateralized to Lt ear
Most probable diagnosis is
 Otosclerosis Lt ear
 OME (otitis media with effusion)
 Ossicular disruption Lt ear*
 CSOM Lt
 Foreign body Lt ear
164. Phenomenon of Paracusis Willisiiis noted in
 OME
 CSOM atticoantral type
 CSOM tubotympanic type
 Ossicular disruption
 Otosclerosis*
165. Clinical features of Meniere’s disease include
 Episodic vertigo remaining for few seconds
 Vertigo lasting for about a week
 Episodic vertigo lasting for more than 20 minutes*
 Ataxia
 Unconsciousness
166. Which of the following part is involved in Vestibular neuronitis
 Membranous labyrinth
 Scala Vestibuli
 Scala tympani
 Vestibular Nerve*
 Vestibular artery
167. Which of the following disease is characterized by Vertigo with hearing loss
 Benign Paroxysmal Positional Vertigo
 Meniere’s disease*
 Vestibular Neuronitis
 Psychogenic vertigo
 Vertigo after taking Alcohol
168. A 55 Yr male patient was diagnosed as a case of Meniere’s disease What are the main
symptoms by which such patients present
 Ear discharge, Earace & Tinnitus
 Tinnitus , hearing loss & itching
 Vertigo, Hearing loss & Tinnitus*
 Hearing loss & vomiting
 Ataxia, 7th N paralysis & tinnitus
169. A 70 Yr retired bank manager C/O difficulty in understanding speech for about 2 Yrs which is
increasing gradually.O/E Tympanic membranes are intact & Rinne’s is positive both ears. What
is the most likely cause for it (RIHS)
OR
170. A 70 Year male bank manager Came with difficulty in understanding speech which is
progressive for last few years. What is the most likely cause for it (RIHS)
 Meniere’s disease
 Noise induced hearing loss
 Presbyacusis*
 Otosclerosis
 CSOM
171. What is the type of Facial nerve
 Mixed motor & Secretomotor nerve
 Mixed motor & Sensory nerve
 Mixed motor, sensory & Secretomotor nerve*
 Only Motor nerve
 Mixed Sensory & Secretomotor nerve
172. Which of the following disease is most commonly associated with otogenic Facial nerve
paralysis
 Otitis media with effusion
 Cholesteatoma*
 Otosclerosis
 Meniere’s disease
 Tympanosclerosis
173. A 30 Yr lady was diagnosed as a case of Bell’s palsy on Rt side What will be the site of muscle
paralysis of face
 Lower half of Rt side of face
 Upper half of Rt side of face
 Whole Left side of face
 Whole Rt side of face*
 Upper half of Lt face
174. Which of the following is topographic (site of lesion) test for facial nerve
 Brain stem evoked response audiometry
 Electrocochleography
 Schirmer test*
 Electroneuronography
 Pure tone audiometry
175. Which part of the tongue is supplied by Chorda tympani nerve
 Anterior one third of tongue
 Anterior two third of tongue*
 Middle third of tongue
 Posterior two third of tongue
 e. Whole of tongue
176. What is the commonest cause of Facial nerve paralysis
 Bell’s palsy*
 Cholesteatoma
 Mastoid surgery
 Parotid surgery
 Hepes Zoster oticus
177. Most common cause of congenital hearing loss in Pakistan is
 Ototoxic drugs during pregnancy
 CSOM
 Consanguinity (cousin marriage)*
 Ossicular fixation
 Microtia
178. Objective test for hearing evaluation in children is
 Pure tone Audiometry
 Distraction test
 Brain stem Evoked Response audiometry*
 Caloric Test
 Tuning fork tests
179. Treatment of choice in a hearing-impaired child with sensorineural hearing loss is
 To do tympanoplasty
 To do mastoidectomy
 To give a Hearing Aid*
 To do Ossiculoplasty
 To give Vitamins
180. Rinne’s test positive means that: (FMDC)
 Air conduction is better than bone conduction*
 Bone conduction is better than air conduction
 Bone is equal to air conduction
 Cholesteatoma
181. Retracted drum is characterized by all the following except:
 Disturbed cone of light
 Prominent malleolar folds
 Decreased drum mobility
 Central drum perforation*
182. Fistula sign may be positive in:
 Central drum perforation
 Atelactatic drum
 Cholesteatoma *
 Otosclerosis
183. The best treatment of otosclerosis:
 Grommet’s tube
 Stapedectomy*
 Myringoplasty
 Mastoidectomy
184. The middle ear cleft includes all of the following except:
 Mastoid air cells
 Middle ear cavity
 Eustachian tube
 Vestibular aqueduct*
185. Safe type of chronic suppurative otitis media is characterized by:
 Scantly offensive ear discharge
 Profuse mucopurulent ear discharge*
 Marginal perforation
 All of the above
186. The semicircular canals and vestibule are responsible for:
 Hearing
 Conduction of sound waves
 Balance*
 Taste
187. In unilateral conductive hearing loss, Webber's test will lateralize to:
 The diseased ear*
 The normal ear
 Both ear
 None of the above
188. Myringotomy is indicated in:
 Secretory otitis media
 Acute otitis media with bulging drum
 Otitic barotrauma
 All of the above*
189. Stapedectomy is the operation of the choice in:
 Atelactatic middle ear
 Meniere's disease
 Otosclerosis*
 Secretory otitis media
190. Ear wash is indicated in all of the following except:
 Dry central perforation*
 Wax
 Otomycosis
 Caloric test
191. Positive Rinne’s test means: (FMDC)
 Bone conduction is better than air conduction
 Air conduction is better than bone conduction*
 Air conduction is equal to bone conduction
 Dead ear
192. One of the signs of otogenic cerebellar abscess is:
 Hemiplegia
 Hemianesthesia
 Aphasia
 Dysdiadochokinesia*
193. Radical mastoidectomy is indicated in:
 Adhesive otitis media
 Unsafe type of otitis media with dead ear *
 Secretory otitis media
 Central dry perforation
194. Surgical treatment of Meniere’s disease may include:
 Myringoplasty
 Ossiculoplasty
 Radical mastoidectomy
 Vestibular nerve section*
195. Hump nose is managed by:
 Reduction rhinoplasty*
 Augmentation rhinoplasty
 Nasal tip reconstruction
 All of the above
196. Schwartz’s sign may be positive in:
 Atelactatic ear
 Otitic barotrauma
 Secretory otitis media
 Otosclerosis*
197. Radical antrum operation is indicated in:
 Acute sinusitis
 Chronic frontal sinusitis
 Oroantral fistula*
 Chronic sphenoiditis
198. In unilateral conductive deafness, Rinne’s test will be:
 Positive
 Negative*
 Reduced positive
 False negative
199. During flying otitis barotrauma occurs:
 During ascent
 During descent*
 During both
 None of the above
200. Ear wash contraindicated in:
 Wax
 Central drum perforation*
 Otomycosis
 Caloric test
201. The main site of ossicular necrosis in chronic suppurative otitis media is:
 Incudomalleolar joint
 Long process of incus*
 Stapedial arches
 Handle of malleus
202. Radical mastoidectomy is indicated in all the following except:
 Cholesteatoma with dead ear
 Chronic specific otitis media*
 Cholesteatoma with brain abscess
 Acute mastoiditis
203. Meniere’s disease is characterized by:
 Fluctuating hearing loss
 Tinnitus
 Vertigo
 All of the above*
204. Weber’s test in unilateral conductive deafness is lateralized to:
 The sound ear
 The diseased ear*
 Both ears
 Zygoma
205. Otitis media with effusion is caused by.
 Obstruction of the Eustachian tube*
 Ascending infection from upper respiratory tract
 Passage of water through a perforated ear drum
 Blood-born infection
 Spread of infection through fractures
206. The fluid in otitis media with effusion is: (IMDC)
 An inflammatory exudate.
 Secreted by minor salivary glands.
 Produced by the bacteria present in the middle ear
 A transudate from dilated blood vessels due to negative intratympanic pressure*
 Entered from the external auditory canal while bathing
207. What is the surgical treatment of Otitis media with effusion? (IMDC)
 Myringoplasty
 Myringotomy with grommets insertion *
 Cortical mastoidectomy
 Atticotomy
 Atticoantrostomy
208. Which of the following nerves are primarily involved in Gradenigo syndrome? (IMDC)
 Trigeminal and facial nerves
 Trigeminal and abducent nerves*
 Abducent and facial neves
 Facial and vestibulocochlear nerves
 Last four cranial nerves
209. Gradenigo syndrome results from spread of infection to: (IMDC)
 Sigmoid sinus
 Mastoid antrum
 Cerebellum
 Petrous apex*
 Extradural space
210. Facial paralysis in Acute suppurative otitis media results from: (IMDC)
 Pressure due to inflammatory edema*
 Destruction by a cholesteatoma
 Compression due to encroachment by granulation tissue
 Pathological temporal bone fractures
 Toxic metabolites of inflammation
211. A patients with ear discharge presents with picket-fence fever (high grade swinging pyrexia)
and severe headache but no neck stiffness, what is the diagnosis? (IMDC)
 Brain abscess
 Lateral (sigmoid) sinus thrombophlebitis*
 Subdural empyema
 Cerebellar abscess
 Petrositis
212. Cholesteatoma of the middle ear is: (IMDC)
 A sac of keratin debris with keratinizing squamous epithelium*
 A benign tumor fat and cholesterol cells.
 Contain cholesterol crystals
 A granuloma formed around cholesterol crystals
 A benign growth of middle ear mucosa
213. A patient presents with longstanding foul smelling ear discharge and fever, headache,
vomiting, fits and neck stiffness. What is the most likely diagnosis? (IMDC)
 Subperiosteal abscess
 Meningitis*
 Citelli’s Abscess
 Bezold’s abscess
 Lucs abscess
214. Eustachian tube obstruction leads to: (IMDC)
 Tympanic membrane perforation
 Facial palsy
 Gradinego syndrome
 Negative middle ear pressure*
 Acute labyrinthitis
215. A red bulging tympanic membrane in the absence of pain and fever is characteristic of: (IMDC)
 High jugular bulb
 Acute otitis media
 Glomus tumor*
 Carotid artery aneurysm
 Sigmoid sinus thrombophlebitis
216. Which of the following theory is not proposed in the formation of a middle ear cholesteatoma?
(IMDC)
 Negative pressure & retraction pocket
 Basal cells hyperplasia
 Epithelial invasion along margins of perforation
 Congenital rest cells
 Long standing otosclerosis*
217. Which of the following route is not implicated in intracranial spread of a middle ear infection?
(IMDC)
 Direct bone erosion
 Fracture lines
 Congenital dehiscence
 Eustachian tube*
 Blood born
218. Otitis media with effusion is caused by. (IMDC)
 Obstruction of the Eustachian tube*
 Ascending infection from upper respiratory tract
 Passage of water through a perforated ear drum
 Blood-born infection
 Spread of infection through fractures
219. Which of the following is the choice treatment for an otogenic extradural abscess? (IMDC)
 Radical Mastoidectomy by an ENT surgeon*
 Craniotomy by a neurosurgeon
 Burr hole aspiration by a neurosurgeon
 Jointly managed by ENT and neurosurgeon
 Intravenous antibiotics alone
220. A 5 years old boy is brought to you with 3 days history of fever, pain and hearing loss in right
ear. One day back there was some bleeding and discharge from the same ear after which the
pain and fever subsided. On examination there is no tenderness or edema in the external
auditory canal. There is mucopurulent discharge in the External auditory canal with a small
perforation in the tympanic membrane. What is your diagnosis. (IMDC)
 Boil of the ear
 Acute suppurative otitis media*
 Subperiosteal abscess
 Bezold’s abscess.
 CSF otorrhea
221. Multiple exostosis of external auditory canal commonly occurs in (IMDC)
 Outer third of the bony canal
 Inner third of the bony canal *
 Mastoid tip
 Cartilagenous portion of the auditory canal
 Hypotympanum.
222. The most common pathogen in boil (furuncle) of the ear is. (IMDC)
 Proteus mirabilis
 Staphylococcus aureus*
 Streptococcus pneumonia
 Anaerobes
 Haemophylus influenzae
223. A young boy of 14 years comes to you with a two days history of fever, severe pain and
tenderness right ear with mild hearing loss. On examination the tragus was swollen,
erythematous and very tender. The child did not allow examination of the ear because of pain.
What is your diagnosis? (IMDC)
 Acute mastoiditis
 Otomycosis
 Furunculosis (Boil) of the ear*
 Traumatic perforation of tympanic membrane
 Perichondritis
224. A cauliflower ear results from: (IMDC)
 Pseudomonasaeroginosa infection in diabetics
 Blunt trauma to the pinna*
 Congenital anomaly due to maternal use of methotrexate
 Frost bite
 Mastoidectomy operation
225. Which of the following treatment is not recommended in otomycosis? (IMDC)
 Aural toilet
 Topical nystatine
 Topical Nystatine
 Topical salicylic acid in alcohol
 Systemic amphotericin B*
226. A 65 years old diabetic woman presented with severe excruciating pain in her left ear, more
during night time. On examination there were granulations in the bony-cartilaginous junction of
the external auditory canal and facial paralysis. What is the most likely diagnosis? (IMDC)
 Acute mastoiditis
 Malignant otitis externa*
 Otomycosis
 Cholesteatoma of external canal
 Keratosis obturans
227. Which of the following conditions is not associated with otomycosis? (IMDC)
 Hot and humid climate
 Prolonged use of topical antibiotics-steroids preparations
 Open mastoidectomy cavities
 Wax impacted in the external auditory canal*
 Prolong use of hearing aids
228. Which of the following is NOT the treatment option for otosclerosis? (IMDC)
 Hearing aids
 Stapedectomy
 Stapedotomy
 Chemical labyrinthectomy*
 Sodium Fluoride
229. Audiogram of a patient with otosclerosis shows a characteristic (IMDC)
 Roll-over phenomenon
 Carhart’s notch*
 Low frequency mixed hearing loss
 High frequency sensorineural hearing loss
 Poor speech discrimination score
230. In Paracusis Willisii a patient: (IMDC)
 Feels pain with loud sounds
 Feels disequilibrium with loud sounds
 Hears better in quite environment
 Hears better in noisy environment*
 Listens loud sounds but cannot understand
231. Which of the following is not a contraindication for Stapedectomy? (IMDC)
 Meniere’s disease
 Young children under 10 years
 Individuals working in airlines
 Sea divers
 Patients with good speech discrimination score*
232. In a Stapedectomy operation: (IMDC)
 Prosthesis is not used
 Footplate of stapes is removed*
 Footplate is left intact
 Tendon of stapedius muscle is left intact
 Lateral semicircular canal is fenestrated
233. What is the average capacity of an adult maxillary sinus? (IMDC)
 10 ml
 15 ml*
 20 ml
 30 ml
 50 ml
234. Mucus drainage of maxillary sinus is: (IMDC)
 Gravity dependent
 Transported by the cilia*
 Flushed by the natural air current
 Sucked by the negative intranasal pressure
 Mechanically removed by blowing the nose
235. Which of the following pathology is responsible for benign paroxysmal positional vertigo?
(IMDC)
 Vestibular neuronitis
 Acute serous labyrinthitis
 Acute purulent labyrinthitis
 Loose calcium carbonate crystals floating in endolymph*
 Vestibulotoxic drugs
236. A post-stapedectomy patient complains of intermittent vertigo and sensorineural hearing loss
specially after sneezing. According to the patient he did not have the problem before surgery.
What is the most probable diagnosis? (IMDC)
 Meniere’s disease
 Labyrinthine concussion
 Perilymph fistula*
 Acute labyrinthitis
 Vestibular nerve damage
237. A 30 years old male presented of hemoptysis, blood stained nasal discharge, crusting, saddle
deformity and septal perforation. He denied any trauma or surgery of his nose. ESR was raised,
X-ray chest showed multiple lung cavities and urine examination revealed red cells, cast and
albumin. What is the most likely diagnosis? (IMDC)
 Tuberculosis
 Syphilis
 Bronchogenic carcinoma
 Wegener’s granuloma*
 Atrophic rhinitis
238. The most consistent histological finding of Meniere's disease is: (YMDC)
 Dilatation of peri lymphatic compartment
 Dilatation of endolymphatic compartment
 Shrinkage of endolymphatic compartment
 Shrinkage of peri lymphatic compartment
 Dilatation of cochlea*
239. Characteristic feature of traumatic tympanic membrane perforation is: (YMDC)
 Multiple perforations
 Perforation in pars flaccid
 Postero-superior marginal perforation
 Central perforation with regular margins
 Irregular and congested margins *
240. Regarding myringotomy choose the best answer: (YMDC)
 it is done in the antero superior portion of tympanic membrane
 It is done in the postero superior portion of tympanic membrane
 It is done in the postero inferior portion of tympanic membrane
 It is done in the antero inferior portion of tympanic membrane *
 It is done in the pars flaccid
241. Choose the ototoxic drug among below: (YMDC/RIHS)
 Cephradine
 Cefixime
 Sulphonamides
 Cetrizine
 Streptomycin*
242. ASOM is more common in children as: (YMDC)
 They have Short, wide and more horizontal Eustachian tube*
 They have low immunity
 They live in a close community
 They are more prone to infection
 They put their finger in ear
243. Which of the following is true regarding otalgia? (YMDC)
 Only ear is examined carefully to rule out cause Otalgia
 Not only ear but oral cavity, pharynx, larynx and cervical spine should be examined *
 Otalgia is examined on the basis of only investigation
 To treat otalgia pain only painkillers should or given
 Otalgia is always accompanied by ear discharge
244. In which of the following statement is true regarding otosclerosis? (YMDC)
 It is a disease of infants
 It is a disease of children
 It is a disease of 20 to 10 years old
 It is a disease of 30 to 40 years old*
 It is a disease of 40 to 50 years old
245. A 30-year female came with H/O insect in her Rt ear She is disturbed with its movements,
which Ear drops should be used to make the insect unconscious before removal:
 Chloroform ear drops*
 Halothane drops
 Ketamine drops
 Xylocaine drops
 Lignocaine drops
246. What is the usual location of boil in the ear
 Outer half of the external auditory canal
 Outer 1/3 of the external auditory canal*
 Outer 2/3 of the external auditory canal
 Inner 1/3 of the external auditory canal
 Any part of the external auditory canal
247. A 5 yr. boy was brought with severe Rt earache & fever since last night. On examination his Rt
tympanic membrane was bulging & congested What are the common organisms responsible for
this condition? (YMDC/RIHS)
 Staph, E coli & Proteus
 Staph, Morexella & E coli
 Strep pneumonia, H influenzas & M catarrhalis*
 Streptococci, Staph & Pseudomonas
 Staph, Bacteroides & Fungus
248. Which of the following investigation is used to confirm OME (otitis media with effusion)?
(YMDC)
 Tuning fork test
 Pure tone audiometry
 Tympanometry *
 Caloric test
 Evoked response audiometry
249. A 4 Yr. boy was brought with H/O round & smooth foreign body insertion LA ear What is the
most suitable instrument to remove this foreign body? (YMDC)
 Crocodile forcer
 Ring probe
 Tilley's forceps
 Syringing *
 Suction
250. Blockage of the Eustachian tube will lead to: (YMDC)
 Positive middle ear pressure
 Negative middle car pressure*
 No change in middle cur pressure
 Sensorineural hearing loss
 Weeding from the ear
251. A CT scan of a patient with CSOM shows extensive cholesteatoma with erosion of ossicular
chain. which surgery should be planned? (HBS/YMDC)
 Cortical mastoidectomy
 Atticotomy
 Tympanoplasty
 Radical or modified radical Mastoidectomy*
 Posterior tympanotomy
252. A 2 years old child has put a piece of plastic in his ear. The child is quite irritable. What is the
best option? (YMDC)
 Remove foreign body under LA
 Remove with metallic foreign body hook
 Leave and remove after a week
 Remove with help of suction
 Remove with use of microscope under GA *
253. A patient presented with pain Rt ear and let fascia) nerve palsy. Examination showed vesicle on
pinna and the external auditory canal. what is the diagnosis? (YMDC)
 Bell’s palsy
 Malignant otitis externa
 ketosis obturans
 Herpes Zoster oticus*
 Myringitis bullosa
254. The syndrome of sensorineural hearing loss associated with hereditary nephritis is called
(YMDC)
 Alports syndrome*
 Fabry's syndrome
 Nail patella syndrome
 Edward syndrome
 Down syndrome
255. An 18 years old girl is a diagnosed case of Bell's palsy that is not responding to steroids. What
will be the further line of management? (YMDC)
 Increase dose of steroids
 Vasodilators
 Antihistamines
 Electric nerve stimulation
 Surgical decompression *
256. Most common cause of congenital hearing loss in Pakistan is: (YMDC)
 Ototoxic drugs during pregnancy
 Consanguinity ( cousin marriage ) *
 Ossicular fixation
 Microtia
 CSOM
257. Objective test for hearing evaluation in children is (YMDC)
 Pure tone Audiometry
 Distraction test
 Brain stern Evoked Response audiometry*
 Caloric Test
 Tuning fork tests
258. A 18 Yr. female patient came in OPD with hard & impacted was in her both can Which of the
following agents is used to soften the wax
 Carbolic acid in glycerine
 Potassium chloride in glycerine
 Sodium bicarbonate in glycerine *
 Silver nitrate in glycerine
259. Treatment of long-standing case of hematoma Auris is: (YMDC)
 Aspiration by a wide bore needle *
 Incision and evacuation of clot
 Incision and evacuation of clot under aseptic measures
 Dressing of the ear
 Intravenous antibiotic
260. A 21 years old man presented with history of hearing loss in right ear of 9 months duration. Ear
drum is intact but mobility is restricted Rinne’s test is negative on right side and positive left
side. Weber is lateralized towards right side, Mention the type of deafness in right ear (FMDC)
 Conductive*
 Sensory
 Neural
 Mixed
261. Rinne’s test positive means that (FMDC)
 Air conduction is better than bone conduction*
 Bone conduction is better than air conduction
 Bone conduction is equal ta air conduction
 Cholesteatoma
262. Tympanic membrane is normal in appearance in all except (FMDC)
 Presbycusis
 Otosclerosis
 Secretory otitis media*
 Meniere's discase
263. The disease associated with sever Otalgia is (FMDC)
 Acute Suppurative otitis media*
 Otosclerosis
 Labyrinthitis
 Secretary otitis media
264. One best way of treatment of living insect in external auditory canal is (FMDC)

OR

265. One best way of treatment of living insect in (RIHS)


 Kill the insect by instilling ether or chloroform in ear
 Remove under general. anesthesia and microscopic control
 Removal by syringing out in OPD
 Kill with water or oil drops and then remove it*
266. in boil ear, all are correct except (FMDC)
 It is infection of hair follicle caused by staphylococcus hemolyticus*
 Its very painful
 It occurs in outer one third of external auditory canal
 Recurrence is common in
 Less dangerous than boil nose
267. Following complications may result from syringe the ear for wax removal except(FMDC)
 Vertigo
 Perforation of ear drum
 injury to external auditory canal
 Sensorineural deafiness*
268. Treatment of choice low secretary otitis media persisting for more than 6 months is (FMDC)
 Antibiotics
 Myringotomy and grommets*
 Mastnidecomy
269. Canal for tensor tympani muscle is present in which part of middle ear (FMDC)
 The floor
 The anterior wall*
 The posterior wall
 The medial wall
 The lateral wall
270. All of the following are present in medial wall of middle ear except: (FMDC)
 Promontory
 Facial recess*
 Sinus tympani
 Tympanic plexus
 Oval window
271. 10 years boy presents with a 2 day H/O otalgia on the Rt. Side O/E there are blebs in the
external canals and on the TM and the child is in severe pain. There is no hearing Loss and there
is no fever. Management would include (FMDC)
 Discussing with the parents the possibility of VII N paralysis
 Passive immunoprophylaxis with IgG
 SCA bolus of steroids*
 Parental antibiotics
 Analgesic
272. Length of external acoustic canal is (FMDC)
 5cm
 24mm*
 20cm
 42mm
273. Head mirror has focal length (FMDC)
 10mm
 24mm
 25cm*
 100cm
274. All of the drugs are ototoxic except (FMDC)
 Chloroquine
 Cisplatin
 Clotrimazole*
 Furosemide
275. Cochlea is developed sufficiently by (FMDC)
 20 weeks of gestation*
 28 days of life
 37 weeks of gestation
 07 days of life
276. Tympanometry Type B cure is seen in (FMDC)
 Normal adults
 OME*
 Otosclerosis
 Ossicular discontinuity
277. Cause of conductive hearing loss (FMDC)
 Meatal atresia
 Wax ear
 Otitis media with effusion
 All of the above*
278. Example of canal wall up procedure (FMDC)
 Atriotomy
 Modified radical mastoidectomy
 Radical mastoidectomy
 Cortical mastoidectomy*
279. Cone of light can be seen in quadrant of tympanic membrane (FMDC)
 Posterosuperior
 Posteroinferior
 Anterosuperior
 Anteroinferior*
280. Five-year-old child presented with unable to hear satisfacto with occasional episodes of
earache. Pneumatic otoscopy showed bilateral dull and immobile tympanic membrane. is
probable diagnosis (FMDC)
 Glue ear*
 CSOM Tubotympanic type
 CSOM Attic antral type
 Aero-otitis media
281. Following are the parts of middle ear cleft except (FMDC/YMDC)
 Aditus to antrum
 Facial ridge, sinus tympani
 Bony labyrinth*
 Eustachian tube
 Both b & d
282. Regarding furuncles in external auditory canal most appropriate statement is (FMDC/YMDC)
 Occurs in the bony part of external auditory canal
 Cause hearing loss of 40d
 Occurs in cartilaginous part of external auditory canal *
 Obliterate post auricular groove.
 Always Limits Examination of External Auditory canal
283. In noise pollution maximum hearing loss is at frequency (FMDC/YMDC)
 8000-10000H
 2000-4000 Hz*
 6000 Hz
 4000 Hz-6000HZ
 250-500 Hz
284. A boxer develops fluctuating swelling over pinna after getting a punch swelling is non tender
and boxer is afebrile, Diagnosis is? (FMDC/YMDC)
 Haematoma *
 Perichondrial abscess
 Sebaceous cyst
 Frunculosis
 Keloid
285. Most appropriate statement regarding otosclerosis is (FMDC/YMDC)
 Bluish tinge on tympanic membrane
 Sensorineural deafness
 Normal tympanic membrane*
 On Audiometry dip at 4000 Hz
 Occurs mostly in children
286. Regarding chronic suppurative Otitis media which statement is incorrect (FMDC/YMDC)
 Cholesteatoma, granulations, foul smelling discharge are the features of unsafe type
 Rinne's test will be negative in the disease ear
 Positive fistula test indicate erosion of bony labyrinth with exposure of endosteum
 Schwartz sign is always present *
 Mastoiditis, facial paralysis, brain abscess are common complications
287. Ahmed 5 years of age presented with delayed speech, nasal obstruction, tympanic membranes
are dull and retracted Diagnosis is (FMDC/YMDC)
 Bilateral otitis media with effusion*
 Bilateral acute suppurative otitis media
 Bilateral barotrauma
 Bilateral cholesteotoms
 Bilateral wax
288. Which of the following is not true about Meniere's disease (FMDC/YMDC)
 Characterized by recurrent attacks of vertigo*
 Sodium fluoride is given in large doses.
 Usually is bilateral
 Fluctuating hearing loss
 Tinnitus is present
289. All are included in Auditory pathway except (FMDC/YMDC)
 Olivary nucleus
 Lateral geniculate body*
 Inferior colliculus
 Lateral Lamniscus
290. A 20 years old boy presented to THQ emergency with Hx of RTA patient was having occipital
blow he was vitally stable & CT brain was not significant The nezt day patient presented in ENT
opd with no hearing in Rt ear & RI sided facial asymmetry since after head injury o/E Rt
tympanic membrane appears blue with complete facial paralysis. what is probable diagnosis.
(FMDC)
 Extradural heamatoma
 Acute suppurative otitis media
 Temporal bone fracture *
 Lateral sinus thrombosis
 Subdural heamatoma
291. 35 yrs old male patient with history of uncontrolled Diabetes presented with painful vesicular.
Tash eruption on bis Rt pinna & Dec hearing O/E there are tender vesical on Rt pinna concha &
in Rt external auditory canal with Rt sided grade 3 facial paralysis On oral cavity examination
there are also painful vesicles on hard palate & Rt buccal mucosa. PTA shows moderate degree
of SNHL in Rt ear. What is ur diagnosis? (FMDC)
 Malignant otitis externa
 Diffuse otitis externa
 Ramsay hunt syndrome *
 Squamous cell carcinoma of Rt Ear
 both a & d
292. Ossicle or part of ossicle which erodes first in CSOM is? (FMDC)
 Handle of malleus
 Long process of incus *
 Body of incus
 Stapes
 Head of malleus
293. A 70 Years old male presented to the ENT OPD with complaint.of progressive B/Lhearing loss
OJE there is BL intact dull Tympanic membranc with Rinne's positive BL and webber's centralised
PTA shows BL mild-moderate degree of sensorineural hearing loss. What is your most probable
diagnosis
 Acoustic Neuroma
 Otitis media with effusion
 CSOM with Complication
 Presbycusis *
 Noise Induced Hearing Loss
294. 45 yrs old male patient is receiving chemotherapy of NORI hospital for bladder cancer after
some time patient developed tinnitus & hearing loss. which chemotherapeutic agent is
responsible for these symptoms of the patient? (FMDC)
 Methotrexate
 Cyclophosphamide
 Cisplatine *
 5FU
 Dacarbazine
295. In 40 years, female patient with left hearing loss. Rinne's test is negative on the left side. She
has tympanogram of type As. The diagnosis is (RIHS)
 Left Secretory otitis media
 Left Otosclerosis*
 Left tympanosclerosis
 Left Eustachian Tube dysfunction
 Left Ossicular disruptions
296. A 56-year-old patient presents in ENT OPD with complaint of recurrent attacks of vertigo
hearing joss & tinnitus associated with nausea & vomiting. What is the probable diagnosis?
(RIHS)
 Benign Paroxysmal positional vertigo
 Vestibula neuronitis
 Meniere’s diseased*
 Acoustic Neuroma
 Intra cranial complication of CSOM3
297. A 5-year-old child has persistent serous effusions is both ears for last 6 months after acute
middle ear infection. He has a 40-dB conductive hearing losing both ears & having problem in
school What would be the BEST treatment for this child? (RIHS)
 Observe the child for another 3 months
 Prescribe broad spectrum antibiotics for 3 days
 Recommend hearing aid
 Place ventilation tubes in both ears*
 Topical ear drops & regular follow up
298. A 70-year-old woman has pain and deafness in the right ear for 24 hours and a vesicular rash
on the right pinna. Vesicles are also seen in the auditory Canal. What treatment should be
given? (RIHS)
 Gentamycin ear drops
 Oral Acyclovir & Steroids*
 Mastoid exploration
 Oral Amoxicillin and analgesics
 Oral antifungal
299. Most common complication of otitis media with effusion is: (YMDC)
 Acute otitis media
 Adhesive otitis media
 Otitis externa
 Tympanosclerosis*
 Cholesterol granuloma
300. Which of the following statements regarding otitis externa is incorrect? (YMDC)
 May be caused by seborrhea
 May follow chronic suppurative otitis media
 May be allergic in nature
 in its malignant type, haemophiles influenza is the causative organism*
 Treatment includes oral and topical antibiotics
301. Child 4 years of age presented with bilateral nasal obstruction, bilateral otitis media with
effusion and sore throat, diagnosis is: (YMDC)
 Hypermorphic adenoids*
 Tonsillitis
 DNS
 Congenital nasal mass
302. A 45 years old lady came with a right ear discharge workup cholesteatoma was suspected.
Regarding cholesteatoma, which of the following is true? (YMDC)
 It consists of squamous epithelium*
 It is a malignant tumour
 It should he left untreated.
 It may metastasize to distant sites.
 it is mainly treated medically
303. Tympanic membrane develops from: (YMDC)
 Ectoderm.
 Endoderm.
 Mesoderm
 A, B, and C*
 A and C
304. A young boy came to ENT outpatient with the complaint of blocked left car. On otoscopy black
color debris was found in external auditory meatus toilet was performing cough response
caused while cleaning the ear mediated by stimulation of: (YMDC)
 The 5th cranial nerve
 Innervation of external ear canal by C1 and C2
 Branches of the 7th cranial nerve
 The 8th cranial nerve.
 The 10th cranial nerve*
305. A 75 years old man, retired PLA pilot presented with hearing impairment, which of the
following is a cause of sensorineural hearing loss: (YMDC)
 Presbycusis*
 Tympanosclerosis.
 Otosclerosis
 Longitudinal fracture of the temporal one.
 Cholesteatoma
306. A young female mother of two children presented with right middle ear pathology. Weber's
test will be (YMDC)
 Lateralized to right side*
 Positive.
 Lateralized to left side.
 Centralized.
 Normal.
307. A 32 years old female presented with the complaint of vertigo; the most common cause of
peripheral sporadic vertigo is: (YMDC)
 Benign paroxysmal positional vertigo*
 Meniere's discase.
 Acoustic neuroma.
 Vascular occlusion of labyrinthine artery
 Labyrinthitis
308. External auditory canal extends from to tympanic membrane (YMDC)
 Attic
 Lobule
 Isthmus
 Concha*
 Umbo
309. A 28 years old pregnant female in her last trimester came to ENT outpatient, she was agitated
and complained that she earns hear her own breathing movements. On otoscopy her ear drum
was moving with each inspiration and inspiration. Tympanic membrane moves with respiration
in: (YMDC)
 Penurious Eustachian tube*
 Eustachian tube dysfunction
 Otosclerosis
 Secretary otitis media.
 Disconnection of the ossifies
310. A newly born baby was referred for Auditory Brainstem Response resting after it failed
Otoacoustic emission test during a screening phase. Otoacoustic emissions are produced by
(YMDC)
 inner hair cells*
 Basilar membrane
 Auditory nerve
 Outer mast cell
 Utricle
311. A 40 years old nan presented with hearing impairment. His pure one audiometry was which
decreased bone conduction. Decreased bone conduction in an audiogram indicates (YMDC)
 Glue ear
 Damage to cochlea*
 tympanic membrane perforation
 Ossicular dislocation
 Ossicular fixation
312. A 45 years old female belonging to lower socioeconomic status with large family has cough low
grade fever for 5 years. For the last 2 years she has left ear discharge which is purulent.
Otoscopic examination shows multiple periorations.in tympanic membrane. The likely diagnosis
is:
 CSOM tube-tympanic
 CSOM attico-antral type
 Wegner granulomatosis
 Tuberculous otitis media*
 Carcinoma of middle ear
313. A 71 years old male is having decreased hearing and tinnitus for 3 years He is having much
problem in understanding the speech. Rest of ENT examination is normal. Your diagnosis is?
 Presbycusis*
 Meniere’s disease
 Acoustic neuroma
 Otosclerosis
 Glomus tumor
314. A young female is having purulent ear discharge for 3 weeks. Now she also developed severe
headache and post-auricular swelling. The treatment of choice is:
 Oral antibiotics
 Topical antibiotics
 Myringotomy
 Cortical mastoidectomy *
315. A young boy was slapped by his teacher on his left ear. He presented to hospital with earache
and decreased hearing. On examination he was having central tympanic membrane
penetration. What treatment should be advised? (RIHS)
 Topical ear drops
 Oral antibiotics
 immediate Myringoplasty
 Precautions lo keep ear dry*
 Oral antibiotics and antihistamines
316. Endo lymph Is formed in the: (RIHS)
 Scala media*
 Scala tympani
 Utricle
 Endolymphatic sac
 Middle ear
317. What is best treatment option for a young child having nasal obstruction, snoring, decreased
hearing and episodic earache for 6 months (RIHS)
 Analytics, antihistamines and analgesics
 Hearing aids
 Modified radical mastoidectomy
 Adenoidectomy and myringotomy*
 Cortical mastoidectomy
318. A young female presents with deafness and tinnitus for 2 years. She can hear Better in noisy
environment. She denies of ear discharge. The likely diagnosis is (RIHS)
 Barotrauma
 Otitis media with effusion
 Meniere’s disease
 Otitis externa
 Otosclerosis*
319. A 55 years old patient presented to ENT OPD with right car progressive sensorineural hearing
loss, tinnitus and Inability to understand speech. Slid also has facial paresthesia and los of
corneal reflex. Your likely diagnosis is (RIHS)
 Nose induced hearing lass
 Ramiay Hunt syndrome
 Glomus tumour
 Acoustic neuroma*
 Nasopharyngeal carcinoma
320. Best treatment for secretory otitis media for over 6 months (FMDC)
 Myringotomy w/grommet*
 Myringoplasty
321. Most severe otalgia is seen on (FMDC)
 ASOM*
 secretory otitis media
 otosclerosis labyrinthitis
322. Syringing causes nystagmus in all of following except (FMDC)
 Vertigo
 Tinnitus
 sensorineural hearing loss
 external meatus injury *
 TM perforation
323. Tympanic membrane will be normal except: (FMDC)
 Presbycusis
 Otosclerosis
 ASOM *
 Meniere's
324. Right Ear Hearing Loss. Rinne is -ve on right and +ve on left. Weber is lateralized to right ear.
(FMDC)
 Conductive*
 Sensory
 Neural
 Mixed
325. The best treatment of insect in ear is (FMDC)
 Removal by syringing in OPD
 Killing by ether and chloroform and
 then removing
 Killing by water and oil drops and then removing*
 Surgically by myringoplasty
326. All is correct about boil ear except (FMDC)
 caused by staph hemolyticus*
 extremely painful
 less dangerous than boil nose
 injury to the external auditory canal
327. All of the following are found on medial wall of middle ear except (FMDC)
 Tympanic plexus
 Promontory
 Oval window
 Facial recess (on post wall)*
328. Canal for tensor tympani is found on (FMDC)
 Anterior wall*
 Posterior wall
 Medial wall
 Lateral wall
329. A 32-year-old male presented to the ENT outdoor with gradual progressive hearing loss in both
ears for the last 2 years. However, in noisy surroundings he hears better than in quiet
surroundings. There is no history of previous ear discharge, trauma or sudden exposure to loud
sounds. His father also developed hearing loss in the 3rd decade and uses hearing aid. On
examination both the tympanic membranes are intact. On tuning fork tests, Rinne’s is negative
bilaterally. Weber is lateralized towards the Right ear. The most likely diagnosis is (FMDC)
 Chronic suppurative otitis media
 Otosclerosis*
 Serous otitis media
 Impacted wax
 Presbyacusis
330. A mother brings her 3-year-old child to the ENT outdoor with the complaint of mild hearing loss
for the last three months. The parents have to raise their voice to make the child understand
what is being said. There is history of mild earache on and off but no history of ear discharge.
The child snores loudly while sleeping. On examination both tympanic membranes were dull
and opaque with loss of light reflex. His tympanometry was advised. Which curve is expected?
(FMDC)
 A
 As
 B*
 Ad
 C
331. A 22-year-old male presented to the ENT outdoor with sudden onset of right sided earache for
the last 4 days. Earache was preceded by upper respiratory tract infection. On examination, the
Right tympanic membrane is congested with leash of blood vessels along the handle of malleus.
Tuning fork tests show conductive hearing loss in the Right ear. The most likely diagnosis is
(FMDC)
 Acute otitis media *
 Serous otitis Media
 Fungal otitis externa
 Tuberculous otitis media
 Bullous myringitis
332. A 14-year-old girl got her Right pinna pierced 2 days ago. She has new presented to ENT
outdoor with right earache and swelling of the Right ear. On examination pinna is red, hot and
painful. The organism most likely involved is(FMDC)
 Staphylococcus aureus *
 H. Influenzas
 Pseudomonas aeruginosa
 Streptococcus Pneumonia
 Moraxella catarrhalis
333. A 10 years old child presented with 4 days history of severe left sided earache fever and
malaise. He received the treatment but for the last 2 days the condition is worsening with
development of high grade fever and fits. On examination the left tympanic membrane
congested and bulging The appropriate treatment will be: (FMDC)
 Change oral antibiotics and wait for the response
 Cortical mastoidectomy
 Myringotomy and Grommets insertion
 Intravenous antibiotics and nasal decongestants
 Myringotomy and systemic antibiotics *
334. The causative organism for acute suppurative otitis media is: (FMDC)
 Strept. Pneumoniae*
 streptococci
 Staph aureus
 Viral
 All of the above
335. The length of the bony external canal of adult person is approximately (HBS)
 8mm
 12mm
 16mm*
 24mm
336. The tympanic membrane is divided into: (HBS)
 Two equal parts called pars tensa and pars flaccida
 A major upper part called pars flaccida and a small lower part called pars tensa
 A small upper part called pars flaccida and a major lower part called pars tensa*
 None of the above
337. The oval window (HBS)
 Lies below and behind the promontory
 Is closed by secondary tympanic membrane
 Leads to the scala tympani of the cochlea
 Is closed by foot plate of stapes*
338. The sensory end-organ of the semicircular canal is (HBS)
 The organ of corti*
 The macula
 The crista
 None of the above
339. Part of the middle ear cavity above the tympanic membrane is called (HBS)
 Attic
 Epitympanum
 Epitympanic recess
 All of the above*
 None of the above
340. The ratio of the functioning area of the tympanic membrane to the foot plate of stapes is (HBS)
 14:1*
 18:1
 23:1
 1:10
341. The most common route for reaching infection in acute suppurative otitis media is through
(HBS)
 Tympanic membrane perforation
 Veins
 Arteries
 Eustachian tube*
 Grommets
342. In a normally hearing person (HBS)
 Rene's test is negative
 Air conduction is better than Bone conduction*
 Air conduction is equal to bone conduction
 Bone conduction is better than air conduction
343. In a patient with right conductive deafness, the sound of the tuning fork placed on the
forehead is: (HBS)
 Lateralized to the left side
 Lateralized to the right side*
 Sound is equally heard in both ears
 None of the above
344. Rinne's test is negative in all of the following except (HBS)
 Stapedial otosclerosis
 Wax impaction
 Ossicular disruption
 Meniere’s disease*
345. The landmarks of the Tympanic membrane on clinical examination include all of the following
except (HBS)
 The cone of light
 The tympanic annulus*
 The handle of malleus
 The foot plate of stapes
346. Type C Tympanogram is consistent with (HBS)
 Secretory otitis media
 Otosclerosis
 Eustachian tube dysfunction *
 Ossicular discontinuity
347. The expected type of Tympanogram in secretory otitis media is (HBS)
 Туре А
 Type b *
 Туре С
 Type As
 Туре Ad
348. ABR ( Aditory Brainstem Response) is used in (HBS)
 Test of hearing in malingerer
 Test for hearing in retrocochlear lesion
 Detection of acoustic neuroma
 Test for hearing in children and infants
 All of the above*
349. Malignant otitis externa is (HBS)
 Truly malignant disease
 Is most commonly seen in elderly uncontrolled diabetics*
 Staph aureus is the causative organism
 Pseudomonas aeruginosa cannot cause this condition
 Though histologically malignant, behaves as malignant
350. The following organism are involved in acute otitis media EXCEPT (HBS)
 Streptococcus pneumonia
 Haemophillus influenzae
 Pseudomonas aeruginosa *
 Moraxella catarrhalis
351. The type of the hearing loss in otosclerosis may be (HBS)
 Conductive*
 Sensorineural
 Mixed
 All of the above
352. The commonest causes of bilateral sensorineural hearing loss in elderly individual is: (HBS)
 Cochlear otosclerosis
 Presbycusis*
 Diabetes mellitus
 Ototoxicity
 Atherosclerosis
353. A 30 year patient with a recurrent attack of vertigo, hearing loss and tinnitus associate with
nausea and vomiting has: (HBS)
 Benign paroxysmal positional vertigo
 Vestibular neuronitis
 Meniere's disease*
 Acoustic neuroma
 Melkersson Rosenthal syndrome
354. All of the following may be seen in the tuborympanic type of disease (CSOM) EXCEPT: (HBS)
 Mucopurulent otorrhea
 Central perforation of the TM
 Marginal perforation of the TM*
 Profuse otorrhea
 Cholesteatoma pearls in the attic region
355. Cholestetoma is characterized by: (HBS)
 Continuous mucopurulent ear discharge
 A foul selling and scanty discharge
 A central TM perforation
 None of the above*
356. Intermittent fever with rigors and headache in a patient with cholesteatoma may be due to:
(HBS)
 Otogenic meningitis
 Gradenigo syndrome
 Kartegner's syndrome
 Raymsay hunt syndrome
 lateral sinus thrombosis*
357. The combination of unilateral ear discharge, deep seated retro orbital pain and diplopia is
known as: (HBS)
 Piere Robin syndrome
 Graenigo syndrome*
 kliple syndrome
 Trigeminal neuralgia
 Eagle's syndrome
358. A 30-year-old female has presented to the ENT OPD with severe right sided ear ache for the last
two days and right sided facial palsy for the last 1 day. There is no history of ear discharge or
trauma to the ears. Currently there are no complaints of fever, ear or nasal discharge. On
examination there are vesicles on the tympanic membrane, deep meatus, concha and retro
auricular sulcus. The most likely diagnosis is (HBS)
 Acute otitis externa
 Acute suppurative otitis media
 Herpes zoster oticus*
 Malignant otitis externa
 Myringitisbullosa
359. Which of the following frequencies are called speech frequencies. (HBS)
 125, 250 and 500
 250, 500 and 1000c.
 500, 1000 and 2000*
 1000, 2000 and 4000
 2000, 4000 and 8000
360. A 60-year-old diabetic has presented to the ENT OPD with severe right sided ear ache & facial
paralysis for the last two days. On otoscopy granulations are present in the external auditory
canal. The most likely diagnosis is (HBS)
 Attic antral disease
 Herpes zoster oticus
 Malignant otitis externa*
 Otitis externa
 Otomycosis
361. A 14-year-old girl got her right pinna pierced 4 days ago. She has now presented to the ENT
OPD with severe earache & swelling. If this condition is not treated aggressively which of the
following cosmetic deformity is likely to develop. (HBS)
 Bat ear
 Cauliflower ear
 Keloid*
 Macrotia
 Microtia
362. A 30-year-old female has presented to the ENT OPD with the complaints of decreased hearing
in the left ear for the last 6 months. On tuning fork examination, Rinne's is -ve in the left ear &
+ve in the right ear. Weber is lateralized towards the left ear. The diagnosis on the basis of
tuning fork tests is. (HBS)
 Left conductive hearing loss*
 Left sensorineural hearing loss
 Mixed hearing loss
 Right conductive hearing loss
 Right sensorineural hearing loss
363. Benign paroxysmal positional vertigo (BPPV) is a brief but violent attack of vertigo provoked by
certain position of head without auditory symptoms. Which clinical test helps in diagnosis?
(HBS)
 Fistula test
 Caloric test
 Epley’s maneuver.
 Hallpike maneuver*
 Romberg test
364. A female was slapped on left side of face by her husband. Ear examination shows central
perforation. What instruction will you give to the patient? (HBS)
 To have X-ray mastoid.
 Immediate pure tone audiometry
 Daily cleaning of ear canal with cotton bud.
 Do not instill any drops of medicine in the effected ear. *
 Take bath daily
365. Scanty foul-smelling painless discharge from ear is characteristic feature of: (HBS)
 ASOM
 Cholesteatoma*
 otitis externa
 herpes zoster oticus
 otitis media with effusion
366. Gradenigo's syndrome is seen in (FMDC)
 Petrositis*
 Subdural abscess
 Sigmoid sinus thrombosis
 Otosclerosis
367. In pure tone audiometry there is conductive hearing loss.
 Unilateral tinnitus
 May cause facial paralysis
 May involve other cranial nerves
368. A 30 years old male presented in OPO with complaints of decreased hearing in right ear after
swimming for the last 3 days. On otoscopy, right tympanic membrane was obscured by wax in
external auditory canal. Left ear was normal. The expected results of Cunning fork tests in this
patient would be: (RIHS)
 False positive Rinne's in right ear
 Negative Rinne’s test in right ear*
 Positive Rennes test in right ear
 Weber is centralized
 Weber’s test will be lateralized to left car
369. A 28 years old male presents with decreased hearing in both the ears for the last 1 year. There
is no history of ear discharge, vertigo or tinnitus. He is known case of allergic rhinitis and is
taking nasal steroids and anti-histamines. The tympanogram in this patient showed flat curve
with normal ear canal volume. The most likely diagnosis will be: (RIHS)
 Eustachian tube dysfunction
 Ossicular dislocation
 Otitis media with effusion*
 Otosclerosis
 Tympanic membrane perforation
370. A 14 years old boy with longstanding history of discharge from the left ear presents with
earache, fever, anorexia, nausea and rigors He has tenderness and swelling over the left mastoid
area. What's the clinical diagnosis? (RIHS)
 Mastoid abscess*
 Septicemia
 Brain abscess
 Meningitis
 Labyrinthitis
371. A patient who has diminished right sided corneal reflex, has decrease hearing and tinnitus. X-
ray of internal auditory canal reveals widened internal auditory canal on right side. What is
probable? (RIHS)
 BPPV
 Acoustic neuroma *
 Wax
 Ototoxicity
 Otosclerosis
372. A 25 year old man has had repeated episodes of pain and discharge from his right ear since
childhood. He has reduced hearing; the drum has a large perforation with a white mass visit
through the perforation. What is required? (RIHS)
 Topical antibiotics
 Tympanoplasty
 Mastoid exploration *
 Myringoplasty
 Suction clearance and follow up
373. A diagnosed case of acute suppurative otitis media after perforation has developed profuse
discharge, headache and high grade fever. Tenderness is present over mattoid. X-ray shows
haziness of cells, thinning and breaking of walls between the cells What is one best type of
treatment? (RIHS)
 Parenteral broad spectrum antibiotics*
 Myringotomy
 Cortical mastoidectomy
 Modified radical mastoidectomy
 Myringoplasty
374. All of them are false about Vasomotor rhinitis except:
 It is an infective process
 It is an allergic process*
 It is divided in to two groups: eosinophilic and non-eosinophilic
 It is a neoplastic process
 It is an infective and allergic process
375. Bell’s palsy is:
 ipsilateral upper motor neuron lesion.
 Contralateral end upper motor neuron lesion
 Ipsilateral lower motor neuron lesion*
 Contralateral lower motor neuron lesion
 Bilateral upper motor neuron lesion
376. Which of the following causes referred otalgia
 malignant otitis externa
 Dental caries*
 Acute otitis media
 Herpes vaster oticus
 Migraine
377. Referred pain in ear in case of larynx carcinoma Is clue to the involvement of
 Glossopharyngeal nerve
 Vagus nerve*
 Spinal part of accessory nerve
 Cranial part of accessory nerve
 C1 and C2 spinal nerves
378. which of the following is an example of objective tinnitus that can be appreciated by examiner
as well:
 Migraine
 Tumor of eight nerve
 Abnormally patent eustacian tube
 Palatal myoclonus*
 psychogenic
379. In facial nerve palsy, lesion proximal to the level of geniculate ganglion can be demonstrated
by:
 Taste test
 Schirmer test *
 Salivary flow test
 Stepedial reflex by tympanometry
 By asking the patient to look upward and observe for wrinkling over forehead
380. Which of the following is not the cause of otalgia?
 Acute tonsillitis
 Furunculosis
 Otitis media with effusion
 Acute suppurative otitis media
 Meniere disease*
381. Commonest cause of facial palsy is
 Viral
 Bactcrial
 Pressure induced
 Idiopathic*
 Trauma
382. In upper motor neuron facial nerve palsy
 Paralysis of the upper face
 Paralysis of the lower face*
 Paralysis of the whole side of face
 None of the above
 eye closure remain intact
383. Theory of Bell's palsy is
 Vascular
 Viral
 Autoimmune
 All of them*
 none of the above
384. The external shape is maintained by skeletal framework which is composed of
 Bone only
 Cartilage
 Bone in upper 1/3 and cartilage in lower 2/3*
 Bone in upper 2/3 and cartilage in lower 1/3
 Bone in upper ½ and cartilage in lower ½
385. A 2 years old child was brought by his mother to Emergency department with history of sudden
onset of shortness of breath and cyanosis while he was playing with toys. What should be the
immediate step in treatment? (RIHS)
 Parenteral antibiotics
 IV Steroids
 Oxygen inhalation
 Direct laryngoscopy*
 Endotracheal intubation
386. An elderly female while sleeping outside the foam suddenly developed severe pain and itching
In right ear. What should the patient do immediately. (RIHS)
 Go to hospital
 Take good analgesics
 Put some oil in ear*
 Take some good anticipates
 Self removal with any Instrument
387. 14 years old child presented with heating loss just after taking bath. The likely cause is. (RIHS)
 Wax ear*
 foreign body
 Otitis externa
 Otitis media
 OME
388. A patient came to Emergency department with severe itching in left ear mild earache. Ear canal
had blackish material with wet blotting paper appearance, Tympanic membrane was normal.
Most likely diagnosis is. (RIHS)
 Malignant otitis externa
 CSOM
 Acute otitis media
 Carcinoma of external ear
 Otomycosis*
 The following is true regarding the Eustachian tube. (RIHS)
 No angulation is present in infants*
 Size is approximately 24mm in adults
 Forms an angle of 55 degree with horizontal
 Bony part 2/3 cartilage part 1/3
389. A 12 years old child presented with acute otitis media severe earache and post auricle swelling
even after use of antibiotics for 8 days. Best treatment option is. (RIHS)
 Analgesics and antibiotics for 10 days
 Cortical mastoidectomy*
 Only bed rest
 Myringotomy and antibiotics
 Topical antibiotics drop
390. Which of the following statements is true regarding external auditory canal. (RIHS)
 its length I836mm in adults.
 It has an outer 2/3 cartilaginous & inner 1/3 bony part
 Narrowest part is present about 6mm lateral to Tympanic membrane*
 It has an outer 1/3 bony and Inner 2/3 cartilaginous part
 Cartilaginous and bony parts are equal In length
391. An elderly female complains of episodic vertigo, fluctuating hearing loss, aural fullness and
tinnitus. The following investigation is most specific. (RIHS)
 Pure tone audiometry
 Tuning fork tect
 Tympanometry
 Electrocochleography*
 CT scan
392. The following discase causes severe pain in the ear. (RIHS)
 Acute otitis media*
 Otosclerosis
 Meniere’s disease
 Glue ear
 Bell's palsy
393. A child with mucoid ear discharge and subtotal central tympanic membrane perforation for 1
year It a case of (RIHS)
 Attic antral type of CSOM
 Tubotympanic type of CSOM*
 Malignant otitis externa
 Otitis media with effusion
 Otomycosis
394. Myringoplasty means: (RIHS)
 incision in Tympanic membrane
 Reconstruction of tympanic membrane*
 Reconstruction of Ossicles
 Reconstruction of oval window
 Removal of Stones footplates
395. A mother has brought her 3 ½ year old baby for the assessment of hearing which is the one
best tool you will select to asses hearing (RIHS)
 Tuning fork test
 Pure tone audiometry
 Tympanometrydxx
 Evoked response audiometry *
 Electrocochleography
396. Least like complication Following syringing the ear for wax removal is (RIHS)
 Vertigo
 Perforation of ear drum.
 Injury to external auditory canal
 Coughing bout
 Sensorineural deafness *
397. 18 years old female has presented with history intermittent discharge and hearing loss in her
right ear of 5 year duration. On examination there is copious mucopurulent odorless discharge,
coming out of small perforation in the antero inferior segment of tympanic membrane close to
margin. What is your diagnosis: (RIHS)
 Acute mastoiditis
 Chronic SOM
 Acute SOM
 Chronic SOM (Tubo tympanic) *
 Chronic SOM (Atico antral)
398. 65 year old male has presented with severe intractable pain in right ear of 5 days duration He is
on injection of amoxicillin + clavulanic, broad spectrum antibiotic drops and analgesics without
effect examination shows granulation on the floor of external auditory canal at junction
between bone and cartilage Blood and urine reveals uncontrolled diabetes mellitus What is
possible diagnosis (RIHS)
 Squamous cell carcinoma external auditory canal
 Boil ear
 Malignant otitis externa *
 Haemangioma external auditory canal
 Otomycosis
399. Type of hearing loss in Meniere’s disease is (RIHS)
 Neural
 Cochlear*
 Mixed
 Conductive
 Neurogenic
400. Salient symptom of otomycosis is (RIHS)
 Itching in the ear *
 Scanty, foul smelling, blood stained ear discharge
 Severe pain in the ear
 Blocked ear
 Wet blotting paper like mass in external auditory canal
401. 35 year eld male professional deep sea diver has reported with a central type big size dry
perforation in his right ear of six months duration. Right car is totally deaf Left is normal his
living is in his diving profession. Give one best advice/suggestion: (RIHS)
 Change of profession
 Use ear plugs when driving *
 operation of myringoplasty
 Cochlear implant right side
 Combine approach tympanoplasty & ossiculoplasty
402. 40 year old lady has presented with 1 week history of severe itching sense of blockage and
dirty mush discharge from left ear. Examination shows black, wet blotting paper like mass in left
ear. Skin is wet and congested. What is one possible diagnosis (RIHS)
 Wax
 Diffuse otitis externa
 Foreign body
 Otomycosis *
 Malignant otitis externa
403. Following structure/s of the ear is mostly involved in control of balance: (RIHS)
 Tympanic membrane
 Ossiclies
 Eustachian tube
 Cochlea
 Semicircular canal*
404. A boy developed severe earache and discharge from the left 2 days after swimming
examination showed edematous auditory canal. The likely diagnosis is: (RIHS)
 Furunculosis
 Malignant otitis externa
 Diffuse otitis externa*
 Acute otitis media
405. A 36 years old lady is having decreased hearing and pulsatile tinnitus in her right ear. Otoscopy
shows red hue behind intact tympanic membrane. What investigation you will prescribe to
confirm your diagnosis (RIHS)
 CT scan *
 Pure tone audiometry
 Tympanometry
 Electrocochleography
 BERA
406. Which of the following statement is true regarding conductive hearing loss? (RIHS)
 Patient speaks loudly and asks others to speak in a low tone
 Audiometry shows A-B gape*
 Rinne test is positive
 Weber is lateralized to normal ear
 Lesion is between cochlea and cerebral cortex
407. 50 year old male known diabetic came in OPD with complaint of earache and tight ear
discharge for last 4 weeks. On examination the canal was having granulation tissues. What best
medicine you will prescribe to him? (RIHS)
 Oral steroids
 Quinolones*
 Antihistamines
 Cloxacillin
408. Choose the ototoxic drug among below: (RIHS)
 Cephradine
 Ceflxime
 Cetirizine
 Streptomycin*
409. A boy was hit by a car & fell on ground. He came to ENT OPD with difficulty in hearing &
tinnitus. The likely cause Is: (RIHS)
 Tympanic membrane perforation
 BPPV
 Vestibular neuronitis
 fracture petrous temporal bone*
 Otosclerosis
410. After recovering from flu a student developed bilateral earache decreased hearing. Otoscopy
showed congested tympanic membrane. The diagnosis. (RIHS)
 Glue ear
 Boil ear
 Diffuse otitis externa
 Acute otitis media*
 Chronic otitis media
411. what is the investigation of choice for diagnosing a patient of acoustic neuroma? (RIHS)
 CT scan
 MRI*
 Audiometry
 Schirmer's test
 BERA
412. A 45 years old female belonging to lower socioeconomic status with large family has cough &
low grade fever for 5 years. For the last 2 years she has left ear discharge which is purulent.
Otoscopic examination shows multiple perforations in tympanic membrane. The likely
organisms is: (RIHS)
 CSOM tubo-tympanic
 CSOM attico-antral type
 Wegner granulomatosis
 Tuberculous otitis media*
 Carcinoma of middle ear
413. A 71 years old male is having decreased hearing and tinnitus for 3 years. He is having much
problem in understanding the speech. Rest of ENT examination is normal. Your diagnosis is.
(RIHS)
 Presbycusis*
 Meniere’s disease
 Acoustic neuroma
 Otosclerosis
 Glomus tumor
414. A young female is having purulent ear discharge for 3 weeks. Now she also developed severe
earache and post-auricular swelling. The treatment of choice is (RIHS)
 Oral antibiotics
 Topical antibiotics
 Myringotomy
 Cortical mastoidectomy*
 Modified radical mastoidectomy
415. Following are the features of a patient with otosclerosis Except: (RIHS)
 Tympanic membrane is normal
 Rinne's is negative with 512Hz tuning fork
 Both sexes are affected equally*
 Audiogram shows air bone gap
 Paracusis wiliisi and tinnitus
416. A patient is having severe vertigo whenever he lies on bed. The vertigo remains episodic which
lasts for less than a minute. What treatment should be advised to him? (RIHS)
 Antihistamines
 Labyrinthectomy
 Intra-tympanic Gentamycin
 Vasodilators
 Epley's maneuver*
417. Myringotomy is not indicated in the following condition: (RIHS)
 Acute suppurative otitis media
 Chronic suppurative otitis media*
 Otitis media with effusion
 Barotrauma
 Atelactatic ear
418. A patient was admitted in high dependency Medical and Dental College. a call was sent to ENT
department to perform caloric test on the patient. Caloric test determines function of: (YMDC)
 Posterior semicircular canal.
 Saccule
 Utricle
 Lateral semicircular canal *
 Superior semicircular canal
419. A four years old child came to ENT outpatient with. Hearing impairment, his tympanometry
showed a Type B tympanogram, which is found in: (YMDC)
 Normal person.
 Tympanosclerosis
 Otosclerosis
 Secretary otitis media*
 Disconnection of the ossicles
420. Teacher of a six years old boy named Hassam that he doesn't pay attention in class, she
suspected hearing impairment. the common cause for bilateral conductive deafness in a child is:
(YMDC)
 Otitis media with effusion*
 Otosclerosis
 Acute otitis media.
 Congenital cholesteatoma.
 Chronic suppurative otitis media
421. A child aged 3 years presented with see sensorineural deafness, he was prescribed hearing aids
but showed no improvement. What is the next le of management? (YMDC)
 Fenestration
 Stapes mobilization
 Cochlear implant*
 Conservative.
 Mastoidectomy
422. A 20 years old female presented with a foul smelling discharge from her left ear, On
examination fistula test. positive, which is seen in: (YMDC)
 Acute mastoiditis
 Erosion in the basal turn of cochlea*
 Erosion of the lateral canal.
 Acute petrositis
 Adhesive otitis media.
423. On otoscopy a normal tympanic membrane bears one of light, it points: (YMDC)
 Posteroinferiorly.
 Superiority.
 Anterioinferiorly*
 Posteriority.
 Centrally
424. A 20 years old boy had running nose followed by severe earache. One day later he had
mucopurulant discharge from ear, Diagnosis is: (YMDC)
 Malignant otitis externa
 Acute suppurative otitis media*
 Eustachian catarrh.
 D Barotrauma
 E. Myringitis Bullosa
425. A 10 year old boy complaints of reduced hearing loss since two months. On examination bone
(…) is than air conduction in left ear and Weber is lateralized to Left side. How would you
interpret these findings? (YMDC)
 Conductive loss in left ear
 Conductive loss in right ear
 Sensory loss in left ear
 Sensory loss in right ear
 Both sensory plus conductive loss in left ear*
426. A 45-year-old female treated for ovarian cancer develops difficulty hearing. Which of the
following agents most likely caused these findings? (YMDC)
 Paclitaxel
 Doxorubicin
 Bleomycin
 5-FU
 Cisplatin*
427. Which of the following conditions causes the maximum hearing loss? (YMDC)
 Ossicular disruption with intact tympanic membrane
 Disruption of malleus and incus as well tympanic membrane*
 Partial fixation of the stapes footplate
 Otitis media with effusion
 Tympanosclerosis
428. A young lady was a known case of Meniere’s disease which of the following is not a typical
feature of Meniere’s disease. (YMDC)
 Sensorineural deafness
 Pulsatile tinnitus*
 Vertigo
 Fluctuating deafness
 Aural fullness
429. A lady has bilateral hearing loss since worsened during pregnancy. Type of impedence
audiometry graph will be: (YMDC)
 Ad
 As*
 B
 C
 A
430. A 37-year-old intravenous drug user is referred by his GP for a suspected Bell’s Palsy. On
examining his inner ear there are a number of vesicles visible on his ear drum. His cranial nerve
examination reveals a weak of the left side of his face. The most likely diagnosis is: (YMDC)
 Steven Johnson Syndrome
 HIV
 Stroke
 Ramsay Hunt Syndrome*
 Bell's Palsy
431. A mother brought her 4 year son with history of insertion of a rounded bead in his Left ear.
What is the most common site for impaction of such foreign body? (RIHS)
 Cartilaginous-bony junction
 Cartilaginous part of external auditory meatus
 Concha
 Isthmus of external ear canal*
 On the tympanic memantine
432. A young male had blunt trauma on Left eye presented with diplopia, ecchymosis of lid and
enophthalmos. X-ray Water's view showed 'tear drop opacity'. The type of fracture is. (RIHS)
 Nasal bone fracture
 Zygoma fracture
 Blow out fracture*
 Mandibular fracture
433. Which of the following is Not true regarding Ramsay-Hunt syndrome. (RIHS)
 Vesicular rash In external ear
 Severe earache
 Paralysis of lower half of muscles of the face on one side*
 Mostly followed by Shingles.
 Treatment is conservative
434. A two year old male child is brought by his mother for consultation as child has failed to
develop speech Antenatal, perinatal and postnatal history is normal. Physical milestone
development is normal. Pinna, external auditory canal and tympanic membrane is normal what
one investigation would you like to carry out on this child (RIHS)
 CT scan middle and inner ear
 Tympanometry
 Brain stem Evoked response Audiometry *
 Distraction test
435. 55 year old male is suffering from Meniere’s disease in right ear for last 15 years. Medical
treatment has failed to control vertigo. Select the one best choice of surgery. Audiogram shows
95 decibel hearing loss in right ear and left ear normal (RIHS)
 Endolymphatic sac decompression.
 Endolymphatic sac and post cranial fossa shunt
 Mannootomy and grommet insertion
 Total labyrinthectomy*
 Superior cervical sympathectomy
436. Which one character of vertigo is specific to Meniere’s disease (RIHS)
 Precipitated by turning side in bed with affected ear undermost
 It is relieved by tying down & closing the eyes
 It is present continuously for three days
 It is sudden, dramatic, spontaneous and may last 30 minutes to 24 hours *
 Duration of vertigo never exceeds from few sec to 30 minutes
NOSE :
1. Anti-cholinergic (topical ipratrobium bromide) is used in the treatment of: (HBS)
 Allergic rhinitis*
 Acute rhinitis.
 Chronic rhinitis
 Non- eosinophilic vasomotor rhinitis.
 Eosinophilic vasomotor rhinitis.
2. A 60-year-old male patient presented with history of nasal blockage with epistaxis and foul-
smelling discharge from left nostril for last 3 months, the is a history of significant weight
loss, which of the following is most likely a diagnosis? (HBS)
 Antrochoanal polyp
 Foreign body nose
 Sinonasal carcinoma*
 Fungal sinusitis
 Simple nasal polyp
3. Blow out fracture of orbit involves (HBS)
 Superior orbital fissure
 Optic canal
 Lateral wall of orbit
 Medial orbital wall
 Floor of orbit * ( infero-medial part )
4. The best investigation in antrochoanal Polyp (HBS)
 smell identification test
 Skin prick test.
 Waters view X-ray.
 x ray postnasal space lateral view*
 Coronal CT scan.
5. A 25 year old male patient underwent Sub mucosal resection, a day later he developed high
grade fever, lethargy and drowsiness, his blood pressure is 96/60mmhg. the main cause of
this condition is due to (HBS)
 Toxic shock syndrome*
 Aspiration pneumonia
 Excessive nasal bleed
 Hypersensitivity reaction
 Trauma to skull base during surgery
6. A thirty-year-old male patient has history of nasal bleeding and haematuria for the last two
years. He also complains of productive cough. On examination septal mucosa looks very
granular anteriorly. On investigations ESR was found to be 60 mm. after 1 hour and
urinalysis showed red cell casts and epithelial cells. The most likely diagnosis in this patient
is: (HBS)
 Tertiary syphilis
 Osler's disease
 atrophic rhinitis
 Wegener granulomatosis*
 Allergic Rhinitis
7. The most common cause of epistaxis is (HBS/YMDC)
 Fracture nasal bone
 Angiofibroma
 Idiopathic *
 Hypertension.
 Blood diseases
8. The most common cause of epistaxis in children: (HBS)
 Fingernail trauma*
 Fracture nasal bone
 Bleeding disorders
 Angiofibroma
 Hereditary hemorrhagic telengectesia
9. Prolonged use of the vasoconstrictor nasal drops result in (HBS)
 Atrophic rhinitis
 Rhinitis medicamentosa*
 Hypertrophic rhinitis
 Rhinitis sicca
 infective rhinitis
10. A 40 years old male presented to ENT OPD with left sided nasal mass, he is suspected to
have Inverted Papilloma. Following is the treatment of choice in his case. (HBS)
 Medial maxillectomy*
 Trans antral exenteration
 cald wel luc surgery
 Endoscopic removal
 Intra nasal excision
11. The antrochoanal polyp: (HBS)
 The patient, commonly a young adult, complains of bilateral nasal obstruction
 The patient, commonly only a young adult, complains of unilateral nasal obstruction which
is worse on expiration*
 The patient, commonly an old female, complains of unilateral nasal obstruction which is
worse on inspiration
 The patient, commonly an old male, complains of unilateral nasal obstruction which is
worse on inspiration
 The patient, commonly an old female, complains of unilateral nasal obstruction which is
worse on expiration
12. All structures open in the middle meatus except (HBS/YMDC)
 The frontal sinus.
 Posterior ethmoid air cells*
 The middle ethmoid air cells
 The maxillary sinus.
 The anterior ethmoid air cells.
OR

13. All structures open in the middle meatus except: (HBS)


 Sphenoid sinus*
 The maxillary sinus.
 The frontal sinus
 The anterior ethmoid air cells
 The middle ethmoid air cells.
14. A 20-year-old female presented to ENT outpatient department with a history of foul-
smelling discharge from nose, along with recurrent episodes of epistaxis and nasal blockage.
On examination, bilateral nasal cavities appear roomy and full of crusts. which of the
following is the most relevant management? (HBS)
 submucosal resection
 young's operation*
 partial turbinectomy
 submucosal diathermy
 septoplasty
15. Mucocele as a complication of infection commonly develops in which paranasal sinus? (HBS)
 Ethmoidal sinus
 Sphenoid sinus
 cavernous sinus
 Maxillary sinus
 Frontal sinus*
16. A 30-year-old male patient presented with nasal furunculosis; the most common organism
responsible for this disease is: (HBS)
 staphylococcus aureus*
 streptococcus pneumonae
 E. Coli
 streptococcus pyogenes
 pseudomonas aeruginosa
17. A 14-year-old boy has massive recurrent epistaxis and left side nasal obstruction for 6
months. On nasal examination he has a fleshy red mass in left nasal cavity filling the nostril.
The most probable diagnosis is. (HBS)
 nasopharyngeal carcinoma
 Rhabdomyosarcoma of nose
 Squamous cell carcinoma of nose
 Haemangioma
 Nasopharyngeal angiofibroma *
18. Rhinophyma is (HBS)
 a benign tumor*
 associated with nasal furunculosis
 a malignant tumor
 associated with acne rosacea
 arise from nasal septum
19. Best treatment line in allergic rhinitis is: (HBS)
 Local vasoconstrictor.
 Allergen avoidance*
 Local antihistamine.
 Systemic corticosteroid.
 Local corticosteroid.
20. Which of the following is an IgE mediated hypersensitivity disease of the mucous membrane
of the nasal air way? (HBS)

OR

21. It is an IgE mediated hypersensitivity disease of the mucous membrane of the nasal air way:
(HBS)

OR

22. An IgE mediated immunological response of body to the airborne allergens is termed as
(HBS)
 Chronic rhinitis.
 Vasomotor rhinitis.
 Acute rhinitis.
 Atrophic rhinitis
 Allergic rhinitis*
23. Saddling of nose can be corrected by (HBS)
 Collumella plasty
 Augmentation rhinoplasty*
 Reduction Rhinoplasty
 Tip plasty
 SMR
24. A healthy 15 years male patient presented to the ENT department with right sided nasal
obstruction, foul-smelling and blood-stained discharge. The most likely diagnosis is: (HBS)
 Foreign body.
 Rhinolith*
 Angiofibroma
 Simple nasal polyp.
 Antrochoanal polyp.
25. 45-year-old male presented to ENT outpatient department with history of clear fluid coming
from the nose, the discharge is continuous and cannot be sniffed back, there is a history of
head injury 2 weeks back. Which of the following is the most appropriate diagnosis? (HBS)
 acute sinusitis
 chronic granulomatous disease
 allergic rhinitis
 cerebrospinal fluid rhinorrhea*
 nasal polyps
26. Treatment of choice for recurrent antrochoanal polyp is. (HBS)
 antrostomy
 Ethmoidectomy
 Intra nasal Polypectomy
 Cald well luc*
 Proof puncture
27. A 40-year-old man who is on steroids developed boil in the nose. Two days later he
developed swelling in eye lids on one side with chemosis and opthalmoplegia of
extraoccular muscles. He also has fever. Which investigation will clinch the diagnosis? (HBS)
 X-ray para Nasal Sinuses
 ESR
 CT scan*
 Random blood sugar
 Total leukocyte count
28. 17-year-old male patient presented to emergency with history of trauma to face, there is
evidence of fracture to the nasal bone, what is the best time for manipulation and reduction
of nasal bone trauma (HBS)
 between 7-21 days
 between 14-21 days
 between 3-7 days*
 between 7-10 days
 after 21 days
29. Orbital complications usually arise from (HBS)
 Acute ethmoiditis*
 Acute frontal sinusitis
 nasal furunculosis
 Acute maxillary sinusitis
 Acute sphenoiditis
30. An 18 years old male was hit on his nose by fist during quarrel last night. He developed
bilateral nasal obstruction by morning time. The only and significant finding is bilateral dark
red, fluctuant swelling occupying anterior part of nasal cavity. Which one is the best line of
treatment? (HBS)
 Aseptic aspiration, antibiotics and analgesic
 Analgesic, oral and topical decongestant drugs
 Aseptic aspiration, nasal packing and antibiotic*
 watchful waiting
 Incision and drainage and antibiotics
31. A young boy of ten years age went for swimming. Next day he developed nasal congestion,
fever and headache. He deteriorated over next 24 hours and developed redness and edema
of both eyelids on left side. His likely diagnosis is (HBS/IMDC)
 Preseptal cellulitis
 Acute maxillary sinusitis
 Erysipelas
 Cavernous sinus thrombosis*
 Acute Ethmoiditis
32. A young patient has severe nosebleed. On examination he has multiple bleeding spots on
nasal septum. He also has multiple telangectasia on tongue. His father also had similar
problem. His bleeding is due to; (HBS)
 Hypertension
 Idiopathic varicosities
 endu-Ostler-Weber syndrome*
 nasopharyngeal angiofibroma
 Nasal picking
33. The poorly treated septal hematoma can lead to which complication in long run (HBS)
 Retracted columella
 Deflected nasal septum
 Septal abscess*
 Nasal tip deformity
 Nasal saddling

OR

 Saddle nose deformity*


 Epistaxis
 Nasal polyps
 Deflected nasal septum
 Retracted columella
34. The nasal vestibule is anterior most part of nasal cavity. The characteristic feature of nasal
vestibule which has clinical significance is (HBS)
 Lined by squamous epithelium
 It has thermal receptors
 It has rich vascularity
 It is small area
 It contains hair*
35. A 7-year-old girl starts unilateral nose bleed at school. Which of the following measures can
be used to stop her bleeding during shifting to hospital? (HBS)
 Apply vasoconstrictive
 drops Injection transamine
 Pack her nose with cotton
 Pinch her nose*
 Make her sit upright
36. A patient has been put on IV heparin for pulmonary embolism. He started nosebleed. His PT
is 50 seconds. His treatment is (HBS)
 Give blood transfusion
 Give transamine
 Give protamine sulphate*
 Vitamin k
 Give normal saline
37. A 45 year old male known case of dengue fever admitted in medical ward , there is a
complaint of nasal bleed from bilateral nasal cavity, which of the following investigation is
most helpful to find the cause of epistaxis in this patient (HBS)
 Activated partial thrombin time
 Blood complete picture *
 Dengue serology
 Prothrombin time
 Clotting time
38. 3-year-old uncooperative child presented to ENT outpatient with history of insertion of
battery cell in right nostril, the appropriate management in this case would be (HBS)
 immediate removal by foreign body hook under general anesthesia*
 removal with forceps in out patient department
 push in nasopharynx for its removal through throat
 watchful waiting for its spontaneous removal
 removal by syringing
39. A 50-year-old male patient presented to ENT outpatient department with a history of nasal
discharge that is clear, watery and increased on coughing straining and bending forward.
There is a history of some nasal surgery 2 weeks back. The most appropriate investigation
for diagnosis will be: (HBS)
 nasal biopsy
 xray paranasal sinus
 B2 transferrin test*
 blood complete picture
 serum IgE level
40. 25 year old patient presented to ENT outpatient department with complaint of external
nasal deformity, there is a history of nasal trauma 2 year back, on examination there is
obvious hump over the nasal dorsum with normal bilateral nasal cavity, what will be the
appropriate management (HBS)
 augmentation rhinoplasty
 septoplasty
 tip plasty
 reduction rhinoplasty*
 septorhinoplasty
41. A 10-year-old kid hit on the nose by ball while plying cricket 5 days back presented with
severe pain and tenderness on the bridge on the nose along with fever and bilateral nasal
obstruction. Anterior rhinoscopy shows bilateral smooth swelling of the nasal septum. This
patient has (HBS)
 Cavernous sinus thrombosis
 Septal abscess*
 DNS secondary trauma
 Septal haematoma
 Bilateral haemangioma
42. One of the patients admitted in the ENT ward for treatment of boil of the nose developed
severe unilateral headache with bouts of vomiting one day after admission. This patient,
also complained of bluring of same side of vision with change of body sensations. This
patient has developed which of the following complications (HBS)
 Cavernous sinus thrombosis *
 Extra dural abscess
 Meningitis
 Acute fronto ethmoiditis
 Orbital abscess
43. A young boy was involved in road accident and sustained facial trauma. He was seen
emergency room and send home on pain killers. He returned to ENT out patients three days
later complaining of high-grade fever, pain and nasal obstruction. On examination there was
tenderness over supra tip area. His leukocyte count was markedly raised. What is the most
likely diagnosis? (HBS)
 Laceration of inferior turbinate
 Nasal bone fracture
 Septal abscess*
 Cavernous sinus thrombosis
 Septal hematoma
44. A 45-year-old lady presented in ENT out patients with complaints of anosmia and nasal
blockage. She has also noticed that people have started avoiding her in gatherings. She gives
history of some surgical operation on her nose 27 years ago. What is the most likely
diagnosis? (HBS)
 Atrophic rhinitis*
 Allergic rhinitis
 Rhinolith
 Infective rhinitis
 Squamous cell carcinoma of maxillary sinus
45. Rhinitis medicamoentosa is caused due to overuse of (HBS)
 Topical corticosteroids
 Topical decongestants*
 Systemic decongestants
 Systemic corticosteroids
 Mast cell stablizers
46. A 45 year old male presented with unilateral nasal blockage. On examination there is a firm
mass in left nasal cavity, biopsy of the lesion taken and it find out to be Inverted papilloma.
The lesion is (HBS)
 Malignant carcinoma of nose
 Benign lesion of the nose*
 Inflammatory nasal lesion
 Chronic granulomatous disease of nose
 Acute infective condition of nose
47. A thirty-year-old girl presented with fluctuant swelling over forehead, with fever and frontal
headache. The is history of purulent nasal discharge from bilateral nasal cavity for last 10
days. The most probable diagnosis is (HBS)
 Pyocoele*
 pots puffy tumor
 intracranial complication of sinusitis
 preseptal cellulitis
 frontoethmoidal mucocele
48. Round, smooth, soft, translucent, yellow or pale, glistening structure which result from
prolapsed lining of the ethmoid sinus and blocks the nose to variable degree depending on
their size: (HBS)
 simple nasal polyp*
 Antrochoanal polyp.
 Nasal turbinate
 inverted papillorna.
 Furuncolosis.
49. A three-year-old child presents with unilateral foul smelling blood stained discharge from
nose for 4 week the most probable diagnosis is. (HBS)
 Foreign body in nose*
 Nasal Myiasis Nasal polyp
 Rhinolith
 Infective rhinitis
50. The authentic test to detect CSF in nasal secretion is (HBS)
 csf pressure measurement
 B2 transferrin test*
 potassium level
 Measuring sugar level
 Measuring NaCl2 level
51. Most common organism seen in case of sinusitis following upper respiratory infections is:
 Hemophilus influenza
 Moraxella catarahis
 Streptococcus pneumonia*
 Pseudomonas aerogenosa
 Staphylococcus aureus
52. 25 year old patient presented to ENT outpatient department with complaint of external
nasal deformity, there is a history of nasal trauma 2 year back, on examination there is
obvious hump over the nasal dorsum with normal bilateral nasal cavity, what will be the
appropriate management
 tip plasty
 septoplasty
 reduction rhinoplasty*
 septorhinoplasty
 augmentation rhinoplasty
53. Patient develop swelling of left cheek following upper molar tooth excision. The appropriate
diagnosis is:
 furunculosis
 acute maxillary sinusitis *
 sinonasal malignancy
 Ludwig's angina
 parapharyngeal abscess
54. A 20 year old female presented with history of foul smelling discharge from nose, along with
recurrent episodes of epistaxis and nasal blockage. On examination, bilateral nasal cavities
appear roomy and full of crusts. Which of the following is the most relevant management
 Partial turbinectomy
 Submucosal resection
 Submucosal diathermy of turbinates
 Young's operation*
 Septoplasty
55. Metallic foreign body of nose is best treated via
 trans antral removal
 lateral rhinotomy incision
 through foreign body hook *
 syringing
 suctioning
56. A 20-year-old male patient presents with history of watery discharge sneezing from the nose
for the last 2 years. He also says he is sensitive to strong smell specially perfume and
tobacco smoke. He is known asthmatic with history of aspirin in tolerance. Which of the
following drugs is helpful in management?
 antibiotics
 corticosteroids*
 analgesics
 beta blockers
 antidepressants
57. A thirty-year-old man complains of pain in nose along with occasional minor epistaxis.
Recently he also noted pain in right hip joint. On examination the nasal septa) mucosa looks
granular. Biopsy of this mucosa showed granulomatous inflammation.
 Chest X-ray
 ESR
 C-ANCA (antinuclear cytoplasmic antibodies)*
 Urine microscopy
 nasal biopsy
58. In seasonal rhinitis, the early symptom will be:
 Facial pain
 Paroxysms of sneezing occur at frequent intervals throughout the day*
 Watery rhinorrhea.
 Loss of smell and loss of taste.
 Post nasal drip is not common.
59. In perennial rhinitis the commonest cause is:
 Fungi spores.
 House dust mites*
 Tree pollen.
 Grass pollen.
 Flour dust.
60. A 20 years' male presented with high grade fever, headache, and bilateral nasal obstruction
for 2 days. Diagnosis is (IMDC)
 Bilateral Ethmoidal polyps.
 Bilateral inferior turbinate hypertrophy
 Acute sinusitis*
 Atrophic sinusitis
 Allergic Rhinitis
61. A 10 years' boy presented recurrent small bleed. Bleed was anterior. Which of the following
arteries is not involved? (IMDC)
 Ant.ethmoidal artery
 Greater palatine artery
 Post. Ethmoidal artery*
 Sup. Labial artery
 Sphenopalatine artery
62. A 20 years' male undergone surgery for septal deflection. SMR done. Which is the most
common complication of this procedure? (IMDC)
 Septal perforation
 Toxic shock syndrome
 Septal hematoma
 Saddle deformity*
 CSF rhinorrhea
63. A 25 years' male presented with crusting and epistaxis after SMR. Diagnosis is? (IMDC)
 Synachae
 Large perforation
 Carcinoma
 Small perforation
 Atrophic rhinitis*
64. A 35 years' male presented with fever, epistaxis, hematuria, ESR was120. Granulations were
seen on nasal septum. Most common diagnosis is (IMDC)
 Tuberculosis
 Wegner's granuloma*
 Midline lethal granuloma
 Leprosy
 Syphilis
65. A 11Years' old boy presented with recurrent epistaxis. Most common cause of epistaxis is
(IMDC)
 Tuberculosis
 Hypertension
 Idiopathic
 Trauma*
 Malignancy
66. A 60 years' male presented with repeated epistaxis from Right nasal cavity for last 7 years.
Mass was in Right nasal cavity. CT scan showed mass filling maxillary sinus. Most probable
diagnosis is (IMDC)
 Transitional cell papilloma
 Malignant growth of maxillary sinus*
 Angiofibroma
 Nasopharyngeal carcinomas
 Mucormycosis
67. Earliest nerve to be involved in a patient with unilateral SNHL, Tinnitus and Mass at CP angle
on MRI: (IMDC)
 Vth nerve *
 VIth nerve
 XIth nerve
 XIIth nerve
 IXth nerve
68. A child of 8 years presented with nasal trauma, pain and bilateral nasal blockage. Patient has
a low-grade fever. What is the probable Diagnosis? (IMDC)
 Septal hematoma
 Fracture nasal bones
 Septal abscess *
 Blood clots
 Septal perforation
69. A 40 years' male comes to Emergency Room with severe respiratory distress, Immediately
the best & easiest way to restore airway is (IMDC)
 Airway placement
 Emergency Tracheostomy*
 Endotracheal intubation
 Laryngotomy
 Elective tracheostomy
70. The little’s area is supplied by: (IMDC)
 Superior labial artery, anterior Ethmoidal artery, greater palatine artery and spheno-
ethmoidal artery*
 Superior labial artery, posterior Ethmoidal artery, greater palatine artery and spheno-
ethmoidal artery
 Superior labial artery, anterior Ethmoidal artery, lesser palatine artery and spheno-
ethmoidal artery.
 Inferior labial artery, anterior Ethmoidal artery, greater palatine artery and spheno-
ethmoidal artery.
 None of the above.
71. Inflammation of the external nose: (IMDC)
 Furunolosis is due to streptococcus infection
 Furunculosis cannot be complicated by cavernous sinus thrombosis.
 Usually painless.
 Needs many investigations for diagnosis.
 Infections should be treated with broad spectrum antibiotics.*
72. All of the following are causes of bilateral nasal obstruction except: (IMDC)
 Adenoids
 Allergic rhinitis.
 Antro-choanal polyp*
 Ethmoidal polyp.
 Atrophic rhinitis.
73. All of the following are complications of sinusitis except: (IMDC)
 Cavernous sinus thrombosis
 Orbital cellulitis
 Toxic shock syndrome.
 Glomerulonephritis. *
 Pott's puffy tumor.
74. The diagnostic test of Allergic Rhinitis is (IMDC)
 Blood CP / ESR
 Serum Ig E
 Skin prick test*
 Nasal provocation test
 RAST
75. A 5 year old patient is scheduled for tonsillectomy. On the day of surgery, he had runny
nose, temperature 37.5°C and dry cough. Which of the following should be the most
appropriate decision for surgery? (IMDC)
 Surgery should be cancelled for one-week treatment of paracetamol,
 Can proceed for surgery if chest is clear and there is no history of asthma
 Should get X-ray chest before proceeding for surgery
 Cancel surgery for 3 weeks and patient to be put on antibiotics*
 No treatment required. Do surgery. As it is viral infection.
76. The most common bacteria leading to Acute Rhinosinusitis is(IMDC)
 Streptococcus pneumonia *
 Haemophilus influenza
 Moraxella catarrhalis
 Staphylococcus specie
 E- coli
77. Which of the PNS is most commonly involved in malignancy? (IMDC)
 Maxillary *
 Ethmoid
 Frontal
 Sphenoid
 All of the above
78. A patient having recurrent maxillary sinusitis having history of first premolar extraction most
common cause is (RIHS/FMDC)
 Upper alveolar carcinoma
 Oroantral fistula*
 Dental cyst
 Carcinoma of right maxillary sinus
 Tooth infection
79. Which of the following Para nasal Sinuses is most commonly involved in malignancy?
(RIHS/FMDC)
 Anterior Ethmoidal
 Frontal
 Maxillary*
 Sphenoid
 Posterior Ethmoidal
80. A 24 years old male is suffering from chronic maxillary sinusitis which is refractory to medical
treatment. You have decided antral lavage under local anesthesia. Which is one best site
you will select for perforating with trocar and cannula? (RIHS/FMDC)
 Middle meatus at hiatus
 Through natural ostium of maxillary sinus
 Through canine fossa
 Through inferior meatus*
 Through superior meatus
81. Foul smelling unilateral nasal discharge in a child is commonly due to (IMDC/RIHS/FMDC)
 Maxillary sinusitis
 Foreign body nose*
 Antrochonal polyp
 Adenoids hypertrophy
 Atrophic rhinitis
82. A young boy presents with epistaxis what is the first aid treatment (RIHS)
 pressure and pinch bridge of nose
 Anterior nasal packing
 Ice packs
 Pinch the soft part of nose
 Pinch the soft part of nose, ice packs*
83. A boy has hit on his head, presents with epistaxis and rhinorrhea on bending head forward
(RIHS/FMDC)
 CSF Rhinorrhea*
 Raised intracranial pressure
 Allergic rhinitis
 Tumour nasopharynx
 Sinusitis
84. A 17 years old boy had operation of SMR. On removal of nasal packing on 3 rd post operative
day he complained of bilateral nasal obstruction as before operation. What is one possible
cause? (RIHS/FMDC)
 In adequate operation
 Postoperative oedema of mucosa of nose
 Septal abscess
 Septal perforation
 Septal hematoma*
85. A 50 year old furniture maker presents with proptosis, swelling over frontoethmoidal region
and repeated left sided episodes of epistaxis for 6 months. What is diagnosis? (RIHS)
 Maxillary sinus carcinoma *
 Mucormycosis
 Bleeding polypus
 Adenoid cystic carcinoma of ethmoid
 Maxillary sinusitis
86. Most important clinical examination to diagnose antrochonal polyp is (RIHS/FMDC)
 Anterior rhinoscopy
 Posterior Rhinoscopy*
 Nasal petancy test
 Probe test
 Spatula test
87. All are true for nasal packing except (RIHS)
 Made up of ribbon gauze
 BIPP is used
 Polyfax is used
 Toxic shock syndrome may occur
 Septal haematoma may develop*
88. Regarding Ethmoidal Nasal Polyps which one is False? (RIHS/FMDC)
 Multiple in number
 Recurrence rate is very high
 Diagnosed on posterior rhinoscopy*
 Allergic in nature
 Best treatment option is FESS
89. A 55 years old patient is having recurrent boil nose. What is one best type of management in
this case? (RIHS/FMDC)
 Take pus for culture and sensitivity
 Treat with broad spectrum oral antibiotics
 Investigate for underlying diabetes mellitus*
 Ask for complete blood picture
 I/V antibiotics and analgesics
90. Which one of the complications contraindicates operation of SMR under the age of 17 years?
(IMDC/RIHS)
 Difficulty in operation
 More chance of Septal perforation
 Retarded growth of nose*
 Saddle deformity
 Septal hematoma
91. Little’s area is supplied by all except. (IMDC/YMDC)
 Spheno palatine artery
 Infra orbital artery *
 Anterior Ethmoidal artery
 Septal branch of superior labial artery
 Greater palatine artery
92. A 20 year’ old male presented with seasonal itching in nose, sneezing & Rhinorhea. On
examination nasal mucosa is edematous and pale blue in color. What is diagnosis? (IMDC)
 Maxillary sinus carcinoma key e
 Bleeding polypus
 Vasomotor rhinitis
 Maxillary sinusitis
 Allergic rhinitis*
93. A 55 year’ male had recurrent boil in nose. What is one best type of management in this
case? (IMDC)
 Take pus for culture and sensitivity
 Treat with broad-spectrum oral antibiotics
 Investigate for underlying diabetes mellitus*
 Ask for complete blood picture
 I/V antibiotics and analgesics
94. A 6 year’ child is brought by mother with complain of nasal obstruction and disturbed Sleep.
The child was dull looking with open mouth. He is had irregular dental arches. What is your
diagnosis? (IMDC)
 DNS
 Antrochoanal polyp
 Acute maxillary sinusitis
 Recurrent adenoiditis*
 Allergic rhinitis
95. Which one of the complications contraindicates operation of SMR under the age of 17 years?
(FMDC)
 Difficulty in operation
 More chance of Septal perforation
 Retarded growth of nose *
 Saddle deformity
 Septal hematoma
96. A newborn just after delivery has become cyanosed and pale. Examination reveals presence
of mucoid discharge in both nasal cavities with no air bubbles in the nasal discharge.
Immediate and life saving measure is (IMDC)
 Endotracheal intubation
 Mini tracheostomy
 Tracheostomy
 Oral air way insertion*
 Laryngeal mask
97. Constituents of nasal septum include all except (IMDC)
 Vomer
 Perpendicular plate of palatine*
 Quadrangular cartilage
 Maxillary crest
 Membranous septum
98. Best time to reduce nasal fractures (IMDC)
 Immediate after trauma. Before swelling appears*
 2-3 days after trauma
 After 2 weeks
 After 5-7 days when swelling subsides
 After 6 months
99. A 20 year’ male suffering from nasal obstruction, Rhinorrhea, post nasal drip, fever facial
pain across the checks below right orbit & right upper jaw for the last 04 days. What is one
most probable diagnosis? (IMDC)
 Acute rhinitis
 Acute pharyngitis
 Acute maxillary sinusitis on right side*
 Frontal sinusitis
 Acute front ethmoiditis
100. An 18 year’s male was hit on his nose at night, developed bilateral nasal obstruction in the
morning time. The only and significant finding was bilateral dark red, convex soft swelling
occupying anterior part of nasal septum. Which one is the best line of treatment?
(IMDC/RIHS)
 Analgesic, oral and topical decongestant drugs
 Aspetic aspiration, antibiotics, analgesic and topical decongestants
 Aspetic aspiration, nasal packing, antibiotic, analgesic, culture and sensitivity of pus
 Incision and drainage, Nasal packing and antibiotics *
 Urgent Septoplasty, Nasal packing and antibiotics
101. Investigation of choice for Nasopharyngeal Angiofibroma is (IMDC)
 MRA*
 CT scan
 Helical CT
 Angiography
 Postnasal space x ray
102. A 30 year’ old women has developed pain over left maxilla five days after root filling she
has temperature of 37.80c. What is diagnosis?
 Left Maxillary sinusitis*
 Tumour maxillary sinus
 Dentigerous cyst
 Oroantral fistula
 Mucormycosis
103. A 45-year-old man complains of facial pain, anesthesia of cheek and recurrent nose bleeds
over a period of 4 weeks an x-ray of her facial bones shows pacification of the right
maxillary sinus.
 What is diagnosis?
 Cancer maxillary sinus*
 Maxillary sinus cyst
 Maxillary sinusitis
 Antrochoanal polyp
 Allergic rhinitis
104. Most common congenital anomaly of nose is:
 Dermoid cyst
 Meningocele
 Choanal atresia*
 Bifid nose
 Glioma
105. A newborn baby was brought with bilateral nasal obstruction A soft rubber catheter was
tried to pass through the nose but failed What is the most likely diagnosis
 Bilateral nasal polyps
 Choanal atresia*
 Hypertrophic turbinates
 Chronic Rhinitis
 Adenoids
106. What is the structure whose persistence results in Choanal atresia (YMDC)
 Frontonasal process
 Median nasal process
 Bucconasal membrane*
 Persistant lymphoid tissue
 Primitive nasal membrane
107. A 3 Yr child was brought with H/o unilateral foul-smelling blood stained nasal discharge
What is the most likely diagnosis?
 Congenital choanal atresia
 Fracture of the nasal bone
 Foreign body nose*
 Nasopharyngeal angiofibroma
 Antrochoanal nasal polyp
108. A 25 Yr male who met a road traffic accident 2 weeks back C/O clear watery nasal discharge
from his Rt nose which increases on straining. What is most likely cause of this nasal
discharge (YMDC)
 Allergic Rhinitis
 Non allergic eosinophilic rhinitis
 CSF Rhinorrhea*
 Discharge from maxillary sinus
 Acute viral rhinitis
109. Confirmatory test to show the presence of CSF is:
 Presence of high glucose level
 High protein level
 Presence of lymphocytes
 β-2 Transferrin test*
 Handkerchief test
110. After a road traffic accident, a 22 Yr male presented with fracture of nasal bone without
any obvious external nasal deformity. How will you treat this patient?
 No treatment*
 Reduction & splintage under GA
 Nasal packing
 Treatment after CT scan
 Plaster of Paris application
111. A 25 Yr male patient was diagnosed as a case of DNS (deviated nasal septum) What is the
most common cause of this condition.
 Nose picking
 Trauma nose*
 Thumb sucking
 Unilateral nasal mass
 Familial
112. On clinical examination of a 35 Yr male patient there was marked C shaped DNS LT Which
turbinate will show compensatory hypertrophy in this patient (RIHS)
 Inferior turbinate on Lt side
 Inferior turbinate on Rt side*
 Middle turbinate on Rt side
 Middle turbinate on Lt side
 Superior turbinate on Rt side
113. A 25 Yr male patient C/O fever, pain & swelling of the nose. On examination there was a
smooth bulge on the nasal septum which was very tender to touch What is the most likely
cause of this condition
 Septal adhesions
 DNS
 Septal abscess*
 Septal spur
 Septal growth
114. A 30 Yr male patient sustained trauma nose 2 days back He C/O bilateral nasal blockage &
broadening of the nose. O/ E there was smooth deep red bulge on both sides of anterior
nasal septum Most likely diagnosis in this patient is
 Septal adhesions
 Septal hematoma*
 DNS
 Nasal polyps
 Septal spur
115. A 30 Yr male patient was planned for SMR operation Which incision is the most appropriate
one for this operation:
 Weber Fergoson’s incision
 Hemitransfixation incision
 Killian’s incision*
 Frere’s incision
 Bitemporal incision
116. SMR operation was done in 20 Yr male patient After how much time nasal packs are to be
removed
 4—6 hours
 24—48 hours*
 4—6 days
 1 week
 10 days
117. Nasal packs were removed in a 25 Yr lady after 24 hours of SMR operation. She came back
after 3 days with H/O bilateral nasal obstruction & broadened nose O/E there was rounded
red swelling on both sides of the nasal septum What is the most likely diagnosis
 Nasal polyp
 Hypertrophic turbinates
 Neoplasm nasal septum
 Septal Hematoma*
 Recurrence of DNS
118. A female patient was undergoing septoplasty During instrumentation she developed CSF
rhinorrhoea What is the most common site of CSF leak in this patient
 Cribriform plate of Ethmoid*
 Floor of frontal sinus
 Roof of Sphenoid sinus
 Lamina papyracea
 Roof of maxillary sinus
119. A 23 Yr male came with nasal obstruction, pain nose & fever O/E he had bilateral septal
swelling which was very tender to touch What is the most likely diagnosis
 DNS
 Nasal polyp
 Acute rhinosinusitis
 Neoplasm
 Septal abscess*
120. A 23 Yr male came with nasal obstruction, pain nose & fever O/E he had bilateral septal
swelling which was very tender to touch What complication can occur in this patient
 CSF Rhinorrhoea
 Septal adhesions
 Nasal hump
 Saddle nasal deformity*
 Septal spur formation
121. A 20 Yr male patient presented in OPD with H/o severe pain tip of nose of 1 day duration.
O/E tip of the nose was swollen & red with marked tenderness Most likely diagnosis is
 Acute Rhinitis
 Boil nose*
 Hemangioma
 Congenital deformity
 Malignant Neoplasm
122. A 20 Yr male patient presented in OPD with H/o severe pain tip of nose of 1 day duration.
O/E tip of the nose was swollen & red with marked tenderness Which organism is most
commonly responsible
 H influenzae
 Streptococci
 Staph aureus*
 Pneumococci
 Pseudomonas
123. Commonest cause of septal perforation is
 Trauma*
 Syphilis
 Lupus
 Fungal infection
 Blood dyscrasias
124. What are the two most important predisposing factors for recurrent nasal boils
 Nasal allergy & bronchial asthma
 Hypertension & nasal allergy
 Diabetes mellitus & scratching the nose*
 Nasal polyps & bronchial asthma
 Swimming in dirty water & renal failure
125. A 60 Yr diabetic male was diagnosed as a case of boil nose. Which of the following
intracranial venous sinus thrombosis is most likely?
 Cavernous sinus*
 Lateral sinus
 Superior sagital sinus
 Inferior sagital sinus
 Sigmoid sinus
126. A 23 Yr male came with H/O nasal obstruction, fever, thick greenish Rt nasal discharge &
pain Rt cheek of 6 days duration O/E there was mucopurulent discharge Rt nostril &
tenderness over Rt cheek Most likely diagnosis in this patient is:
 Allergic Rhinitis
 Acute rhinitis
 Acute Rt maxillary sinusitis*
 Acute Rt ethmoid sinusitis
 Periodontal infection
127. A 23 Yr male came with H/O nasal obstruction, fever, thick greenish Rt nasal discharge &
pain Rt cheek of 6 days duration O/E there was mucopurulent discharge Rt nostril & tender
ness over Rt cheek. X-Ray PNS was ordered in this patient Which view will be most helpful
for diagnosis
 Lateral view
 Occipitomental view*
 Lateral oblique view
 Anteroposterior view
 Submentovertical view
128. X-Ray PNS Occipitomental view (Water’s view) was advised in a 24 Yr male Which of the
sinus is difficult to visualize in this view
 Maxillary sinus
 Frontal sinus
 Ethmoid sinus*
 Sphenoid sinus
 All of the above
129. A 16 Yr female patient was diagnosed a case of boil nose which part of the nose is most
common site of origin of boil
 Nasal cavity
 Inferior turbinate
 Vestibule of nose*
 Middle meatus
 Floor of nose
130. A young female patient came with H/O sneezing, watery nasal discharge , fever & body
ache of 2 days duration Which of the following organisms is most likely involved (YMDC)
 Pseudomonas
 Adenovirus*
 Immune deficiency virus
 E Coli
 Aspergillus
131. Proof puncture(Antral wash out) was planned in a 30 Yr male patient What structure is
drained in this operation (YMDC)
 Maxillary sinus*
 Ethmoid sinuses
 Frontal sinus
 Sphenoid sinus
 Frontal & sphenoid sinus
132. Proof puncture(Antral wash out) was planned in a 30 Yr male patient What common
complications do you expect in this operation (YMDC)
 Damage to teeth
 Orbital & cheek injury*
 Meningitis
 Injury to infraorbital nerve
 CSF rhinorrhoea
133. Trephenation was planned in a 18 Yr boy to drain pus Which of the following structure is
drained
 Maxillary sinus
 Nasal cavity
 Frontal sinus*
 Sphenoid sinus
 Ethmoid sinuses
134. A 20 Yr girl came with H/O nasal obstruction & recurrent epistaxis for about 2 Yrs. O/E both
nasal cavities were wide & filled with foul smelling dirty crusts What is the most likely
diagnosis
 Chronic sinusitis
 Atrophic rhinitis*
 Foreign body nose
 Rhinolith
 Malignant tumour
135. A 20 Yr girl came with H/O nasal obstruction & recurrent epistaxis for about 2 Yrs. O/E both
nasal cavities were wide & filled with foulsmelling dirty crusts She was diagnosed a case of
Atrophic Rhinitis Which of the following solutions will be used for instillation in the nose
 25% Glucose in glycerine*
 25% sodium bicarbonate in glycerine
 25% Silver nitrate in glycerine
 25% Sodium chloride in glycerine
 25% Potassium iodide in glycerine
136. A young female patient was diagnosed as a case of Atrophic Rhinitis. She received
conservative treatment for 2 Yrs but no relief Which surgical procedure will be tried for
treatment
 Caldwell Luc operation
 External Ethmoidectomy
 Lateral Rhinotomy
 Young’s procedure*
 FESS
137. “FESS” is the abbreviation for
 Frontoethmoid sinus surgery
 Fast endoscopic sinus surgery
 Functional endoscopic sinus surgery*
 Field enhanced sinus surgery
 Fast endoscopic sinus surgery
138. Which of the following Telescopes are commonly used in FESS
 0, 20 & 60 degrees
 0, 30 & 70 degrees*
 10, 20 & 30 degrees
 0, 30 & 90 degrees
 0, 70 & 50 degrees
139. A 55 Yr male presented with a polypoidal mass in Lt nostril & was diagnosed a case of
Antrochoanal polyp. Which of the following should be considered as differential diagnosis?
(YMDC)
 Allergic fungal sinusitis
 Juvenile nasopharyngeal angiofibroma
 Dermoid cyst
 Inverted papilloma*
 Squamous papilloma
OR
140. A 45-year-old male patient presented with a polypoidal mass in the right nasal cavity and
diagnosed clinically as a case of antrochoanal polyp. Which of the following should be
considered in the differential diagnosis in this case? (YMDC)
 allergic fungal sinusitis
 dermoid cyst
 inverted papilloma*
 squamous papilloma
 osteoma of maxillary sinus
141. A 30 Yr male was diagnosed as a case of bilateral ethmoidal nasal polyps He also gives H,O
bronchial asthma Which of the following medicines he should avoid
 Ephedrine
 Aspirine*
 Quinine
 Erythromycine
 Xylocaine
142. The importance of Fossa of Rossenmuller is because it is a common site to give origin to
 Carcinoma nasopharynx*
 Antrochoanal polyp
 Nasopharyngeal angiofibroma
 Branchial cyst
 Adenoids
143. Antrochoanal polyp arises from
 Maxillary sinus*
 Ethmoid sinuses
 Spenoid sinus
 Frontal sinus
 Lateral wall of nose
144. Ethmoidal polyposis is usually associated with
 Tuberculosis
 Bronchial asthma*
 Diabetes mellitis
 Pneumonia
 Malignancy
145. Surgical treatment of choice for chronic maxillary sinusitis is
 Caldwel Lucs operation
 FESS*
 Antral wash out
 Intranasal antrostomy
 Intranasal ethmoidectomy
146. Malignant tumour of nose & paranasal sinuses most commonly associated with hard wood
industry is
 Adenocarcinoma *
 Adenoid cystic carcinoma
 Squamous cell carcinoma
 Malignant melanoma
 Sarcoma
147. A 25 year male came in OPD with compliant of right sided nasal obstruction for last 3 years.
Posterior rhinoscopy showing a smooth grayish mass covered with nasal discharge which is
hanging down from nasopharynx. Anterior rhinoscopy is unremarkable. The most probable
diagnosis is (RIHS)
 Nasopharyngeal angiofibroma
 Carcinoma nasopharynx
 Adenoids
 Antrochoanal polyp*
 Ethmoidal polyp
148. A 17 year female came in OPD with compliant of right sided nasal obstruction for last 3
years. Posterior rhinoscopy showing a smooth grayish mass covered with nasal discharge
which is hanging down from nasopharynx. Anterior rhinoscopy is unremarkable. The most
probable diagnosis is (RIHS)
 Nasopharyngeal angiofibroma
 Carcinoma nasopharynx
 Adenoids
 Antrochoanal polyp*
 Ethmoidal polyp
149. A 55 Yr male presented with a polypoidal mass in Lt nostril & was diagnosed a case of
Antrochoanal polyp which has recurred after intranasal polypectomy. What is the most
appropriate treatment for this patient
 Lateral rhinotomy
 Caldwell Luc’s operation*
 Intranasal ethmoidectomy
 Steroid nasal drops
 Systemic steroids
150. Which of the following fungal infection presents with bilateral multiple nasal polyps
 Acute fulminant fungal sinusitis
 Allergic fungal sinusitis*
 Fungal ball(Mycetoma)
 Invasive fungal sinusitis
 Granulomatous fungal sinusitis
151. A 30 Yr female patient presented with extensive bilateral ethmoidal polyps She also C/O
change in her voice What will be the character of her voice
 Hypernasality
 Hyponasality*
 Hoarseness of voice
 “Hot potato voice”
 Stuttering
152. A 30 Yr female patient presented with extensive bilateral ethmoidal polyps & an increase in
the intercanthal distance of the eyes. Which investigation is the most appropriate one for
this patient
 X-Ray PNS Water’s view
 Biopsy for histopathology
 CT scan paranasal sinuses*
 Nasal smear for culture
 Ultrasound orbits
153. Surgery was done in a patient who had suspected fungal sinusitis Which of the following
solutions will be used to send the specimen for fungal culture
 10% formaline
 5% glucose
 Normal saline*
 Sodium bicarbonate
 Silver nitrate solution
154. On anterior rhinoscopy of a 6 months old baby a polypoidal mass was seen in Lt nostril
Which of the following conditions should be excluded before labeling it as a nasal polyp
 Aplasia of the sinuses
 Choanal atresia
 Cavernous sinus thrombosis
 Meningocele
 Hypertrophic turbinate*
155. 13 years old male presents with severe recurrent spontaneous nose bleed. The most
probable diagnosis is:
 Bleeding polyp of the nose.
 Nasopharyngeal angiofibroma*
 Foreign body in the nose
 Bleeding from Little’s area.
 Fracture nasal bone
156. What is the other name for Inverted papilloma
 Squamous papilloma
 Transitional papilloma*
 Antrochoanal polyp
 Bleeding polyp
 Ethmoidal polyp
157. In which of the following groups Inverted papilloma is most commonly seen
 Young females
 Adolescent males
 Middle to old age males*
 Male child
 Female child
158. Caldwel Luc’s operation was planned in a young patient Which incision will be used for this
operation
 Sublabial horizontal incision*
 Gingival incision
 Palatal incision
 Weber Furguson incision
 Incision between bridge of nose & medial canthus
159. One of the contraindications to Caldwel Luc’s operation
 Foreign body in the Antrum
 Antral cyst
 Unerupted teeth in children*
 Mucocele
 Fungal ball
160. Following is Not the complication of External ethmoidectomy
 Lacrimal sac injury
 CSF leak
 7th nerve paralysis*
 Supraorbital nerve injury
 Obvious scar
161. Regarding epistaxis choose most appropriate answer (YMDC)
 Most common site is lateral wall of nose
 Commonly occurs in cold climate
 Intractable cases are managed by ligation of internal carotid artery
 It is most commonly idiopathic*
 Bleeding disorders are the most common cause
162. The most common site for epistaxis is
 Anterior end of inferior turbinate
 Anterior end of the middle turbinate
 Upper part & roof of the nasal cavity
 Antroinferior part of the nasal septum*
 Posterosuperior part of the nasal septum
163. A 20 Yrs boy C/O recurrent epistaxis of mild to moderate nature What will you advise to do
during an acute episode at home
 Lie supine with head up position
 Lie supine with head down position
 Sit upright with backward inclination
 Rinse the head with cold water
 Sit upright, forward inclination & pinch the nose*
164. What material is most appropriate one for anterior nasal packing
 Spongston
 Ribbon guaze*
 Cotton
 Cotton in glove fingerstall
 Rubber sponge
165. Best treatment of mild epistaxis from Little’s area is
 Anterior nasal packing
 Cautery of the area*
 Postnasal packing
 Blood transfusion
 Arterial ligation
166. Anterior nasal packing was done ia 25 Yr girl who had severe epistaxis What is the most
appropriate time after which packs should be removed
 3 to 6 hours
 6 to 12 hours
 24 to 48 hours*
 5 to 7 days
 After 1 week
167. Kiesellbach’s plexus is located at
 Posterior end of inferior turbinate
 Near sphenopalatine foramen
 Anteroinferior part of the nasal septum*
 Posterosuperior part of the nasal septum
 Vestibule of nose
168. Most commonly LASER used in ENT is. (RIHS)
 KTP-532 LASER
 ND YAG LASER
 Helium-neon LASER
 CO2 LASER*
 Argon LASER
169. Temperature of cryoprobe is usually
 30 C
 – 30 C*
 20 C
 0C
 40 C
170. Other name of “Lupus vulgaris” of nose is
 Aspergillosis
 Rhinosporidiosis
 Low grade tuberculosis*
 Wegner’s granuloma
 Leprosy
171. A young female patient who C/O sneezing, watery rhinorrhea & itching eyes especially
during March & April. What typical finding on nasal mucosa do you expect in this patient
 Blue & thin
 Red & congested
 Pale blue & swollen*
 Atrophic & crusty
 Red & thick
172. A young patint was diagnosed as a case of Nasal Allergy due to house dust mite What type
of hypersensitivity reaction occurs in this patient
 IgE mediated type 1 hypersensitivity reaction*
 IgM mediated hypersensitivity reaction
 IgG mediated type 111 reaction
 Type IV hypersensitivity reaction
 IgA mediated hypersensitivity reaction
173. An 8 Yr boy was exposed to house dust mite & produced allergen specific IgE antibodies.
On which of the cells these antibodies will be fixed to produce nasal allergy on future
exposures Pharynx, Larynx, Oral cavity, Head & Neck(13.3,4,5&7)
 Red blood cells
 Lymphocytes
 Mast cells*
 Platelets
 Neutrophils
174. Bulla ethmoidalis is present in:
 Inferior nasal meatus
 Middle nasal meatus*
 Superior nasal meatus
 Nasopharynx
175. F.B bronchus is treated by:
 Eosophagoscopy
 Direct laryngoscopy
 Tonsillectomy
 Bronchoscopy*
176. Beck’s triad occurs due to:
 Quinsy
 Retropharyngeal abscess
 Parapharyngeal abscess *
 Vincent angina
177. Reactionary bleeding after adenoidectomy is treated by:
 Removal of the remnants and posterior pack
 Antibiotics
 Anterior nasal pack
 Ligation of the bleeding vessel*
178. Secondary post – adenoidectomy bleeding is treated by:
 Antibiotics and posterior nasal pack *
 Anterior nasal pack
 External carotid artery ligation
 Cautery of the little’s area
179. A patient with epistaxis showing a bleeding point in little’s area is best managed by:
 Cautery*
 Anterior nasal pack
 Posterior nasal pack
 Coagulants
180. Primary atrophic rhinitis is characterized by all of the following except:
 Recurrent epistaxis
 Anosmia
 Bad odor smelled by the neighbors
 Affects male only*
181. Functional endoscopic sinus surgery is the operation of choice in all of the following except:
 Chronic sinusitis
 Mucocele of the paranasal sinuses
 Nasal polyposis
 Twisted nose*
182. Unilateral watery nasal discharge is most likely to suggest a diagnosis of:
 Acute viral rhinitis
 CSF rhinorrhea*
 Allergic rhinitis
 Rhinoscleroma
183. Unilateral offensive nasal discharge may suspect:
 Acute rhinitis
 Nasal polyposis
 Epistaxis
 Nose F.B*
184. One of the predisposing factors for post cricoid carcinoma is:
 Plummer Vinson syndrome*
 Vitamin E deficiency
 Vitamin D deficiency
 Chronic tonsillitis
185. Trotter’s triad is a clinical diagnosis of:
 Nasopharyngeal angiofibroma
 Nasopharyngeal cyst
 Petrositis
 Nasopharyngeal carcinoma*
186. The following paranasal sinuses open in the middle meatus except:
 Sphenoid sinus *
 Maxillary sinus
 Frontal sinus
 Anterior ethmoid air cells
187. Posterior nasal packing is indicated in all of the following situations except:
 Severe posterior epistaxis
 Antro-choanal polyp*
 Reactionary post-adenoidectomy bleeding
 After removal of nasopharyngeal angiofibroma
188. Facial nerve gives secretomotor supply to:
 Parotid gland
 Submandibular salivary gland*
 Pituitary gland
 Thyroid gland
189. Anterior septal perforation may be caused by:
 Polypectomy
 Adenoidectomy
 Rhinoscleroma
 Bilateral cautery for epistaxis *
190. All the following are possible complications of acute sinusitis except:
 Temporal lobe brain abscess *
 Cavernous sinus thrombosis
 Orbital cellulitis
 Osteomylaitis
191. An infant with bilateral choanal atresia will present by:
 Conductive deafness
 Epistaxis
 Postnasal discharge
 Respiratory distress*
192. In epistaxis, bleeding point in the little’s area is best controlled by:
 Cautery*
 Anterior nasal pack
 Posterior nasal pack
 Coagulants
193. Allergic rhinitis is characterized by all the following except:
 Attacks of sneezing
 Watery or mucoid rhinorrhea
 Antrochoanal polyp*
 Pale or bluish nasal mucosa
194. Nasopharyngeal angiofibroma is characterized by all the following except:
 Lower motor facial nerve paralysis *
 Recurrent severe epistaxis
 Occurs in adolescent boys
 May cause frog face deformity
195. The value of functional endoscopic sinus surgery is to:
 Restore sinonasal functions
 Preserve sinus drainage through natural ostia
 Avoid external scars
 All of the above*
196. Septal heamtoma is treated by:
 Antibiotics
 Antihistaminc
 Drainage and pack*
 Septal perforation
197. Nasal septal perforation is caused by all the following except:
 Bilateral cautery for epistaxis
 Allergic rhinitis *
 Syphilis
 T.B
198. Unilateral nasal obstruction can be the result of the following except:
 Unilateral choanal atresia
 Antro-choanal polyp
 Marked septal deviation
 Septal perforation*
199. Unilateral offensive nasal discharge may be due to:
 Allergic polyp
 Bleeding polypus
 Neglected F.B nose*
 Nasal osteoma
200. Reservoir sign is positive in:
 Acute petrositis
 Chronic labyrinthitis
 Acute mastoiditis*
 Cholesteatoma
201. Rhinoscleroma is treated by the following except:
 Streptomycin
 Rifampicin
 Radiotherapy *
 Puncture and lavage
202. The most common cause of reactionary post- adenoidectomy bleeding is:
 Adenoid remnant
 Adenoid enlargement
 Hypertension *
 Fever
203. Recurrent severe epistaxis in adolescent boy is most likely to be due to:
 Nasopharyngeal angiofibroma*
 Antro-choanal polyp
 Nasopharyngeal carcinoma
 Chronic sinusitis
204. The most common cause of Bell’s palsy is:
 Trauma to the temporal bone
 Cholesteatoma
 Glomus tympanicus
 Idiopathic*
205. A shelf-like projection in the nasal septum is called: (IMDC)
 Crooked nose
 Thickened nasal septum
 Anterior dislocation
 Spur*
 Saddle deformity
206. Which type of deviated nasal septum (DNS) causes more nasal obstruction? (IMDC)
 High DNS*
 DNS along the floor
 Caudal septal dislocation
 Septal spur
 Deflected vomer
207. Cottle test is done to evaluate nasal obstruction at which of the following areas? (IMDC)
 Anterior nares
 Nasal valve*
 Inferior meatus
 Posterior nares
 Roof of the nasal cavity
208. A seven years old child is brought to with complaints of bilateral nasal obstruction after
sustaining trauma to the nose while falling from a swing. On examination there was smooth
fluctuant bulge on either side of the septum. What is your diagnosis? (IMDC)
 Septal abscess
 Septal hematoma*
 DNS
 Nasal polyp
 Hypertrophied inferior terbinates
209. A patient who underwent SMR operation 3 months back is complaining of whistling sounds
during respiration, crusting and occasional epistaxis. What is the most probable diagnosis?
(IMDC)
 Septal Hematoma
 Adhesions
 Septal prolapse
 Septal perforation*
 Foreign body
210. Once diagnosed, a septal abscess should be drained: (IMDC)
 Immediately*
 Within 24 hours
 After a 48 hours antibiotics trial
 On the next routine operation list
 Over a period of one week
211. A 13 years old girl complains of left side nasal obstruction more on expiration than
inspiration since 6 months. On examination no mass can be seen on anterior rhinoscopy but
posterior rhinoscopy reveals a smooth grayish mass covered in nasal discharge which is
insensitive to touch and mobile. What is your diagnosis? (IMDC)
 Foreign body
 Rhinolith
 Antrochoanal polyp*
 Angiofibroma
 Rhabdomyosarcoma
212. What is the proposed etiology of antrochoanal polyp? (IMDC)
 Nose and sinus infection*
 Nasal allergies
 Benign neoplasia
 Vascular hypertrophy
 Metastatic growth
213. Bilateral nasal polyps are most commonly seen in: (IMDC)
 Nasal allergies*
 Acute infection of nasal mucosa
 Chronic bacterial sinusitis
 Atrophic rhinitis
 Rhinitis medicamentosa
214. Nasal polypi commonly arise from which site of the nasal cavity? (IMDC)
 Middle turbinate
 Middle meatus*
 Inferior meatus
 Septum
 Floor
215. What is the treatment of a recurrent antrochoanal polyp? (IMDC)
 Inferior meatus antrostomy
 Medial maxillectomy
 Caldwell Luc’s operation*
 Antral lavage
 Avulsion with a snare
216. The etiology of recurrent respiratory papillomatosis is: (IMDC)
 Fungal infection
 Gram Positive bacteria
 Mycobacteria tuberculosis
 Human papilloma virus*
 Autoimmune laryngitis
217. The normal pH of nasal secretion is: (IMDC)
 Acidic*
 Alkaline
 Neutral
 Undetermined
 Changes with altitude
218. The largest and most prominent cell of ethmoid sinuses is called? (IMDC)
 Agar nasi
 Heller cell
 Concha bullosa
 Bulla ethmoidalis*
 Lateral sinus of Grunwald
219. The nasolacrimal duct opens at: (IMDC)
 Superior meatus
 Middle meatus
 Maxillary sinus
 Inferior meatus*
 Sphenoethmoidal recess
220. The narrowest part of the internal nose is called: (IMDC)
 Anterior choana
 Pyriform aperture
 Vestibule of the nose
 Posterior nares
 Nasal valve area*
221. Which of the following arteries of nose arises from internal carotid system? (IMDC)
 Greater palatine
 Smaller palatine
 Sphenopalatine
 Anterior ethmoidal*
 Superior labial
222. Sympathetic and parasympathetic fibers reach the nasal mucosa through: (IMDC)
 Vidian nerve*
 Jacobson’s nerve
 Arnold nerve
 Mandibular nerve
 Cauda tympani nerve
223. The slit-like gap between the posterior border of uncinate process and bulla ethmoidalis is
called: (IMDC)
 Suprabullar recess
 Hiatus semilunaris*
 Infundibulum
 Posterior Fontanel
 Retrobullar recess
224. Hair follicles and vibrissae are found in which part of the internal nose? (IMDC)
 Nasal vestibule*
 Inferior turbinate
 Septal mucoperichondrium
 Roof of the nasal cavity
 Mucosa floor of the nose
225. Which of the following is confirmatory test for Sjogren’s Syndrome? (IMDC)
 Schirmir’s test
 Rheumatoid factor
 Erythrocytes sedimentation rate
 Sublabial biopsy
 Antinuclear antibodies*
226. What is the commonest site of impaction of a tracheobronchial foreign body? (IMDC)
 Carina
 Cervical trachea
 Thoracic brachea
 Right main bronchus*
 Left main bronchus
227. What is the earliest sign on an X-ray of a radiolucent foreign body partially obstructing the
lumen of a bronchus? (IMDC)
 Hyperinflation of the same side lung*
 Hyperinflation of the opposite side of lung
 Edema at the site of impaction
 Collapse of the opposite side lung
 Pneumothorax
228. Which of the following tests is the most sensitive and specific for CFS rhinorrhea? (IMDC)
 High sugar content
 High protein content
 Beta Transferrin*
 Higher cellular count
 High concentration of Sodium and chloride ions
229. The most cardinal sign of CSF rhinorrhea is: (IMDC)
 A gush of fluid from nose on bending forward or straining*
 A constant discharge of clear fluid from the nose
 Thread of fluids that can be easily sniffed back
 A fluid which stiffens a piece of cloth after drying
 A thick turbid fluid from the nose on lying supine
230. Low grade tuberculous infection of the nasal vestibule is called: (IMDC)
 Rhinoscleroma
 Lupus vulgaris*
 Lichens planus
 Rhinitis sicca
 Ozaena
231. A fifty years old man presented with right sided nasal obstruction, purulent discharge and a
hard-pale yellow growth with trivial bleeding on touch. He was previously operated twice
for the condition but it recurred quickly. What is the most likely diagnosis? (IMDC)
 Fungal polyp
 Antrochoanal polyp
 Ethmoidal allergic polyp
 Inverted papilloma*
 Carcinoma
232. Which carcinoma of the nose is associated hardwood dusts and furniture industry workers?
(IMDC)
 Malignant melanoma
 Adenocarcinoma*
 Squamous cell carcinoma
 Nasopharyngeal carcinoma
 Olfactory neuroblastoma
233. Regarding epistaxis, choose the best answer:
 The most common site is lateral wall of nose
 It is more common in cold climate
 Intractable cases can be managed by ligation of internal carotid artery
 Intractable cases can be managed by ligation of both external & internal carotid arteries
 It is mostly idiopathic in nature*
234. A 3 years old child presents with unilateral nasal obstruction and unilateral blood-stained
mucopurulent nasal discharge. The most probable diagnosis Is: (YMDC)
 Antrochoanal polyp
 DNS
 Rhinitis
 Angiofibroma
 Foreign body nose *
235. A 5-year boy came with history of trauma twit 4 days back and bilateral nasal obstruction
for last 2 days. There was no history of bleeding from nose. On examinations patient was
afebrile & both nasal cavities revealed sword rounded fluctuant swellings of the septa. The
meat probable diagnosis is: (YMDC)
 DNS
 Clotted blood in both nasal cavities
 Septal Hematoma *
 Septal abscess
 Nasal polyp
236. SMR operation was done in 20 Yr. male patient After bow much time nasal packs are to be
removed: (YMDC)
 4-6 hours
 24-48 hours *
 4-6 days
 1 week
 10 days
237. A young female patient came with Ito meeting, watery nasal discharge, fever & body ache
of 2 days duration which of the following organisms is most likely involved:
 Pseudomonas
 Adenovirus*
 Immune deficiency virus
 E. coli
 Aspergillus
238. The following sinus is the most commonly infected sinus: (YMDC)
 Maxillary*
 Sphenoidal
 Right Ethmoidal
 Frontal
 Left Ethmoidal
239. A 20 Yr. male patient presented in OPD with H/o severe pain tip of nose of 1 day duration.
0/E tip of the nose was swollen & red with marked tenderness Most likely diagnosis is:
(YMDC)
 Acute Rhinitis
 Boil nose*
 Hemangioma
 Congenital deformity
 Malignant Neoplasm
240. The following type of DNS usually causes headache. (YMDC)
 Spur*
 C shaped deformity
 Thickened septum
 S shaped deformity
 caudal end dislocation
241. A 25 Yr. male who met a road traffic accident 2 weeks back C/O clear watery nasal
discharge from his Rt nose which increases on straining. What is most likely cause of • this
nasal discharge?
 Allergic Rhinitis
 Non allergic eosinophilic rhinitis
 CSF Rhinorrhea*
 Discharge from maxillary sinus
 Acute viral rhinitis
242. Recruitment phenomenon is seen in (YMDC)
 Otosclerosis
 Meniere's disease*
 Acoustic nerve schwannoma
 Otitis media with effusion
 Vestibulitis
243. A 19 years old girl developed saddling of nose after trauma a month ago. Which one is the
best treatment? (YMDC)
 Septoplasty
 SMR
 Augmentation rhinoplasty*
 Reduction rhinoplasty
 Septorhinoplasty
244. Mucormycosis is a fungal infection of nose and paranasal sinuses which my impulse the
brain. It is mostly seen in patients (YMDC)
 Of chronic renal failure
 with uncontrolled hypertension
 with uncontrolled diabetes *
 taking antibiotics
 chronic sinusitis
245. The landmark on lateral surface of temporal bone to locate the mastoid antrum in mastoid
surgery is called (YMDC)
 Trauman’s triangle
 temporal triangle
 MacEwen's triangle*
 Notch of Rivinus
 Inframeatal triangle
246. Bezold abscess is located in (YMDC)
 Submandibular region
 deep to sternocleidomastoid muscle*
 infratemporal region
 Posterior root of zygoma
 Over the mastoid
247. What is the structure whose persistence results in Choanal atresia
 Frontonasal process
 Median nasal process
 Bucco nasal membrane*
 Persistent lymphoid tissue
 Primitive nasal membrane
248. Confirmatory test to show the presence of CSF in nasal discharge is (YMDC)
 Presence of high glucose level
 High protein level
 Presence of lymphocytes
 β-2 Transferrin test*
 Handkerchief test
249. A 22 yr. old male had a road traffic accident. X ray showed fracture of nasal bones without
any obvious external nasal deformity. How will you treat this patient? (YMDC)
 No treatment*
 Reduction & splintage under GA
 Nasal packing
 Treatment after CT scan
 Plaster of Paris application
250. X-Ray PNS Occipitomental view (Water's view) was advised in it 24 Yr. male Which of the
sinus is difficult to visualize in this view? (YMDC)
 Maxillary sinus
 Frontal sinus
 Ethmoid sinus*
 Sphenoid sinus
 All of the above
251. The following type of DNS can be seen with anterior rhinoscopy without speculum? (YMDC)
 Spur
 Caudal end dislocation *
 C shaped deformity
 thickened septum
 S shaped deformity
252. A 23 Yr. male came with H/O nasal obstruction, fever, thick greenish Rt nasal discharge &
pain Rt cheek of 6 days duration O/E there was mucopurulent discharge Rt nostril &
tenderness over Rt cheek. X-Ray PNS was ordered in this patient Which view will be most
helpful for diagnosis (YMDC)
 Lateral view
 Occipitomental view*
 Lateral oblique view
 Anterior posterior view
 Submentovertical view
253. A 60 yr. diabetic male was diagnosed as a case of boil sole. Which of the following
intracranial venous sinus thrombosis is most likely? (YMDC)
 Cavernous sinus*
 Lateral sinus
 Superior sagittal sinus
 Interior sagittal sinus
 Sigmoid sinus
254. Syringing was advised in a 25 Yr male patient which of the following agent will be used for
this purpose (YMDC)
 Distilled water at room temperature
 Normal saline at body temperature*
 Tap water at room temperature
 Alcohol
 methylated spirit
255. A 6 years old child is brought by mother that her child is having nasal obstruction, disturbed
sleep and increases the volume of TV. He is dull & apathic looking with open mouth. He is
also having irregular dental arches. What is your diagnosis? (IMDC/FMDC)
 DNS
 Ethmoidal polypi
 Antrochoanal polypi
 Acute maxillary sinusitis
 Recurrent adenoiditis*
256. In case of injury nose, the best time to reduce fracture nasal bone is (FMDC)
 After three weeks
 After the swelling over the nose subsides*
 First get X-rays nasal bone and then do it
 After the healing of skin laceration
257. A patient is suffering from recurrent episodes of sneezing, Rhinorhea, nasal obstruction,
itching in nose, throat and eyes and occasional attacks of wheezing. His mother and one
sister also suffer from similar problems. What is the one most probable clinical diagnose?
(RIHS/FMDC)
 Deviated nasal septum
 Vasomotor rhinitis
 Allergic rhinitis*
 Chronic rhino sinusitis
258. A 4 years old male child is brought with history of mouth breathing, snoring at night, runny
nose and disturbed sleep at night due to checking of breath. The most common cause of
nasal obstruction at this age is (FMDC)
 Deflected nasal septum
 Antrochonal polyp
 Ethmoidal polypi
 Enlarged adenoids*
259. A 17 years old boy had operation of SMR. On removal of nasal packing on 3rd post-
operative day he complained of bilateral nasal obstruction before operations. What is one
possible cause (FMDC)
 In adequate operation
 Post operative oedema of mucosa of nose
 Septal abscess*
 Septal perforation
260. A 20 years male is having bilateral nasal obstruction/E there is C-shaped deviation of nasal
septum to left side. Right inferior turbinate is markedly enlarged. You performed
Septoplasty with total inferior turbinectomy right. Postoperative recovery is smooth with
patent both nasal cavities. Patient reports after 6 months with the complaints of foul smell
from the nose with blood stained nasal discharge. Examination reveals unduly wide right
nasal cavity with crust. left nasal cavity is normal. The most likely complication is (FMDC)
 Septal abscess
 Septal perforation
 Flapping septum
 Atrophic rhinitis*
 Septal hematoma
261. A 25 years young adult is having nasal obstruction bilateral. He gives history of RTA in early
childhood. Examination reveals S-shaped DNS with external deformity of nose. Which is the
treatment option (RIHS/FMDC)
 Septoplasty
 SMR
 Rhinoplasty
 Septo Rhinoplasty*
 Septo dermoplasty
262. All are true of Nasal Septal Abscess except (FMDC)
 Can lead to life threatening complication
 Usually follows untreated Septal hematoma
 Destroys alar cartilages*
 Can lead to Septal perforation
263. Saddle deformity of nose results from destruction of (FMDC)
 Vomer
 Perpendicular plate of ethmoid
 Septal cartilage*
 None of the above
264. Nasal polyp are associated with all of the following conditions except (FMDC)
 Vasomotor rhinitis
 Allergic rhinitis
 Atrophic rhinitis *
 Aspirin sensitivity
265. Unilateral nasal polyp may be (FMDC)
 Antrochonal polyp*
 Meningocele
 Neopiasia
 All of the above
266. Bilateral nasal obstruction in antrochoanal polyp is as a result of (FMDC)
 Maxillary sinusitis
 Deviated nasal septum
 Large size of polyp *
 Turbinate hypertrophy
267. Following are the features of ethmoidal nasal połyp Except (FMDC)
 Edematous hypertrophic mucosa*
 Non Neoplastic lesions
 Multiplicity & biiaterality
 Responsiveness to corticosteroids
 Usually a young adult
268. Bacteriology of chronic sinusitis is (FMDC)
 Strept. Pneumonia
 Mixed flora*
 influenzae
 Staph. Aureus
269. Rhinophyma is associated with (FMDC)
 Hypertrophy of sebaceous glands*
 Hypertrophy of sweat glands
 Hyperplasia of endothelial cells
 Hyperplasia of epithelial cells
270. Rhinophyma is associated with all of the following except (FMDC)
 Allergic rhinitis
 Palatal paralysis*
 Adenoids
 Nasal polyps
271. Incision given in septoplasty (FMDC)
 Freer's incision*
 Rosen's incision
 Wilde's incision
 Gluck-Sorenson's incision
272. View used to see maxillary sinuses in X-Ray PNS (FMDC)
 Schillers view
 Law's view
 Water’s view*
 Towne's view
273. Instrument used to reduce fractures of nasal septum. (FMDC)
 Asch's forceps*
 Negus forceps
 Walsham's forceps.
 Luc's forceps
274. Roomy nasal cavities are seen in (FMDC)
 Rhinosporidiosis
 Mucor mycosis
 Oznena*
 Irritative rhinitis
275. Four-year child presented in ENT OPD with H/O right sided foul smelling nasal discharge for
5 days, the provisional diagnosis is (FMDC)
 FB Rt Nostril*
 Inverted papilloma Rt
 Killian's polyp Rt
 DNS Rt
276. A 17 years old boy has operation for DNS. On removal of nasal packing on 3rd post-
operative day he complains of bilateral nasal obstruction as before operation. What is one
possible cause? (RIHS)
 Inadequate operation
 Postoperative oedema of mucosa of nose
 Septal abscess
 Septal perforation
 Septal hematoma*
277. A 3 years child is having right sided nasal discharges which is blood stained and foul
smelling. Which one is correct? (RIHS)
 Child is having foreign body right nostril*
 Child is having chronic maxillary sinusitis
 Child is having recurrent adenoiditis
 Child is having nasal allergy
 Child is suffering from unilateral choanal atresia
278. A patient presents to OPD with septal hematoma. What will be appropriate treatment
option (YMDC)
 One should wait for one week and trial of antibiotic should be given
 Aspiration
 Incision and drainage
 incision & drainage followed by antibiotic*
 Oral and topical steroids
279. Which one of the following is not a cause of nasal polyposis? (YMDC)
 Allergic Rhinosinusitis
 h. Fungal rhinosinusitis
 Aspirin intolerance
 MeCune Albright Syndrome*
 Kartagener syndrome
280. Regarding Maggots in nose which statement is correct (YMDC)
 Are larsvae of an insect
 Should be removed by forceps
 Are larval stage of housefly and are treated by suction
 Are larval stage of housefly and treated by instillation of chloroform and turpentine oil and
then suction done*
281. Most consistent feature of ethmoidal nasal polyps is: (YMDC)
 They are always multiple and bilateral*
 Recurrence rate is 80 to 90 %o alters endoscopic sinus Surgery
 Antihistamines can reduce its size
 Always allergic in nature
 Endoscopies removal is the only option of treatment
282. In acute sinusitis which is incorrect: (YMDC)
 Most common is maxillary sinusitis
 Most common organismic are gram +ve bacteria and viruses
 On anterior rhinoscopy pus is seen below middle turbinate
 Surgical drainage is 1st line of treatment*
 Headache is common complaint
283. Boy of 14 years presented with recurrent episodes of severe epistaxis, mass in nasopharynx
and in right nasal cavity, probable diagnosis is: (YMDC)
 Cancer nasopharynx.
 Antrochoanal polyp
 Bleeding polypus of nose.
 Angiofibroma*
 DNS
284. About Septal perforation which statement is incorrect (YMDC)
 Usually Patient remains unaware
 Can present with crusting and epistaxis
 May be complication of SMR
 Carcinoma nose cannot present with perforation*
 TB, Wegener's granulomatosis are included in causes
285. Regarding allergic rhinosinusitis which statement is incorrect: (YMDC)
 Type II hypersensitivity reaction*
 Topical steroid drops are very effective
 Patient may develop asthma later on
 itching, sneezing and rhinorrhea are the common symptoms
 Immunotherapy is a treatment option
286. In a 2 years old previously healthy child with a history of sudden onset of wheezing right
chest. Recurrent bouts of cough after an unattended session in the playground the most
likely diagnosis would be (YMDC)
 Bronchial asthma*
 Acute epiglottilis
 F.B inhalation
 Laryngotracheobronchitis
287. 3 Years old boy suddenly developed cough, stridor, difficulty in breathing & Cyanosis while
playing (RIHS)
 Acute epiglottitis
 Tumor Larynx
 Laryngeal Web
 Foreign body airway*
 Laryngomalcia
288. A patient comes to Emergency Room with severe respiratory distress Immediately the best
& easiest way to restore airway is
 Airway placement
 Tracheostomy
 Endotracheal intubation
 Cricothyrotomy (Minitracheostomy)*
 Elective tracheostomy
289. Which of the following symptoms is Not present in patient with Atrophic rhinitis? (RIHS)
 Anosmia
 hyposmia
 Nasal obstruction
 Nasal secretions
 Hypertrophied turbinate*
290. A mother accompanying her 3 years old boy reported to ENT clinic. She was a bit upset and
complained that for the last one week she noticed an offensive foul smell from his nose. On
enquiring she added H/O unilateral nasal discharge for the last six weeks What is the most
likely diagnosis? (FMDC)
 Rhinolith*
 Infective Rhinitis
 Nasal Allergy
 Sinusitis
 Vasomotor Rhinitis
291. Best method for removing a rounded FB from nose is by: (FMDC)
 Suction
 Crocodile forceps
 Tilly forceps
 Eustachian catheter*
 Jobson probe
292. Best way to assess nasal bone fracture is: (FMDC)
 Digital palpation*
 Xray nasal bone lateral view
 X-ray nasal bone AP view
 X-ray PNSE
 CT scan PNS
293. A young man presented to ENT clinic with H/O excessive sneezing, nasal discharge and
some nasal blockage. He also gave H/O itching in the nose and eyes. On enquiring he added
that these symptoms typically disturbed him in the month of February, March and April. In
your opinion what is the problem with this individual? (FMDC)
 Seasonal allergic rhinitis *
 Perineal rhinitis
 Vasomotor rhinitis
 infective rhinitis
 Rhinitis Medicumentosa
294. A 25 years of age female patient presented with facial pain which typically aggravated by
bending down. She also c/o post nasal drip. What do you think this lady in suffering from?
(FMDC)
 Rhinitis
 Sinusitis *
 Neurogenic pain
 Unexplained headache
295. A 6 years old boy came to your clinic with his parents they complaints their son has
recurrent epistaxis from left nostril for the last three weeks. On examination there was no
active bleeding, but a bleeding point was visible on the anterior half of left side of nasal
septum. How would you proceed with this ease? (FMDC)
 Anterior nasal pacing
 Postnasal packing
 Chemical cautery with silver nitrate.*
 Electric cautery
 Pinching the nose
296. A known case of allergic rhinitis visited you for consultation. He was quite disturbed from
his diseases. What is the best treatment option? (FMDC)
 Antihistamine*
 Analgesic
 Antihistamine plus topical corticosteroid
 Topical decongestant
 Polyfax ointment
297. Skin prick tests are recommended for investigation of: (FMDC)
 Allergic rhinitis*
 Vasomotor rhinitis
 Infective rhinitis
 Rhinosinusitis
 Pansinusitis
298. Bluish discoloration of nasal mucosa is a characteristic feature of (FMDC)
 VMR
 Allergic rhinitis*
 Viral rhinitis
 Bacterial rhinitis
 Fungal rhinitis
299. All are complications of SMR except (FMDC)
 Septal perforation
 Septal abscess
 Nasal hump*
 Sadular deformity
300. About Septal perforation all true except (FMDC)
 Usually Patient remain unknown
 Small one can present with crossing and epistaxis
 May be complication of SMR
 Carcinoma Cannot present with perforation*
301. Recurrent respiratory papillomatosis is caused by (FMDC)
 respiratory syncytial virus
 Adenovirus
 Arbrovirus
 Coksackia virus
 None of the above *
302. The most common diagnosis in young boy with history of profuse nasal bleeding is
 juvenile angiofibroma *
 Nasal Polyposis
 deviated nasal septum
 cirrhosis
303. Maxillary sinus opens into
 superior meatus
 infundibulum
 inferior meatus
 middle meatus*
 none of above
304. which one of the complications contraindicates operation of SMR under age of 17 years?
(FMDC)
 difficulty in operation
 more chance of septal perforation
 retarded growth of nose*
 saddle nose
 septal hematoma
305. Regarding ethmoidal nasal Polyposis which one false (FMDC)
 multiple in number
 recurrence rate is very high
 diagnosed on posterior rhinoscopy*
 allergic in nature
 best option is FESS
306. Constituents of nasal septum include all except
 Vomer
 perpendicular plate of ethmoid*
 quadrangular cartilage
 maxillary crest
 membranous septum
307. 20-year-old male has presented with boil nose. Which one is most serious complication he
may develop if untreated? (RIHS)
 cellulitis of face
 cavernous sinus thrombosis*
 septal abscess
 septal perforation
 Vestibulitis
308. A patient presented with fever and rhinosinusitis. Most common pathogen is
 staph aureus
 streptococcus pneumonia*
 streptococcus pyogenese
 H. influenza
 Pseudomonas
309. Pathophysiology of nasal polyp are all except:
 complex anatomy
 poor ventilation
 allergy
 poor blood supply*
 infection
310. Complications following septal abscess include all except
 severe epistaxis*
 depression of nasal bridge
 meningitis
 cavernous sinus thrombosis
 A&c
311. Narrowest part of airway in children. (HBS)
 Glottis
 Supraglottis
 Subglottis*
 Carina
 Above epiglottis
312. A 15-year-old has unilateral nasal obstruction, mass in the cheek and recurrent epistaxis,
the diagnosis is:
 Cancer of nasopharynx.
 Inverted papilloma nose.
 Maxillary sinusitis.
 Bleeding tendency.
 Angiofibroma*
313. Round, smooth, soft, translucent, yellow or pale, glistening structure which result from
prolapsed lining of the ethmoid sinus and blocks the nose to variable degree depending on
their size: (HBS)
 Antro choanal polyp.
 ethmoidal nasal polyp*
 Inverted papilloma.
 Furuncolosis.
 Nasal turbinate.
314. Blood supply of the nose: (HBS)
 Supplied by external carotid artery only.
 Supplied by internal carotid artery only.
 There is no connection with the sagittal sinus*
 The maxillary artery is the main arterial supply to the nasal fossa.
 The facial artery is a terminal branch of the external carotid artery
315. The most common cause of epistaxis is: (HBS)
 Hypertension.
 Idiopathic*
 Fracture nasal bone.
 Blood diseases
 Angiofibroma.
316. A complication of common cold: (HBS/IMDC)
 Otitis externa.
 Quinsy.
 Facial palsy.
 Maxillary sinusitis*
 Inverted papilloma.
317. Symptoms of acute staphylococcus infection of nasal vibrissae include all but: (HBS)
 Pain.
 Fever
 Swelling
 epistaxis. *
 Headache
318. Mast cell stabilizers: (HBS)
 Can be used in treatment of all kinds of allergic disease.
 Usually used as the main treatment line in allergic rhinitis patients.
 Mostly used as prophylactic in seasonal rhinitis.
 Used as one of the cornerstones in the treatment of allergic rhinitis.
 All the above*
319. The main problem in using immunotherapy (hypo-sensitization) in allergic rhinitis patient
is: (HBS)
 The cost.
 Risk of hypersensitivity reaction.
 Difficulty with identification of the antigen.
 The technique.
 All the above*
320. Treatment of common cold include all but: (HBS)
 Steam inhalation
 Warm fluid
 Steroid spray*
 Bed rest
 Analgesia
321. In seasonal rhinitis, the early symptom will be: (HBS)
 Paroxysms of sneezing occur at frequent intervals throughout the day*
 Watery rhinorrhea.
 Loss of smell and loss of taste.
 Postnasal drip is not common.
 Facial pain
322. Proof puncture (Antral wash out) was planned in a 30 Yr male patient What common
complications do you expect in this operation:
 Damage to teeth
 Orbital & cheek injury*
 Meningitis
 injury to infraorbital nerve
 CSF rhinorrhea
323. The single most valuable X-ray of the mid-face is:- (FMDC)
 Water's view*
 Lateral view
 Caldwell view
 Towne's view
437. A 50 year old man cocaine sniffer presents with repeated episodes of epistaxis associated with
whistling on breathing in through the nose. The likely diagnosis is (RIHS)
 Maxillary sinus carcinoma
 Septal perforation*
 Bleeding polypus
 Bleeding diathesis
 Maxillary sinusitis
438. A 45 years old lady presents with left sided nasal obstruction, blood stained discharge and foul
smell for the last 5 years. Nasal obstruction is progressive. On examination, there is dirty grayish
white mass at the floor and inferior meatus of nasal cavity on left side. It is stony hard with
grating sensation on palpation with a probe What is one probable diagnosis? (RIHS)
 Osteoma maxilla
 Hypertrophic turbinate
 Malignant growth
 Rhinolith*
 Choanal atresia
439. A 17 year old boy presents with history of massive recurrent epistaxis, nasal obstruction and
anemia. His posterior rhinoscopy reveals a reddish mass filling the nasopharynx The most likely
diagnosis is. (RIHS)
 Adenocarcinoma
 Angiofibroma*
 Adenoids
 Nasopharyngeal Carcinoma
 Squamous cell carcinoma
440. You are planning operation in 30 years male for nasal obstruction, On examination there is PNS
left with right sided hypertrophy of inferior turbinate, x-ray PNS sinus shows opaque maxillary
sinuses. Which one is best option? (RIHS)
 Septoplasty &Proof puncture
 Submucoperichondral resection and Proof puncture
 Septoplasty and Proof puncture and right inferior turbinectomy*
 SMR and right inferior Turbinectomy
 SMR Only
441. Dorsum of the nose is formed by all except.
 Septal cartilage.
 Nasal bone
 Upper lateral cartilages
 Vomer*
 Lower lateral cartilages
442. Mucosa of the nasal cavity is formed by all but:
 Skin
 Columnar ciliated epithelium
 Olfactory epithelium
 trans epithelium *
 Respiratory epithelium
443. What is most true about the function of the paranasal sinuses?
 They protect the eye
 Help in olfaction
 They reduce the weight of the skull
 Do not appear to have a function
 They produce resonance of sound*
444. All are false about Nasal septal hematoma except:
 It is only associated with trauma
 It is benign tumour
 It is a collection of blood between nasal mucosa and perichondrium*
 It is mostly unilateral
 It is mainly treated by aspiration
445. Regarding deviated nasal septum. which of the following is false?
 Deviated nasal septum (DNS) is a common physical disorder
 It is most frequently caused by impact trauma
 Deviated septum is associated with genetic connective tissue disorders such as Madan
syndrome*
 Septal haematoma and semi abscess are complications of its correction
 Mostly treated surgically
446. The true statement regarding Fractured nose is:
 It Occur in lower one third part of nasal dorsum
 It is fracture of septal cartilage
 Is the most common facial injuries*
 Its classification depends on the velocity of trauma only
 Diagnosis solely depends on the X-ray finding
447. The most common cause epistaxis in elderly patient is
 Nose picking
 Carcinoma nasal cavity
 High blood pressure*
 Granulomatous disease of nose
 Hemangioma of nasal septum
448. Treatment of choice in a 6-year-old boy deviated nasal septum is:
 SMR
 septoplasty
 external rhinoplasty
 endoscopic septoplasty
 conservative*
449. The lymphatic drainage of the nose:
 Go directly to the inferior deep cervical lymph nodes
 Never pass to the submental lymph nodes.
 The tip of the nose is devoid of lymphatic drainage
 The posterior part of the nose is drained to the retropharyngeal and upper deep cervical
lymph nodes*
 None of the above
450. Allergic rhinitis:
 It is an IgG mediated
 Rarely associated with bronchial asthma
 Productive cough is the main symptom
 It associated with purulent nasal discharge
 Local corticosteroids have a role in its management*
451. Unilateral foul smelling nasal discharge is a child could be due to
 Allergic rhinitis
 Foreign body nose*
 Vasomotor rhinitis
 Acute sinusitis
 Angiolibroma
452. Allergic rhinitis:
 It is type 2 allergic hypersensitivity reaction.
 It involves IgM reaction
 The mast cells have a major role in pathogenesis*
 Neutrophil count is essential for diagnosis
 It is the main cause of atopy
453. The most common diagnosis in a young boy with history of profuse nasal bleeding is
 Juvenile Neso pharyngeal angiofibroma*
 Nasal Polyp
 Deviated nasal septum
 Cirrhosis
 Congenital heart disease
454. All about the vestibule of the nose are true except
 It is the entrance to the nasal cavity
 It is lined skin
 It contains sebaceous gland and hair
 It is lined by mucus membrane of respiratory *
 Can be wen without nasal speculum
455. A 45 years old watchman has presented with complain of bilateral nasal obstruction of
2months duration which is of gradual progressive onset. He had two operations in the nose in
the past for the same condition. He in otherwise healthy. Examinations shows both nostrils
filled with grayish white grape like structures with glistening surface. MRI show involvement of
sphenoid & ethmoidal sinusitis both sides. Give one best choice of treatment: (RIHS)
 Caldwell welluc and external spheno ethmoidectomy
 Transitional ethmoidectomy (hergan’a operation)
 Functional endoscopy sinus surgery*
 Acute canine of oral and local
 Bilateral intra nasal polyectomy
456. A woman came to ENT OPD with nasal obstruction, nasal discharge, postnasal discharge and
facial pain for 3 Week, What yours diagnosis? (RIHS)
 Acute rhinitis
 Chronic rhinitis
 Acute sinusitis
 Chronic sinusitis*
 Atrophic rhinitis
457. A patent after RTA presented In Emergency department with severe bleeding from the nose
shortness of breath and multiple wounds in the body. The first important thing you will do is.
(RIHS)
 Pass I/V canula
 Anterior nasal packing
 Pinching the no se
 Secure airway*
 Blood transfusion
458. A 14 years old boy Is having nasal obstruction, nasal discharge and episodic epistaxis for last 3
months. Posterior rhinoscopy shows soft mass in the nasopharynx. The most likely diagnosis is
(RIHS)
 Angiofibroma*
 ethmoidal polyp
 Adenoids hypertrophy
 Antrochoanal polyp
 Nasopharyngeal carcinoma
459. A boy has fever and severe pain in vestibule of the nose. The area is swollen and tender, What
organism is mostly involved: (RIHS)
 H. Influenzas
 Streptococcus viridans
 Pseudomonas aeruginosa
 Staphylocorrus aureus *
 Moraxella Catarrhalis
460. Chronic rhinosinusitis is best diagnosed on (RIHS)
 X-ray PN5 and anterior rhinoscopy*
 Clinical examination only
 History and Examination
 CT scan and Nasoendoscopy
 Transillumination lest
461. A A0 years old female known diabetic presented with fever, nasal obstruction and discharge.
On examination she had black necrotic area over the hard palate and inferior turbinate Your
diagnosis Is (RIHS)
 Nasopharyngeal carcinoma
 Ethmoidal polyp
 Acute fungal sinusitis*
 Leprosy
 Maxillary sinusitis
462. A 25 years old female has facial pain, nasal obstruction and discharge for 8 months. X-ray PNS
shows haziness in sinuses. What is the best treatment option for: (RIHS)
 Septoplasty after 1 weeks
 Nasal decongestants & steroids/antibiotics/analgesics
 Endoscope sinus surgery*
 I/V antibiotics and Proof puncture.
 Caldwell Luc operation
463. Rhinosinusitis is clinically diagnosed when there are at least (RIHS)
 3 major symptoms.
 4 major symptoms
 1 major and 2 minor symptoms*
 2 major and 2 minor symptoms
 2 major and 3 minor symptoms
464. In Young's operation for Atrophic rhinitis, the Following statement is true (RIHS)
 Partial closure of one nasal cavity
 Partial closure of both nasal cavities
 Complete closure of one nasal cavity*
 Elevation of vestibule of nose
 Irrigation with saline solution
465. A female patient complaining of nasal obstruction and rhinorrhea for 2 months. She is using
nasal spray for the last 2 months. What treatment will you advise (RIHS)
 Contribute spray for 2 more months
 Discontinue nasal spray*
 Oral decongestants
 Septoplasty
 Antibiotic and antihistamines
466. The most common complaint of a patient with Hypertrophic rhinitis is (RIHS)
 Anosmia
 Nasal obstruction*
 Nasal discharge
 Bleeding from nose
 Pain in the nose
467. A patient suffering from nasal obstruction, rhinorrhea, post nasal drip, fever, facial pain across
the cheeks below night orbit & right upper jaw for the last 04 days. What is one most probable
diagnosis (RIHS)
 Acute rhinitis
 Acute pharyngitis
 Acute maxillary sinusitis right side *
 Frontal sinusitis
 Allergic rhinitis
468. A 30 years old female has frequent recurrence of boil nose. What is one possible cause (RIHS)
 Resistant organism
 Inadequate treatment
 Any underlying immuno compromised state *
 Poor hygiene
 Tympanic perforation
469. Most serious foreign body in the nose is: (RIHS)
 Вead
 Rubber
 Peanut
 Battery cell *
 Wood piece
470. A boy was hit on the nose while playing football, He had mild bleeding from the nose. After day
he developed bilateral nasal obstruction. On anterior rhinoscopy there was swelling on both
side of nasal septum. The likely diagnosis is: (RIHS)
 DNS
 Septal hematoma *
 Septal abscess
 Fracture of nasal septum
 Fracture of nasal bone
471. The simplest form of fracture nasal bone which is caused by minimal trauma is called: (RIHS)
 Chavallet fracture*
 Jarjavay fracture
 Nasoethmoido-orbital fracture
 Le-Fort 1 fracture
 Lefort 11 fracture
472. One of the Following statement is Not correct: (RIHS)
 Septoplasty can be done in children even at 8 years of age
 Post-operative Septal hematoma should be drained immediately
 Depression of nose is common in SMR as compared to Septoplasty
 Chances of septal hematoma is more common after Septoplasty*
 Past-operative nasal packing is usually done both in SMR & Septoplasty
473. A 14 years old girt is having unilateral nasal obstruction, discharge and episodic bleeding for
the last few months. On examination she is having smooth insensitive mass in left nasal cavity
extending and hanging posteriorly into the nasopharynx. The diagnosis is: (RIHS)
 Ethmoidal polyp
 Anto-choanal polyp*
 Angiofibroma
 Septal hematoma
 Foreign body nose
474. Which of the following is Not included in features of a patient with allergic rhinitis? (RIHS)
 Excessive sneezing
 Rhinorrhea
 Nasal obstruction
 Shortness of breath*
 Itching in nose
475. A boy developed fever, nasal obstruction and discharge a day after swimming. He is having
headache only whenever he bends downward. The likely cause is: (RIHS)
 Frontal sinusitis*
 Maxillary sinusitis
 Allergic rhinitis
 Atrophic rhinitis
 Acute rhinitis
476. Which of the following statement is true regarding choanal atresia: (RIHS)
 the most common variety is bilateral choanal atresia
 90% of it is bony and 10% is membranous*
 10% of it is bony and 90% is membranous
 Unilateral choanal atresia is an emergency
 Treatment of choice is medical
477. A 2 years old child is having foul smelling unilateral nasal discharge for 4 months. He has been
taking medications for 2 weeks. Anterior rhinoscopy shows purulant discharge. What should be
done next? (RIHS)
 Oral antibiotics
 Nasal steroids
 X-ray nose
 Examination under anesthesia*
 Suction and cleaning of nose
478. A 32 year old man with nasal polyp is admitted to hospital for polypectomy, After he surgery
developed unilateral watery nasal discharge. What is probable cause of it? (RIHS)
 Common cold
 Septal hematoma
 Acute Sinusitis
 Allergic rhinitis
 CSF Rhinnorhea*
479. In Antral wash-out procedure for sinusitis, the maxillary sinus is approached through: (RIHS)
 Superior meatus
 Middle meatus
 inferior meatus*
 Canine fossa
 Nasal septum
480. Following organism is very lethal and can cause death in a patient with fungal sinusitis: (RIHS)
 Candida
 Aspergillus niger
 Aspergillus fumigates
 Culvlaria
 Mucor*
481. A newly born child is cyanosed which subsides on crying and this cycle is continuous. The newly
born is exhausting and in severe respiratory distress. On anterior rhinoscopy, nasal passages
appear adequate. Your provisional diagnosis is: (RIHS)
 Choanal Atresia*
 DNS
 Laryngeal papilloma
 Laryngon alacia
 Septal Hematoma
482. Vidian neurectomy is advised to a lady having excessive rhinorrhea. She is suffering from:
(RIHS)
 CSF rhinorrhea
 Allergic rhinitis
 Vasomotor rhinitis*
 Chronic sinusitis
 Rhinitis medicamentosa
483. A 15-year-old boy has unilateral nasal obstruction, mass in the check and recurrent epistaxis
diagnosis is: (YMDC)
 Cancer of Nasopharynx.
 Inverted papilloma
 Maxillary sinusitis.
 Bleeding tendency,
 Angiofibroma*
484. It is an IgE mediated hypersensitivity disease of the tympanic membrane if the nasal air way:
(YMDC)
 Acute rhinitis.
 Chronic rhinitis
 Vasomotor rhinitis.
 Allergic rhinitis*
 Furunculosis.
485. A ten year old girl presented with pain between the eye frontal headache, discharge from the
nose. past nasal drip and high fever; what is the provisional diagnosis: (YMDC)
 Acute frontal sinusitis*
 Acute ethmoidal sinusitis
 Acute sphenoidal sinusitis
 Sphenoidal tumor
 Chronic ethmoidal sinusitis
486. A 70 years old diabetic male presented with a painful Swelling of the nose. Inflammation of the
external nose. (YMDC)
 Furunculosis is due to streptococcus infection
 Furunculosis cannot be complicated by cavernous sinus thrombosis.
 Usually painless
 Need many investigations to diagnosis
 infection should be treated antibiotics*
487. A healthy 15 years male patient presented to the ENT department with right sided as nasal
obstruction, foul smelling and blood stained discharge. The most likely diagnosis is: (YMDC)
 Simple nasal polyp
 Rhinolith*
 Antrochoanal polyp
 Foreign body.
 Angiofibroma
488. The most common three causative bacterial agent of acute sinusitis are: (YMDC)
 Streptococcus pneumoniae, H. influenzas, Moraxella catarrhalis*
489. Local precipitating factor for maxillary sinusitis: (YMDC)
 Poor diet
 fatigue
 Nasal obstruction*
 Chilling
 Irritating atmospheric conditions
490. A young male, mountaineer by profession was referred to ENT department of Yusra Medical
and Dental College General physician in Gilgit with a provisional diagnosis of maxillary sinusitis.
Definite diagnosis of acute maxillary sinusitis is confirmed by: (YMDC)
 facial pain
 Antral lavage.
 Tenderness over the chest
 X-ray. waler s view*
491. The antrochoanal polyp: (YMDC)
 is common.
 It’s mostly unilateral,
 Extends immediately to the posterior
 All the above*
 None of the above
492. Best radiological test for maxillary sinus is: (YMDC)
 Occipitofrontal view.
 Occipitomental view.
 lateral view.
 CT scan*
 Submentovertical view
493. A 30 years old lady presented with antrochoanal causing nasal obstruction and rhinitis
Treatment of Antrochoanal poylp is: (YMDC)
 Topical steroids.
 Systemic steroids.
 Anti Histamine
 immunotherapy
 Surgery*
494. A previously healthy active 18-month-old child presents with unilateral nasal obstruction and
foul smelling discharge. The child's examination is other unremarkable. The most likely
diagnosis is? (YMDC)
 Foreign body*
 Nasal polyps
 Frontal sinusitis
 Deviated septum
 Choanal atresia

THROAT :
1. A 66-year-old Chinese man presents with a two-month history of neck swelling in the right
posterior triangle. He also has conductive hearing loss in right ear with blocked nose and
significant weight loss. What is the likely diagnosis? (HBS)
 Lymphoma.
 Nasopharyngeal carcinoma*
 Parotid tumor
 Lipoma
 Nasopharyngeal angiofibroma
2. A 21-year woman presents with two days history of sore throat and dysphagia. On
examination she is pyrexial with whitish membrane over left tonsil & pharynx. there is
cervical lymphadenopathy. What is the likely causative organism for her condition? (HBS)
 Epstein-Barr virus
 Haemophilus influenzae
 Corynebacterium diphtheriae*
 Neisseria gonorrhea
 Streptococcus pyogenes
3. A 55-year-old man presents with gradually enlarging right sided neck swelling with severe
ipsilateral otalgia for last 3 months. There is history of 10kg weight loss. He smokes 10
cigarettes per day and drinks 15 units of alcohol per week. Examination of the neck reveals
a 4 cm x 4 cm firm, mobile, non-fluctuant, and non-pulsatile swelling on the right side of
the neck. Rest of ENT examination is normal. Likely diagnosis in this case is (HBS)
 Branchial cyst.
 Malignant node containing squamous cell carcinoma*
 Infective lymph node.
 Submandibular gland stone.
 Non-Hodgkin lymphoma.
4. A 4-year-old child presented with ingestion of coin 30 minutes back, patient is
asymptomatic. The first step of management should be (HBS)
 Shift to operation theatre and esophagoscopy
 Shift to operation theatre and bronchoscopy
 Xray neck and chest*
 Heimlich maneouvre
 Encourage oral feed for spontaneous passage
5. The most common and earliest manifestation of carcinoma of the glottis is: (HBS)
 Hoarseness*
 Haemoptysis
 Palpable Cervical lymph nodes
 Stridor
 Dysphagia
6. What virus is associated with nasopharyngeal carcinoma? (HBS)
 Human Papilloma Virus
 Herpes Simplex Virus
 Epstein-Barr Virus*
 Coxsakie Virus
 Varicella-Zoster Virus
7. The most common cause of bilateral recurrent laryngeal nerve paralysis is: (HBS)
 Laryngeal trauma,
 Iatrogenic damage in total thyroidectomy*
 A tumor mass expanding in upper part of mediastinum,
 Toxic polyneuropathy of recurrent laryngeal nerves,
 Acute subglottic laryngitis.
8. 62-year-old patient has presented with non-healing ulcer on right lateral border of tongue
for last 4 months, there is history of significant weight loss. On examination right sided
cervical lymph nodes are palpable and painless, patient is known smoker for last 30 years.
the likely diagnosis is: (HBS)
 Aphthous ulcers
 Behcet syndrome
 Squamous cell carcinoma*
 Lichen planus
 Chemical burn due to cigarette smoking
9. 15-year-old boy with left nasal blockage and swelling of cheek is suspected of Juvenile
angiofibroma. The main feature in this disease is: (HBS)
 Recurrent epistaxis*
 It is a tumor of young girls
 Bone erosion of the greater wing of the sphenoid does not occur
 Sensorineural hearing loss
 External beam radiotherapy is first line treatment
10. A 2-month-old child presented with noisy breathing more during inspiration with supine
position and disappears on prone position. The appropriate diagnosis would be: (HBS)
 Acute epiglottitis
 Croup
 Choanal atresia
 Laryngomalacia*
 Sub glottis stenosis
11. A 77-year-old lady presented with dysphagia. There is history of 8kg weight loss over a
period of 3 months. On indirect laryngoscopy, there is pooling of saliva in right pyriform
sinus with swollen right arytenoid, the most likely diagnosis is: (HBS)
 Adenocarcinoma of pyriform sinus.
 Squamous cell carcinoma of pyriform sinus*
 Squamous cell carcinoma of posterior pharyngeal wall.
 Squamous cell carcinoma of larynx.
 Squamous cell carcinoma of Postcricoid.
12. 15-year-old male presented with history of odynophagia, fever and sore throat for last 3
days. On examination there is a membrane over tonsils. There is no history of dyspnea or
cervical lymphadenopathy. Which of the following tests will help to make final diagnosis?
(HBS)
 Throat swab Gram staining*
 Monospot test
 CRP (c reactive protein)
 Complete blood count
 Urine examination
13. A newborn baby presented with weak cry and hoarseness, There is inspiratory stridor that
disappears on prone position. The appropriate management would be (HBS)
 Give corticosteroids
 Tracheostomy
 Wait and watch for spontaneous resolution of condition*
 Give 1-week course of antibiotics
 Nebulize with epinephrine solution
14. Which is the most common aetiological agent of epiglottitis (HBS)
 Staphylococcus aureus
 Haemophilus influenza (type B)*
 Rhinovirus
 Adenovirus
 Moraxella catarrhalis
15. A 42-year-old female presented with lump in left preauricular region that is gradually
progressively increasing for last 4 years, there is no pain, facial weakness or trismus. The
appropriate diagnosis will be (HBS)
 Adenoid cystic carcinoma
 Acinic cell carcinoma
 Pleomorphic adenoma*
 Mucoepidermoid carcinoma
 Sialolithiasis
16. Recurrent multiple aphthous ulcers of oral cavity are best treated by: (HBS)
 NSAIDS
 Multivitamins
 Topical Corticosteroids*
 Tricyclic Antidepressants
 Warm Saline gargles
17. Sialolithiasis is a disease of salivary glands characterized by formation of stone. The most
commonly affected salivary gland is (HBS)
 Sublingual salivary gland
 Submandibular salivary gland*
 Parotid gland
 Minor salivary glands
 Accessory parotid duct
18. 65-year-old male patient presented with dysphagia, regurgitation of food and halitosis,
barium swallow show blind pouch just below the level of cricopharynx. The appropriate
diagnosis in this case is (HBS)
 Carcinoma of hypopharynx
 Zenker's diverticulum*
 Foriegn body in esophagus
 Cardiac achalasia
 Esophageal stricture
19. A 15-year-old mạle has unilateral nasal obstruction, mass in the cheek and recurrent
epistaxis, the diagnosis is: (HBS)
 Cancer of nasopharynx.
 Inverted papilloma nose.
 Maxillary sinusitis.
 Bleeding tendency.
 Angiofibroma*
20. Easiest way to diagnose Group A streptococcal pharyngitis in clinic is; (HBS)
 Complete blood count.
 Throat swab for Gram staining
 Rapid antigen detection test*
 Throat swab for culture
 Procalcitonin levels
21. 45-year-old female complains of dysphagia, angular stomatitis, koilonychia and tongue
inflammation. Her complete blood picture reveals hemoglobin level 8mg/dl with increase
total iron binding capacity suggesting iron deficiency anemia. The likely diagnosis is (HBS)
 Cardiac Achalasia
 Plummer vinson syndrome*
 Esophageal stricture
 Diffuse esophagitis
 Diffuse esophageal spasms
22. A three-month-old boy get low grade fever, continuous cough with biphasic stridor, the
resident pediatrician sent for lateral neck x-ray and anteroposterior cervical spine X-ray
that shows steeple sign. The likely diagnosis is: (HBS)
 Acute pharyngitis.
 Acute simple laryngitis.
 Laryngo-tracheo-bronchitis*
 Acute epiglottitis.
 Adenoids hypertrophy
23. Lymphoid tissue aggregate that is midline and located in nasopharynx is: (HBS)
 Palatine tosils
 Lingual tonsils
 Adenoids*
 Pharyngeal bands
 Tubal tonsils
24. 32-years-old male presented with history of diffuse swelling in submental region for last 2
days, on palpation, swelling is tender and having woody hard feel, patient is having marked
trismus and fever, the likely diagnosis is (HBS)
 Sublingual sialadenitis
 Submental lymphadenitis
 Ludwig's angina*
 Plunging ranula
 Infected dermoid cyst
25. 32-year-old male presented with history of diffuse long standing obstruction due to
enlarged tonsils and adenoid can cause (HBS)
 Left ventricular hypertrophy
 Bundle branch block
 Cor pulmonale*
 Cardiac ischemia
 Rheumatic heart disease
26. A 35-year-old male presented with fever, trismus, odynophagia. On examination left sided
neck swelling and tonsil is pushed medially, there is a history of 3d molar tooth extraction 1
week back. The diagnosis is (HBS)
 Retropharyngeal abscess
 Peritonsillar abscess
 Parapharyngeal abscess*
 Ludwig's angina
 Parotid abscess
27. Earache following tonsillectomy is referred through which nerve: (HBS)
 Vagus nerve
 Glossopharyngeal nerve*
 Accessory nerve
 Trigeminal nerve
 Facial nerve
28. 40-year-old patient underwent esophagoscopy for obstructed meal bolus. 6 hours after
surgery patient develop severe pain in interscapular region. The most likely diagnosis is
(HBS)
 Retained foreign body
 Esophageal perforation*
 Mediastinal abscess
 Major vessel injury
 Diffuse esophagitis
29. which of the following is the narrowest part of airway? (HBS)
 Supraglottis
 Subglottis
 Glottis*
 Trachea
 Cricopharynx
30. Which of the following is the abductor of larynx? (HBS)
 Lateral cricoarytenoid
 Posterior cricoarytenoid*
 Cricothyroid
 Transverse arytenoid
 Vocalis
31. 1-month old child is suspected for laryngomalacia, his fibreoptic laryngoscopy is planned,
which of the following best describe the shape of epiglottis in this patient (HBS)
 "C" shaped
 Omega shaped*
 Elongated
 Delta shaped
 Flat
32. A three-year-old child presented to emergency department with complaint of high grade
fever, dyspnea and lethargy, condition is somewhat relieved by sitting position with
bending forward. The likely diagnosis in this case would be (HBS)
 Acute laryngotracheobronchitis
 Acute epiglottis*
 Acute peritonsillar abscess
 Acute tonsillitis
 Parapharyngeal abscess
33. A boy, 6 years of age present with a mass in the midline of neck just below the hyoid bone
noted 6 months back. It moves with protrusion of tongue. The most probable diagnosis is:
(HBS)
 Cystic hygroma
 Thyroglossal duct cyst*
 Hemangioma.
 Branchial cleft cyst
 Dermoid
34. 6-year-old child complaints of mouth breathing. snoring and recurrent apneic spells during
sleep. Her x-ray nasopharynx was done showing marked adenoid hypertrophy, she also
complaints of decreased hearing and her tympanometry shows type B flat curve showing
conductive hearing loss, the appropriate management will be (HBS)
 Wait and watch
 Give nasal decongestants with antibiotics
 Adenoidectomy
 Adenoidectomy, myringotomy with insertion of grommets*
 Myringotomy with grommet insertion
35. Six-month-old child present with barking seal type cough, there is low grade fever with mild
respiratory distress, the main causative organism for this condition is: (HBS)
 Hemophilus influenza
 Streptococcus pneumonia
 Influenza virus*
 Moraxella catarrhis
 Staphylococcus aureus
36. One of the main indication of tracheostomy is: (HBS)
 Lower airway obstruction
 Cardiac ischemia
 Prolong ventilatory support*
 Bronchiectasis
 Pneumothorax
37. Most common cause of stridor in infant is: (HBS)
 Vocal cord paralysis
 Laryngeal web
 Subglottic stenosis
 Laryngomalacia*
 Subglottic hemangioma
38. Post streptococcal glomerulonephritis is a rare complication that is associated with
streptococcal (HBS)
 Acute pharyngitis
 Acute tonsillitis*
 Acute adenoiditis
 Acute laryngitis
 Acute rhinosinusitis
39. Koplik's spots are pathognomonic of; (HBS)
 Diphtheria
 Streptococcal sore throat
 Mumps
 Measles*
 Glandular fever
40. Demonstration of an air containing sac in the neck after Valsalva maneuver suggests: (HBS)
 Bronchial cleft cyst
 Thyroglossal duct cyst
 Laryngocele*
 Zenker's diverticulum
 Cricoaryteroid muscle
41. A 25 year old male presents with 5 days history of progressive right sided odynophagia
along with drooling. patient cannot take per oral. There is history of fever, tachycardia, and
trismus. On examination there is a bulge in right oropharynx with shift of uvula to opposite
side. The appropriate diagnosis is (HBS)
 Acute epiglottitis
 Aeritonsillar abscess*
 Bezold Abcess
 Acute tonsillitis
 Retropharyngeal abscess
42. A 7-year-old male patient is brought in OPD with history of mouth breathing, recurrent
chest infection and disturbed sleep. On examination this child has prominent incisors. He is
breathing through the mouth with high arched palate and on ear examination, he has
bilateral conductive hearing loss. The child has (HBS)
 Chronic rhino sinusitis
 Adenoids hypertrophy*
 Angiofibroma
 Chordoma
 Bilateral intra nasal polyp
43. When an emergency opening into the airway is required, the cricothyroid membrane
should be pierced immediately (HBS)
 Superior to thyroid cartilage
 Between 2d and 3d tracheal ring
 Inferior to cricoid cartilage
 Superior to cricoid cartilage*
 Superior to thyroid isthmus
44. 20-year-old male patient underwent tonsillectomy. 1 week later patient presented with
bleeding from mouth with hematemesis, the most common cause of secondary
hemorrhage after tonsillectomy is (HBS)
 Slippage of ligature
 Damage to blood vessels
 Infection*
 Bleeding disorders
 Trauma to soft palate
45. which nerve is at risk while performing total parotidectomy for parotid carcinoma (HBS)
 Ophthalmic branch of Trigeminal nerve
 Maxillary branch of trigeminal nerve
 Facial nerve*
 Vagus nerve
 Mandibular branch of trigeminal nerve
46. Airway obstruction in case of Ludwig's angina results due to (HBS)
 Swelling in submaxillary space
 Cellulitis in floor of mouth
 Pus formation in submental space
 Pushing back of tongue*
 Infection spreading to parapharyngeal space
47. Peritonsillar abscess mostly affects (HBS)
 Children
 Adults*
 Males
 Females
 Elderly
48. First step in treatment of peritonsillar abscess is; (HBS)
 Oral antibiotics
 Intramuscular analgesic
 Incision and drainage*
 Hot tonsillectomy
 Tracheostomy
49. Which head and neck space runs from base of skull to соссух? (HBS)
 Retropharyngeal space
 Parapharyngeal space
 Prevertebral space*
 Submaxillary space
 Parotid space
50. Chronic retropharyngeal abscess is caused by. (HBS)
 Staphylococcus aureus
 Streptococcus pyogenes
 Pseudomonas aeruginosa
 Mycobacterium tuberculosis*
 Hemophilus influenzae
51. 45-year-old patient, teacher by profession develops roughness of voice for last 3 month, on
examination there is bilateral round swellings over vocal cords, the appropriate diagnosis is
 Vocal polyp
 Vocal nodules
 Rienke‘s edema*
 Vocal papillomatosis
 Carcinoma of larynx
52. Cause of hoarseness after total thyroidectomy is due to the iatrogenic damage to
 Superficial laryngeal nerve
 Vagus nerve
 External laryngeal nerve
 Internal laryngeal nerve
 Recurrent laryngeal nerve*
53. Omega shaped epiglottis is seen in:
 Sub glottis stenosis
 Acute epiglottitis
 Vocal cord paralysis
 Laryngomalacia*
 Acute laryngotracheobronchitis
54. A 2 year old child presented with noisy breathing more during inspiration along with stridor
and high grade fever. Condition gets improved on leaning forward and worse on lying
down. The appropriate diagnosis would be:
 Choanal atresia
 Laryngomalacia
 Croup
 acute epiglottitis*
 Sub glottis stenosis
55. Ludwig's angina is the infection of following space:
 Paritonsillar
 Parapharyngeal
 Parotid
 Retropharyngeal
 Submandibular*
56. 5 year old child presented with midline neck swelling that moves with deglutition and
tongue protrusion. The likely diagnosis is
 Goiter
 Lingual thyroid
 Lymphadenitis
 Thyroglossal duct cyst*
 Dermoid cyst
57. 15 year old patient presented with bilateral parotid swelling for last 5 days, associated with
fever, mild flue and lethargy, 2 week later patient developed bilateral scrotal swelling with
pain. The likely diagnosis is
 Parotid tumor with metastasis
 Sialoadenosis
 Bacterial parotitis
 Sjogren 1 s syndrome
 Mumps *
58. On CT scan of PNS, the uncinate process of ethmoid bone articulates with: (IMDC)
 Superior turbinate
 Middle turbinate
 Inferior turbinate *
 Ethmoid bone
 Palatine bone
59. On IDL examination, Unilateral vocal cord is immobile. The most common cause of
unilateral vocal cord paralysis is (IMDC)
 Thyroid surgery*
 Thoracic surgery
 Stroke
 Idiopathic
 Tuberculosis
60. A 25 years' patient presented with permanent hoarseness of voice, two nodular swellings
seen on both vocal cords. Which of the following is true? (IMDC)
 They are often seen in men
 They are mostly bilateral*
 They are mostly treated surgically
 They are rarely associated with vocal abuse
 CO2 LASER is rarely helpful
61. A 25 years old women with a turbulent married life is brought to emergency with 3 hours
history of inability to speak. She is otherwise conscious. IDL shows normal-appearing vocal
cords that fail to adduct on attempted phonation. Her cough is normal. Choose the most
likely pathology from the following? (IMDC)
 Bilateral abductor palsy
 Vocal cords nodules
 Myasthenia gravis
 Functional aphonia *
 Fixation of cricoarytenoid joint
62. After Thyroid surgery, patient presented with hoarseness of voice, respiratory distress on
exertion. IDL examination will be (IMDC)
 Bilateral vocal cord paralysis with vocal cords near the midline
 Bilateral vocal cord paralysis with vocal cord away from the midline in cadaveric position*
 Unilateral cord is in lateral position
 Loss of tension in one cord
 one cord was normal, other close to midline and immobile
63. In a patient tracheostomy was planned, will help the patient in all except (IMDC)
 Bilateral vocal cord paralysis with vocal cords near the midline
 Bilateral vocal cord paralysis with vocal cords away from the midline in cadaveric position
 Laryngomalacia*
 Left Unilateral vocal cord paralysis
 Acute epiglottitis
64. In 35 years' patient Voice is good with severe dyspnea (IMDC)
 Bilateral recurrent nerve paralysis*
 Bilateral superior laryngeal nerve paralysis
 unilateral recurrent nerve paralysis
 unilateral vagus nerve paralysis
 Bilateral Vagus nerve paralysis
65. In 35 years' patient, slight dysphonia without dyspnea (IMDC)
 Bilateral recurrent nerve paralysis
 Unilateral superior laryngeal nerve paralysis
 unilateral recurrent nerve paralysis*
 unilateral vagus nerve paralysis
 Bilateral Vagus nerve paralysis
66. A 25 years' hawker presented with hoarseness of voice, all are caused by voice abuse
except (IMDC)
 Vocal nodules
 Reinke’s Edema
 Vocal cord polyp
 Papilloma larynx*
 Chronic hypertrophic laryngitis
67. Internal branch of superior laryngeal nerve pierce (IMDC)
 Cricothyroid membrane
 Thyrohyoid membrane *
 Cricovocal membrane
 Aryepiglottic membrane (Fold)
 Quadrangular membrane
68. Examination of larynx by palpation includes (IMDC)
 Indirect laryngoscopy (IDL)
 Fibre-optic laryngoscopy
 Laryngeal crepitus *
 Direct laryngoscopy
 Laryngopharyngoscopy
69. Bell’s palsy not responding to steroid what will be the further line of management? (IMDC)
 Increase the dose of steroid
 Vasodilators and ACTH
 Surgical decompression*
 Electrical nerve stimulation
 Facial physiotherapy
70. A 20 years old girl presented to ENT OPD with odynophagia and trismus for the last 2 days.
One examination, floor of mouth is swollen & tongue seems to pushed upward &
backward. she has Hx of tooth extraction some days ago. The probable diagnose is (IMDC)
 Retropharyngeal abscess
 Parapharyngeal abscess
 Ludwig’s angina *
 Parotid abbess
 Dental abscess
71. A 65 years old male presented to ENT OPD with dysphagia to solids from last 2
months .patient is malnourished and gives Hx of gurgling sound is produced on swallowing
. He also complains of undigested food and regurgitation at night. Which investigation will
you carry out to confirm diagnosis? (IMDC)
 MRI
 CT Scan
 Barium swallow *
 Blood CP
 X ray lateral neck soft tissue
72. “Rat ail “’ appearance is the diagnostic feature of (IMDC)
 Hiatus hernia
 Pre-vertebral space abscess
 Achalasia Cardia *
 Plummer Vinson syndrome
 Pharyngeal pouch
73. Most common cause of facial nerve paralysis is? (IMDC)
 Bell’s palsy*
 Cholesteatoma
 Mastoidectomy
 Parotid surgery
 Herpes zoster otitis
74. Slow growing tumor involving the facial nerve is (IMDC)
 Warthin tumour
 Lipoma
 Oncocytoma
 Mucoepidermoid carcinoma *
 Pleomorphic adenoma
75. Tip of tongue drains into (IMDC)
 Sub mandibular node
 Deep cervical nodes
 Jugulodiagestric nodes
 Superficial cervical node
 Sub mental lymph node*
76. Chances of Post cricoid CA in patients of Plummer Vinson syndrome IS (IMDC)
 30%
 10% *
 80%
 90%
 25%
77. Cortical mastoidectomy is also called (IMDC)
 Canal wall up mastoidectomy*
 Atticotomy
 Canal will down mastoidectomy
 Modified radical mastoidectomy
 Tympanoplasty
78. Adenoidectomy is indicated in all of the following conditions except: (IMDC)
 Otitis media with effusion.
 Nasal obstruction due to adenoidal hyperplasia.
 Allergic rhinitis in children. *
 Recurrent otitis media in children.
 Sleep apnea syndrome
79. Which statement is CORRECT regarding Juvenile Angiofibroma: (IMDC)
 Patients have often had repeated episodes of epistaxis. *
 It is a tumour of young girls with a mean age of 14 at presentation.
 Bone erosion of the greater wing of the sphenoid does not occur.
 Cervical lymphadenopathy is present
 External beam radiotherapy is first line treatment.
80. All are causes of congenital STRIDOR except: (IMDC)
 Acute epiglottitis *
 Laryngeal web.
 Subglottic stenosis
 Laryngomalacia.
 Vascular anomaly.
81. Absolute indication of tonsillectomy is: (IMDC)
 Chronic tonsillitis.
 Diphtheria carrier.
 Obstructive sleep apnea *
 Glossopharyngeal neurectomy
 Rheumatic fever.
82. Which of the following is not a complication of acute tonsillitis? (IMDC)
 Peritonsillar abscess
 Bezold’s abscess *
 Acute rheumatism
 Acute nephritis
 Acute infection of middle ear cleft
83. Trismus accompanying Quinsy is due to spasm of which muscle? (IMDC/YMDC)
 Masseter
 Pharyngeal constrictors
 Medial pterygoid *
 Temporalis
 Palatopharyngeus
84. Indication of tracheostomy: (IMDC)
 In all cases of acute laryngitis
 In any intubated patient within 3 days
 Unilateral Choanal atresia
 Bilateral vocal cord paralysis *
 If there is suspicion of Laryngomalacia
85. Lymphatic drainage of palatine Tonsil is (IMDC)
 Jugulodiagestric *
 Prelaryngeal
 Jugulo-omohyoid
 Sub mental
 Para tracheal
86. Commonest causative organism leading to Acute Laryngo-tracheo-bronchitis is (IMDC)
 Staphylococcus aureus
 Streptococcus
 Para influenza *
 Corynebacterium diphtheria
 H.Influenza
87. A boy 9 years of age presents with a mass in the midline of neck just below the hyoid bone
noted 6 months back. It moves with protrusion of tongue. The most probable diagnosis is:
(IMDC)
 Cystic Hygroma.
 Thyroglossal duct cyst*
 Haemangioma.
 Branchial cleft cyst.
 Dermoid cyst.
88. Nasal perforation in bony part is caused by (IMDC)
 Syphilis *
 Atrophic rhinitis
 Systemic lupus erythematosis
 Tuberculosis
 Wagner granulomatosis
89. Regarding bell’s palsy all are true except (RIHS/FMDC)
 Chorda tympani may be involved
 Idiopathic in nature
 Upper motor neurone type paralysis*
 Lesion is at Geniculate ganglion
 Family history may be important
90. Best treatment for attico antral type of disease is (RIHS/FMDC)
 Cortical mastoidectomy
 Canal wall down surgery*
 atticortomy
 myringotomy
 Tympnaoplasty
91. The most common malignancy of Oral Cavity is (RIHS)
 Adenocarcinoma
 Basal cell carcinoma
 Squamous cell carcinoma*
 Lymphoma
 Leiomyosarcoma
92. All are true about Rannula except (RIHS/FMDC)
 Swelling of sublingual gland
 Retention cyst
 Blue in color
 Punjent type seen in neck
 Is a tumour*
93. Adult male having swelling and pain in left submandibular region aggravates on chewing
especially sore foods for the last 2 years. On bimanual examination, left Submandibular
gland is enlarged and tender. What is the single most probable diagnosis? (RIHS)
 Malignant growth left Submandibular gland
 Benign growth left Submandibular gland
 Stone in left submandibular duct*
 Acute Submandibular abscess
 Acute sialadenitis
94. Trismus accompanying Peritonsillar abscess is due to spasm of which muscle? (RIHS)
 Masseter
 Pharyngeal constrictors
 Medial Pterygoid*
 Temporalis
 Lateral Pterygoid
95. A 4 years old male child is suffering from low grade fever and sore throat. On examination,
he has toxic look &has membrane adherent to tonsils and adjoining pharynx which is
difficult to be removed. His pulse is fast and thready. Multiple lymph nodes in neck are also
enlarged. What is the one most probable clinical diagnosis? (RIHS)
 Acute follicular tonsillitis
 Pharyngeal diphtheria*
 Ludwig’s angina
 Infectious mononucleosis
 Peritonsillar abscess
96. A 48 years old male complains of persistent sore throat more on right side for the last 2
months. It is not responding to antibiotics. On examination of throat an ulcer is seen on
right tonsil. Neck examination reveals enlarged right jugulodigastric lymph node, firm in
consistency but non-tender. Which one is the best investigation to confirm the diagnosis?
(RIHS)
 Throat swab for smear examination and culture
 Blood CP & ESR
 Biopsy and histopathology of lesion *
 Bone marrow biopsy
 CT Scan
97. Most common pre-operative investigation for Angiofibroma (RIHS/FMDC)
 MRI
 CT Scan *
 Angiography
 X-ray nasopharynx
 Biopsy
98. A 7 years child who underwent tonsillectomy 5 days ago has reported in Emergency room
with history of high grade fever and bleeding from the throat. The best option regarding
the treatment is (RIHS)
 Cautery of bleeding point
 Control of bleeding with tight gauze packing
 Ligation of the bleeding vessel
 Admission + antibiotics
 Admission + antibiotics + blood transfusion if required *
99. A young patient presents with history of Dysphagia more to solids than liquids. The First
investigation you will do is (RIHS)
 Barium swallow
 Esophagoscopy*
 Laryngoscopy
 CT scan chest
 Lateral neck soft tissue x ray
100. Fossa of Rosenmullar is common site for (RIHS)
 Angiofibroma
 Adenoids
 Nasopharyngeal carcinoma*
 Nasal polyps
 Hypopharyngeal carcinoma
101. A 4 years old child is suffering from dyspnea, stridor & dysphagia. Clinical diagnosis is
Acute Epiglottitis. Which is first and most important step to be taken in this patient? (RIHS)
 Start broad spectrum I/V antibiotics
 Carry out clinical ENT examination to confirm the diagnosis
 Ask for throat swab C/S
 Secure airway by intubation/tracheostomy*
 Lateral neck soft tissue x ray
102. A 50 year old man who smokes 20 cigarettes a day presents with a 2 month history of
hoarseness and dyspnea. A mass is palpable in neck on right side at level III. What is the
diagnosis? (RIHS)
 Carcinoma of larynx*
 Intubation granuloma
 Thyrotoxicosis
 Vocal cord nodules
 Vocal cord polyp
103. Early glottis carcinoma is best treated with (RIHS)
 Radiotherapy*
 Antibiotics
 Total laryngectomy
 Cryosurgery
 Chemotherapy
104. year old boy suddenly developed stridor, difficulty in breathing & Cyanosis while playing
(RIHS)
 Acute epiglottitis
 Tumour Larynx
 Laryngeal Web
 Foreign body airway*
 Laryngomalcia
105. Commonly used laser in ENT (RIHS)
 Carbon dioxide*
 ARGON
 YAG
 KAPT
 CRYO
106. A picnic party is going on a riverbank about 35kms away from the town. Suddenly a 35
years old male became severely dysphonic due to impaction of bolus of food in larynx. All
non-provocative procedures have failed to relive dyspnea. The person is becoming
increasingly cyanosed. Which one is the only choice to overcome this problem? (RIHS)
 Elective tracheostomy
 Cricothyrotomy*
 Intubation
 Direct Laryngoscopy and removal of foreign body
 Laryngeal mask
107. A 17 years old female presents in ENT OPD with complain of a swelling in front of her
neck. On examination, there was a single, non-tender swelling which was moving on
swallowing and on tongue protrusion. What is the most likely diagnosis? (RIHS)
 Branchial cyst
 Dermoid cyst
 Thyroglossal cyst*
 Thyroid nodule
 Cystic Hygroma
108. A 55 years old female patient has developed hoarseness of voice immediately following
thyroidectomy operation for follicular carcinoma. On indirect Laryngoscopy, left vocal cord
is immobile and lying in cadaveric position. What is one most probable cause? (RIHS)
OR
109. A 55 years old female patient has developed respiratory distress immediately following
thyroidectomy operation for follicular carcinoma. On indirect Laryngoscopy, left vocal cord
is immobile and lying in midline. What is one most probable cause? (RIHS)
 Injury to superior laryngeal nerve left side
 Infiltration of vocal cord by thyroid carcinoma
 Injury to left recurrent nerve*
 Injury to both recurrent laryngeal nerves
 Injury to superior laryngeal nerve right
110. A 35 years old female patient complains of persistent lump in throat and sticking of food
during swallowing for the last 2 years, X-rays lateral view soft tissue neck, barium swallow,
endoscopy, blood picture, electrolytes, PH and manometery studies have been normal.
What is the possible cause? (IMDC/RIHS)
 Severe esophagitis
 Gastro esophageal reflux
 Globus hystericuspharyngeous*
 Hiatus hernia
 Plummer Vinson syndrome
111. A 25 years old lady is suffering from progressive dysphagia for solids, of one year duration.
Now she developed hoarseness of voice for few days. On examination she looks anemic,
neck examination NAD. One most probable diagnosis (IMDC/RIHS)
 Reflux stricture
 Malignant growth oesophagus
 Plummer Vinson’s syndrome*
 Achlasiacardia
 CA hypo pharynx
112. A 28 year’ male presents with recurrent pain in the sore throat and swelling
submandibular region of left side which exaggerates on chewing and especially with soar
food for the last 2 years. On bimanual examination, left submandibular gland is enlarged
and tender.What is the single most probable diagnosis? (IMDC/RIHS)
 Malignant growth left submandibular gland
 Benign growth left submandibular gland
 Calculus left submandibular gland*
 Acute submandibular abscess
 Ludwig angina
113. Commonly used laser in ENT and in diseases of esophagus and larynx (IMDC)
 Carbon dioxide*
 Argon
 Yag
 Kapt
 Cryo
114. A 5 year’ old male presented with severe pain in chest during meal and dysphagia for last
5 years. On barium swallow rat tail appearance was seen. Most probable diagnosis. (IMDC)
 Carcinoma lower end of esophagus
 Plummer winson’ syndrome
 Gastro esophageal reflux
 Cardiac achalasia*
 Foreign body esophagus
115. A 50 year’ old male presented with swelling of right parotid region, facial nerve is intact.
Most common tumour of this gland at this age is. (IMDC)
 Pleomorphic adenoma*
 Mucoepidermoid
 Monomorphic adenoma
 Squamous cell carcinoma
 Adeno carcinoma
116. A 5 year’ old boy suddenly developed strider and severe odynophagia after sore throat
and had difficulty in lying on bed. How will you confirm the diagnosis? (IMDC)
 X-ray soft tissue neck lateral view*
 Nasopharyngeal endoscopy
 CT scan
 MRI
 Culture sensitivity with throat swab
117. A 25 year’ old female, talkative in nature, developed permanent hoarseness of voice for
last 3 months. On IDL there was nodular swelling on vocal cords at junction of anterior 1/3 rd
post 2/3rd. Treatment is (IMDC)
 Total laryngectomy
 Steam inhalation and antibiotics
 Steam inhalation and speech therapy
 Speech therapy and use of laser*
 Speech therapy and micro laryngeal surgery
118. 2nd year MBBS student examining larynx specimen in anatomy lab, he found only one
complete ring. (IMDC)
 Which one is that?
 Thyroid cartilage
 Cricoid cartilage*
 Arytenoid
 Epiglottis
 Corniculate
119. A 25 years old woman with a turbulent married life is brought to emergency with 3 hour
history of inability to speak. She is otherwise conscious. IDL shows normal-appearing vocal
cords that fail to adduct on attempted phonation. Her cough is normal. Choose the most
likely pathology from the following (IMDC)
 Bilateral abductor palsy
 Vocal cords nodules
 Myasthenia gravis
 Functional aphonia*
 Fixation of cricoarytenoid joint
120. Post thyroidectomy patient presents with hoarseness of voice and severe respiratory
distress, on IDL vocal cords in paramedian position. Which important muscle is for
abduction is paralyzed? (IMDC)
 External Cricothyroid
 Posterior crico arytenoid*
 Vocalis
 Inter arytenoid
 Thyro arytenoids
121. A 40 year’ old known smoker developed Permanent hoarseness of voice, growth seen on
the right vocal cord. Diagnosis is (IMDC)
 Vocal cord nodule
 Vocal cord polyp
 CA larynx*
 Papilloma larynx
 Leukoplakia
122. A 50-year-old furniture maker presents with proptosis, swelling over frontoethmoidal
region and repeated left sided episodes of epistaxis for 6 months. What is diagnosis?
(IMDC)
 Maxillary sinus carcinoma *
 Mucormycosis
 Bleeding polypus
 Adenoid cystic carcinoma of ethmoid
 Maxillary sinusitis
123. A 40 year’ female presents in ENT OPD with severe pain in throat (Right side), On
examination there were red swollen tonsils with yellow spots. Patient also had pain in
right ear. Blood examination revealed the 18000/cm, Neutrophils are 84% What is your
diagnosis? (IMDC)
 Acute tonsillitis *
 Acute Retropharyngeal abscess
 Peritonsillar abscess
 Diphtheria
 Ludwig’s angina
124. A 14 year’ boy with longstanding discharge from the ear presents with earache, fever,
anorexia, nausea and rigors. He has tenderness and swelling over the left mastoid area.
 Mastoid abscess*
 Septicemia
 Brain abscess
 Meningitis
 Labyrinthitis
125. Keratosis obturan’ is (IMDC)
 Fungal infection
 Accumulation of wax over a foreign body
 Accumulation of soft cerumen
 Faulty migration of squamous epithelial cells *
 Hard wax
126. A 7 years child who underwent tonsillectomy 5 days ago has reported in Emergency room
with history of high-grade fever and bleeding from the throat. The best option regarding
the treatment is
 Cautery of bleeding point
 Control of bleeding with tight gauze packing
 Ligation of the bleeding vessel
 Admission + antibiotics
 Admission + antibiotics + blood transfusion if required *
127. Where are the palatine tonsils located
 Oral cavity
 Nasopharynx
 Hypopharynx
 Oropharynx*
 Base of tongue
128. Regarding Ranula which is the most appropriate answer
 It is a salivery gland cyst
 Needs FNA for diagnosis
 Most commonly arises from sublingual gland*
 Should not be excised surgically
 Arises in posterior part of floor of mouth
129. A 22 Yr old girl presented with a progressively increasing swelling in the floor of mouth.
O/E there was smooth, rounded, cystic bluish mass on Lt side of floor of mouth What is
most likely diagnosis
 Sebaceous cyst
 Squamous cell carcinoma
 Ranula*
 Dermoid cyst
 Periodotal cyst
130. Regarding oral cavity respond to the most appropriate one
 Oral thrush is a viral infection
 Parotid duct opens into the frenulum
 Retention cyst should be excised surgically*
 Antibiotics should be given for oral thrush
 Common cause of Ludwig’s angina is acute tonsillitis
131. Following is the part of oral cavity (RIHS)
 Base of tongue
 Lingual tonsils
 Vellecula
 Soft palate
 Retromolar trigone*
132. Oral cavity extends
 From lips to posterior of soft palate
 b. From base of skull to hard palate
 c. From lips to posterior border of hard palate*
 d. From hard palate to hyoid bone
 e. From lips to posterior faucial pillars
133. A patient was planned for tonsillectomy to approach a cranial nerve in the tonsillar bed.
Which of the following nerve can be approached through this operation?
 Hypoglossal nerve
 Vagus nerve
 Facial nerve
 Lingual nerve
 Glossopharyngeal nerve*
134. In which of the following conditions “Hot potato” voice is typically seen
 Vocal nodules
 Cleft palate
 Peritonsillar abscess*
 Nasal polyps
 Growth larynx
135. Following is Not the part of Waldeyer’s Ring
 Adenoids
 Palatine tonsils
 Tubal tonsils
 Jugulodigastric lymph nodes*
 Lateral pharyngeal bands
136. Anterior faucial pillar is formed by
 Palatoglossus muscle*
 Palatopharyngeus muscle
 Salpingopharyngeus muscle
 Superior constrictor muscle
 Glossopharyngeus muscle
137. Posterior faucial pillar is formed by (RIHS)
 Palatoglossus muscle
 Palatopharyngeal muscle*
 Salpingopharyngeus muscle
 Superior constrictor muscle
 Glossopharyngeal muscle
138. Regarding palatine tonsils respond to most appropriate one
 Located in nasopharynx
 Have no capsule
 Crypta megna is located near upper pole*
 Have no lymphoid follicles
 One in number
139. A 7 Yr child was planned for tonsillectomy. In which of the following plane dissection
should be done
 Within the tonsillar tissue
 Within superior constrictor muscle
 Within loose areolar tissue between capsule & muscle*
 Within capsule of the tonsil
 Lateral to the superior constrictor muscle
140. The tonsilar bed is formed by
 Palatoglossus muscle.
 Palatopharyngeus muscle
 Tensor palate muscle
 Styloglossus muscle.
 Superior constrictor muscle*
141. Oropharynx extends
 From base of skull to hard palate
 From base of skull to soft palate
 From base of skull to hyoid bone
 From base of skull to lower border of cricoid
 From hard palate to hyoid bone*
142. Typical location of Adenoids is
 Lateral & posterior wall of oropharynx
 Lateral wall of nasopharynx
 Posterior & lateral wall of nasopharynx
 Roof & posterior wall of nasopharynx*
 Lateral wall of hypopharynx
143. Most common presenting symptom of enlarged adenoids is
 Hyposmia
 Earache
 Mouth breathing*
 Hearing loss
 Prominent upper incisors
144. Inferior constrictor muscle of the pharynx takes origin from
 Hyoid bone
 Mandible
 Maxilla
 Thyroid and cricoid cartilage*
 Arytenoid Cartilage
145. Regarding Quinsy what is the most appropriate
 Pus collects outside superior constrictor muscles.
 Trismus is its characteristic feature*
 There is swelling in the neck
 Incision is made in the middle of the tonsil for incision/drainage.
 It is bilateral
146. A 7 Yr boy was diagnosed as a case of Acute tonsillitis Which bacterial organisms are most
common cause of this condition
 H influenza
 β hemolytic streptococcus*
 Morexella catarrhalis
 Staph aureus
 Pseudomonas.
147. In Acute Tonsillitis, there is
 Low grade gradual onset fever
 Membrane covering the tonsil and extending to soft palate
 Leucopoenia
 Indication for Tonsillectomy if two attacks in a year
 Odynophagia*
148. A 20 Yr male patient presented with severe left sided pain throat, fever, odynophagia,
trismus & dribbling of saliva for 3 days. What is the most likely diagnosis
 Acute Epiglottitis
 Acute tonsillitis
 Peritonsillar abscess*
 Acute retropharyngeal abscess
 Acute pharyngitis
149. Peritonsillar abscess is differentiated from acute tonsillitis by:
 Pain Throat
 Trismus*
 Leukocytosis
 High grade fever
 Odynophagia
150. A 14 Yr girl reported in ENT OPD with H/O severe pain throat, high grade fever & chills O/E
both tonsils were congested & covered with pus spots What is the most likely diagnosis
 Ac Catarrhal tonsillitis
 Ac Follicular tonsillitis*
 Ac Membranous tonsillitis
 Pharyngeal Diphtheria
 Agranulocytosis
151. Tonsillectomy is a very common but nonetheless controversial operation. Which of the
following are accepted criteria which would justify tonsillectomy?
 Peritonsillar abscess*
 Crypts over the tonsils
 Bilateral non-obstructing hypertrophy
 Cleft palate
 Recurrent tonsillitis 3 episodes in last one and a half year
152. Regarding tonsillectomy what is the most appropriate
 Is almost completely painless with modern technique.
 Is indicated in children with two episodes of acute tonsillitis
 Can be complicated by fatal bleeding*
 Is helpful in the management of poor general health e.g. Low weight/height
 Should now be reserved for children with immunodeficiency
153. The most common cause for pharyngeal and oral ulceration is
 Behcet disease.
 Aphthous ulcers*
 Tuberculous ulcers.
 Syphilitic ulcer
 Carcinomatous Ulcer
154. The earliest and commonest complication of diphtheria is
 Cardiac failure
 Palatal paralysis.
 Laryngeal obstruction*
 Acute nephritis
 Diplopia
155. In diphtheria, the antitoxic serum is given
 To neutralize the circulating titoxin*
 To neutralize the fixed antitoxin
 To kill the diphtheria bacilli.
 To prevent further attack
 To prevent Laryngeal Obstruction
156. The site of incision for drainage of Quinsy is :
 On Posterior pharyngeal wall
 At inferior pole of tonsil..
 Through the crypta magna of the tonsil.
 Along the horizontal line drawn at base of Uvula and Vertical line along anterior pillar*
 Along the horizontal line drawn at base of Uvula and Vertical line along posterior pillar
157. A 20 year-old man with poor dentition is found to have an acute onset of a swollen,
tender, boardlike mass in the submental and submandibular spaces. His anterior floor of
mouth rises above the level of his mandibular incisors. His airway is tenuous. What is most
likely diagnosis?
 Vincent's angina
 Ludwig's angina*
 Prinzmetal's angina
 Parapharygeal space abscess
 Angina Pectoris
158. Chronic Retropharyngeal Abscess
 Is secondary to peritonsillar abscess.
 Penicilline is the drug of choice
 Is disease of children.
 Causes dysphonia
 Is associated with caries spine*
159. A 60 Yr male patient presented with a malignant tumour of the oral cavity What is the
most likely histology of the tumour at this site
 Adenoid cystic carcinoma
 Lymphoma
 Squamous cell carcinoma*
 Mucoepidermoid carcinoma
 Sarcoma
160. Most important contraindication to tonsillectomy is
 H/O peritonsillar abscess
 Hemoglobin 11 Gm%
 Acute upper respiratory infection*
 Age less than 6 yrs
 Fibrotic tonsils
161. Which of the following condition predominantly occurs in children below three years of
age?
 Acute retropharyngeal abscess*
 Chronic retropharyngeal abscess
 Parapharyngeal abscess
 Peritonsillar Abscess
 Apical tooth abscess
162. The nasopharynx lies below the
 Base of skull*
 Nose
 Hard palate
 Hyoid bone
 Hypopharynx
163. Following laryngeal cartilage never calcifies
 Thyroid cartilage
 Epiglottis*
 Cricoid cartilage
 Cuneiform cartilage
 Arytenoid cartilage
164. The only intrinsic laryngeal muscle innervated by the superior laryngeal nerve is the:
 Cricothyroid*
 Thyroarytenoid
 Lateral cricoarytenoid
 Posterior cricoarytenoid
 Interarytenoid
165. Following intrinsic muscle of larynx is Not supplied by recurrent laryngeal nerve
 Cricothyroid*
 Thyroarytenoid
 Lateral cricoarytenoid
 Posterior cricoarytenoid
 Interarytenoid
166. Normal colour of the true vocal cords is
 Pink
 Velvety
 White
 Pearly white*
 Red
167. Sensory supply of larynx above the vocal cords is
 External branch of superior laryngeal nerve
 External branch of recurrent laryngeal nerve
 Internal branch of superior laryngeal nerve*
 Vagus nerve
 Hypoglossal nerve
168. Following muscle is the only abductor of the larynx
 Posterior cricoarytnoid*
 Lateral cricoarytnoid
 Thyroarytnoid
 Cricothyroid
 Interarytnoids
169. Most common site for impaction of a foreign body in airway is
 Trachea
 Larynx
 Right main bronchus*
 Left main bronchus
 Carina
170. Following is quite significant sign of foreign body airway
 Pooling of saliva in pyriform fossae
 Wheeze
 Laryngeal crepitus
 Decreased air entry*
 Increased air entry
171. Some foreign body got impacted in Rt main bronchus of a 6 Yr old boy Which of the
following foreign body is most harmful for the patient
 Plastic whistle
 Metallic screw
 Peanut*
 Piece of spongston
 Plastic bead
172. Symptoms of larynx Do Not include.
 Difficulty in breathing
 Difficulty in swallowing*
 Stridor
 Wheezing
 Hoarseness
173. Regarding larynx which of the following is most appropriate
 All Laryngeal diseases typically presents with hoarseness.
 Voice is produced by vibration of the Aryepiglottic fold
 The vocal cords abduct from the midline to prevent aspiration of fluids into the lungs.
 Laryngeal obstruction is quickly fatal unless an alternative airway is established.*
 Indirect laryngoscopy requires the use of a rigid endoscope.
174. The following laryngeal cartilage has complete ring
 Corniculate
 Cunieform
 Thyroid
 Epiglottis
 Cricoid*
175. Adam’s apple is formed by
 Corniculate Cartilage
 Cricoid Cartilage
 Cunieform Cartilage
 Epiglottis Cartilage
 Thyroid Cartilage*
176. Both Vocal cords are near midline during
 Expiration*
 Speaking
 Sleeping
 Running
 At rest
177. After a road traffic accident, a 20 Yr boy sustained severe injury on his neck. While being
shifted to hospital he died What is the most common cause of his death
 Spinal cord injury
 Respiratory obstruction*
 Injury to the vagus nerve
 Heamorrhage
 Injury to carotid vessels
178. A professional wrestler comes to the ER after receiving a kick on the larynx at the hands of
another wrestler. The patient is experiencing progressively worsening stridor and air-
hunger. He has subcutaneous emphysema from his mandible to his nipples. He coughs up
some blood and begins to look worse. What is your next step in management?
 Depends on the result of blood gas measurements
 Barium swallow
 CT scan of the larynx
 X-Ray Chest
 Tracheostomy*
179. Initial Signs of Laryngeal injury in mass trauma are
 Obvious*
 Rare to find
 Most common
 Subtle
 Not important.
180. Most important sign in laryngeal injury diagnosis
 Gross bleeding
 Massive Trauma
 Pain Neck
 Discoloration of Skin Neck
 Surgical Emphysema*
181. Common Cause of death on the spot in RTA is
 Cervical Spine Injury
 Foreign Body Tracheobronchial Tree.
 Vascular Injury Neck
 Epistaxis
 Laryngeal Injury*
182. Monitoring in Laryngeal Injury includes
 Serial laryngoscopy
 Repeated CT Scan
 Close observation of increase in Emphysema
 Any Increase in Pain Neck
 Signs of dyspnea (Pulse Rate / Stridor / Dyspnoea / Blood de oxygenation)*
183. Vocal nodules are formed at Extended Matching
 Middle of true vocal cords
 Junction of anterior 1/3 & posterior 2/3 of false vocal cords
 Junction of anterior 1/3 & posterior 2/3 of true vocal cords*
 Junction of anterior 2/3 & posterior 1/3 of true vocal cords
 Junction of anterior 2/3 & posterior 1/3 of false vocal cords
184. In a 20 years old female, Voice is good with dyspnoae, in.
 Bilateral vocal cord paralysis with vocal cords near the midline*
 Bilateral vocal cord paralysis with vocal cord away from the midline in cadaveric position
 Laryngomalacia
 Unilateral vocal cord paralysis
 Carcinoma Hypopharynx
185. Voice is hoarse in the beginning but recovers with passage of time.
 Bilateral vocal cord paralysis with vocal cords near the midline
 Bilateral vocal cord paralysis with vocal cord away from the midline in cadaveric position
 Laryngomalacia
 Unilateral vocal cord paralysis*
 Carcinoma Hypopharynx
186. Stridor but tracheostomy has no role to play in
 Bilateral vocal cord paralysis with vocal cords near the midline
 Bilateral vocal cord paralysis with vocal cord away from the midline in cadaveric position
 Laryngomalacia*
 Unilateral vocal cord paralysis
 Carcinoma Hypopharynx
187. There is hoarseness of voice in
 Bilateral recurrent laryngeal nerve paralysis
 Papiloma Epiglottis
 Carcinoma Pharynx
 Carcinoma Glottis*
 Carcinoma false vocal cords
188. In one of the following Voice may be good without dyspnoe.
 Bilateral vocal cord paralysis with vocal cords near the midline. (Bil. Recurrent Nerve
Paralysis)
 Bilateral vocal cord paralysis with vocal cord away from the midline in cadaveric position
(Bil. Combined Paralysis of laryngeal nerves)
 Laryngomalacia
 Left unilateral vocal cord paralysis*
 Reinke’s oedema
189. Voice is good with dyspnoae in
 Bilateral vocal cord paralysis with vocal cords near the midline. ( Bil. Recurrent Nerve
Paralysis)*
 Bilateral vocal cord paralysis with vocal cord away from the midline in cadaveric position
(Bil Combined Paralysis of laryngeal nerves)
 Left unilateral vocal cord paralysis
 Reinke’s oedema
 Superior Laryngeal nerve paralysis.
190. Voice abuse has no role to play in;
 Reinkes Oedema
 Vocal cord Polyp
 Papilloma larynx*
 Singer’s node.
 Laryngitis
191. Hoarseness occurs in
 Tumour of epiglottis
 Swelling of vestibular fold
 Haemangioma of subglottis
 Vocal nodules*
 Bronchitis
192. Voice is hoarse in the beginning but recovers with passage of time
 Bilateral vocal cord paralysis with vocal cords near the midline. ( Bil. Recurrent Nerve
Paralysis)
 Bilateral vocal cord paralysis with vocal cord away from the midline in cadaveric position
(Bil Combined Paralysis of laryngeal nerves)
 Laryngomalacia
 Left unilateral vocal cord paralysis*
 Vocal Cord Polyp
193. Unilateral vocal cord paralysis causes
 Aspiration
 Stridor
 Dyspnoea
 Dysphagia
 Hoarseness*
194. A 50 years old singer who smokes 30 cigarette a day has hoarseness for last 20 days.
Which one of the following will you rule out first?
 Singer’s node
 CA Larynx*
 Vocal cord Papilloma
 Reinke’s Oedema
 Laryngitis
195. Regarding carcinoma Larynx respond to most appropriate
 Is commoner in women
 May present with airway obstruction*
 Histopathology is usually adenocarcinoma
 Is cured by chemotherapy
 Prognosis is poor for glottis carcinoma
196. The commonest malignant tumour of ENT is
 Squamous Cell carcinoma*
 Basal Cell Carcinoma
 Adenoid cystic Carcinoma
 Adenocarcinoma
 Sarcoma
197. 211-212-213 Select the three characteristics of T2 N0 M1 Squamous Cell Carcinoma Larynx
 Medium size tumour
 Curative treatment is available for the tumour.
 Symptoms of malignancy are present
 Metastases to lymph nodes are present
 Tumour extend to two site of larynx*
 Hoarseness is present
 Systemic metastases are present
 Patient is weak cachectic emaciated
 Tumor has spread locally beyond the Larynx*
198. The presentation for Squamous cell carcinoma Larynx includes.
 Non healing ulcer*
 Laryngeal spasm
 Stridor
 Dysphagia
 Wheezing
199. Pre-malignant condition in head and Neck includes.
 Chronic Tonsillitis
 Papilloma oropharynx
 Atrophic Rhinitis
 Plummer Vinson Syndrome*
 Oroantral fistula
200. A patient with Foreign body trachea will have
 Inspiratory stridor
 Expiratory stridor*
 Biphasic stridor
 Hoarseness of voice
 Dysphagia
201. A 3 Yr boy was brought with H/O swallowing a 5-rupee coin two hours back He c/o
difficulty in swallowing What is the most likely site of impaction of the coin (RIHS)
 Larynx
 Lower (cardiac) end of oesophagus
 Rt main bronchus
 Upper end (cricopharyngeus) of oesophagus*
 Vellecula
202. The most common cause of a neck mass in males over 60 years of age is:
 Metastatic carcinoma*
 Brachial cleft cyst
 Laryngocoele
 Bacterial lymphadenitis
 Carotid aneurysm
203. You have been promoted to Otolaryngology resident. Your call night begins with a page to
the pediatric intensive care unit. They have a 2-month-old with stridor. The PICU staff
suspect laryngomalacia. If so, laryngoscopy would reveal:
 Vocal cord paralysis
 No visible lesion as laryngomalacia is a neuromuscular disorder
 Aryepiglottic fold flaccidity and an "omega" shaped epiglottis*
 Pinhole airway at the subglottis
 A thin, perforate epiglottis
204. In trying to determine which patients will have bleeding problems associated with
tonsillectomy, the most complete and cost-effective method is:
 CBC, platelet count, and bleeding time*
 Platelet count
 Bleeding time
 Prothrombin time and partial thromboplastin time
 Careful history
205. You escort your patient to the recovery room after total thyroidectomy. Removal of the
endotracheal tube is followed by severe airway obstruction and stridor. The drain is
functional, and the wound looks fine. The most likely diagnosis is:
 Subglottic edema*
 Subglottic spread of cancer
 Laryngospasm
 Arytenoid dislocation
 Bilateral vocal cord paralysis
206. A 3-year-old female recovering from a URI develops mild airway compromise, fever, and a
muffled cry. Her exam is notable for a stiff neck and a bulging mass just off the midline of
the posterior pharynx visible through the mouth. You treat her with:
 Antibiotics
 Ice water pharyngeal irrigation and antibiotics
 "Hot" tonsillectomy
 Decompressive laminectomy for spiral osteomyelitis
 Drainage of her retropharyngeal abscess *
207. A boisterous (Loud, noisy, and lacking in discipline) 5 year-old male with a 3 month history
of hoarseness is found to have bilateral tiny swellings at the junction of the anterior and
middle third of the vocal cords bilaterally. Both his parents are smokers. He has:
 Granulomatous laryngitis
 Vocal nodules*
 Vocal polyps
 Respiratory papillomas
 Chondosarcoma of the larynx
208. There is hoarseness of voice in
 Bilateral recurrent laryngeal nerve paralysis
 Papiloma epiglottis
 Carcinoma pharynx
 Carcinoma glottis *
 Tracheostomy
209. Examination of Larynx by palpation includes. (IMDC)
 Indirect Laryngoscopy.
 Fibre-optic Laryngoscopy.
 Laryngeal Crepitus *
 Direct Laryngoscopy
 Examination by tongue depressor.
210. In Injuries of Larynx priority is
 Laryngeal repair of mucosa
 Reduction of fractured cartilages
 Tracheostomy*
 Correction of hoarseness of voice
 prevention of hoarseness of voice
211. An elective Tracheostomy was planned in a 40 Yr male before surgery on oral cavity. What
is the most ideal site to open the trachea?
 Immediately below the cricoid cartilage
 Between 1st & 2nd tracheal rings
 Between thyroid & cricoid cartilage
 Between 3rd & 4th tracheal rings*
 Any visible tracheal ring
212. After tracheostomy a 23 Yr female developed perichondritis of cricoid cartilage What
could be the most common reason for this complication
 Antibiotics had not been prescribed
 Regular suction was not done
 High tracheostomy was performed*
 Patient was weak & anaemic
 Trachostomy tube was made of irritant material
213. A young male met a road traffic accident & developed respiratory distress & cyanosis. A
doctor in BHU immediately performed Mini tracheostomy. In which part incision is made
for this procedure
 Cricoid cartilage
 Thyroid cartilage
 Cricothyroid membrane*
 Thyrohyoid membrane
 Cricotracheal membrane
214. Recurrent laryngeal nerve is motor to all of the intrinsic muscles of the larynx except:
 Vocalis muscle
 Posterior cricoarytenoid muscle
 Lateral cricoarytenoid muscle
 Cricothyroid muscle*
215. The only abductor muscle in the larynx is:
 Sternothyroid muscle
 Lateral cricoarynoid muscle
 Cricothyroid muscle
 Posterior cricoarynoid muscle*
216. Secondary post-tonsillectomy bleeding is due to:
 Incomplete removal
 Foreign body aspiration
 Sepsis*
 Posterior nasal packing
217. Pharyngeal diverticulum occurs through:
 The superior constrictor muscle
 The middle constrictor muscle
 The inferior constrictor muscle*
 Palatopharyngus muscle
218. Unilateral vocal cords paralysis may occur due to:
 Viral infection
 Thyroidectomy
 Mediastinal lesion
 All of the above*
219. Cortical mastoidectomy is indicated in:
 Acute mastoiditis not responding to medical treatment*
 Bezold’s abscess
 Mastoid abscess
 All of the above
220. Stapedectomy is the operation of the choice in:
 Atelactatic middle ear
 Meniere's disease
 Otosclerosis*
 Secretory otitis media
221. Acute pulmonary edema occurring on opening of the trachea during tracheostomy is due
to:
 Sudden drop of pressure in the tracheobronchial tree and lungs*
 Sudden wash of carbon dioxide
 Injury of the esophagus
 None of the above
222. The commonest cause of esophageal stenosis in children is:
 Corrosive intake*
 Carcinoma
 Achalasia of the cardia
 Diphtheria
223. The inferior constrictor muscle of the pharynx takes origin from:
 Hyoid bone
 Mandible
 Maxilla
 Thyroid and cricoid cartilages*
224. Apnea immediately after tracheostomy is treated by:
 Multiple skin incisions
 Inhalation of oxygen *
 Temporary closure of the tracheostomy opening
 Intubation
225. Multiple papillomata of the larynx is characterized by the following except:
 Occurs in children
 Affect only the glottic area *
 Recurrence is common
 Best treated by laser
226. The main arterial supply of the tonsil is derived from:
 Sphenopalatine arterytoward
 Maxillary artery
 Facial artery *
 Internal carotid artery
227. The constrictor muscles of the pharynx are responsible for:
 Deglutition*
 Aspiration
 Glottic closure
 Reflux disease
228. Tonsillectomy is contraindicated in:
 Cervical lymphadenopathy
 Meniere’s disease
 Enlarged adenoid
 Blood diseases*
229. Enlarged adenoids causes all the following:
 Adenoid facies*
 Sleep apnea
 Secretory otitis media
 Plummer Vinson syndrome
230. Early glottic carcinoma is best treated by:
 Radiotherapy*
 Total laryngectomy
 Antibiotics
 Cryosurgery
231. On tracheostomy, apnea developed after incising the trachea is due to:
 Sudden wash of CO2 in the trachea*
 Sudden release of the pressure in the trachea
 Reflex vagal stimulation
 Non of the above
232. The most common cause of esophageal stricture in children is:
 Esophageal carcinoma
 Diphtheria
 Achalasia of the cardia
 Corrosive intake*
233. Stridor means:
 Difficult expiration
 Difficult inspiration
 Both*
 Dyspnea
234. Tonsillectomy is absolutely contraindicated in:
 Menstruation
 Upper respiratory tract infection*
 Quinsy
 Bleeding disorders
235. The following are signs of chronic tonsillitis except:
 Enlarged cervical lymph nodes
 Inequality of the size of the tonsils
 Pus in the tonsillar crypts
 Edema of the uvula *
236. In multiple laryngeal papillomata, all of the following are true Except:
 Occurs in children
 Affects only the glottic area*
 Recurrence is common
 Best treated by laser surgery
237. T.B of the larynx mostly affects:
 Posterior part of the larynx*
 Supraglottis
 Anterior part of the larynx
 Subglottis
238. Reactionary hemorrhage after tonsillectomy is caused by:
 Incomplete removal
 Slipped ligature *
 Extubation spasm
 Nasal obstruction
239. The most serious complication following adenotonsillectomy is:
 Reactionary hemorrhage*
 Respiratory obstruction
 Respiratory infection
 Non of the above
240. All the following are signs of chronic tonsillitis except:
 Inequality of size
 Congested anterior pillars
 Presence of crypts over the medial tonsillar surface*
 Enlarged cervical lymph nodes
241. Laryngoscleroma is characterized by:
 Subglottic stenosis *
 Vocal cord paralysis
 Mouth ulceration
 Cervical lymph node
242. Stridor in children can be caused by:
 Chronic tonsillitis
 Acute rhinitis
 Deviated nasal septum
 Inhaled foreign body*
243. Tonsillar artery is a branch from
 Lingual artery
 Maxillary artery
 Facial artery*
 Occipital artery
244. Inferior constrictor muscle of the pharynx takes origin from:
 Hyoid bone
 Mandible
 Maxilla
 Thyroid and cricoid cartilages*
245. The pharyngeal pouch passes through:
 Superior constrictor muscle
 Killian dehiscence*
 Middle constrictor muscle
 Hyoid bone
246. The usual cause of reactionary post-tonsillectomy bleeding is:
 Slipped ligature *
 Wound infection
 Tonsillar remnant
 Early extubation
247. Which of the following arteries does not contribute to Keisselbach’s plexus? (IMDC)
 Greater palatine artery
 Ascending palatine artery*
 Sphenopalatine artery
 Anterior ethmoidal artery
 Superior labial artery
248. Which of the following pathogen is most commonly involved in acute epiglottitis? (IMDC)
 Streptococcus pneumoniae
 Pseudomona Aeroginosa
 Haemophillus Influenzae*
 Parainfluenza virus A and B
 Staphylococcus aureus
249. Which of the following is not an option for maintaining airway in case of acute epiglottitis?
(IMDC)
 Propped up position
 Endotracheal intubation
 Tracheostomy
 Oropharyngeal airway*
 Intravenous corticosteroids
250. A child suffering from fever and sore throat developed hoarseness, cough, inspiratory
stridor and dyspnea. Examination revealed an adherent grayish white membrane over
pharynx and soft palate. What is the most likely diagnosis? (IMDC)
 Laryngeal diphtheria*
 Infectious mononucleosis
 Acute membranous tonsillitis
 Laryngomalacia
 Acute laryngo-tracheo-bronchitis
 Old age
251. Which of the following conditions is not an indication for tracheostomy? (IMDC)
 Airway obstruction
 Retained secretion
 Dead space reduction
 Foreign body esophagus*
 Foreign body trachea
252. A patient with prolonged airway obstruction underwent tracheostomy under local
anaesthesia. Just after insertion of tracheostomy tube he developed apnea. Which of the
following explanation is correct about this condition? (IMDC)
 Compression of trachea
 Tracheo-esophageal fistula
 Sudden washing out of carbon dioxide*
 Vaso-vagal shock
 Hypovalemia due to hemorrhage
253. Which of the following arteries do not supply palatine tonsils? (IMDC)
 Tonsillar branch of facial artery
 Ascending pharyngeal artery
 Descending palatine artery
 Sphenopalatine artery*
 Dorsal lingual artery
254. Which of the following bacteria is most commonly involved in acute tonsillitis? (IMDC)
 Haemolytic streptococcus*
 Pseudomonas aeroginosa
 Proteus mirabilis
 Staphylococcus aureus
 Corynebacteria diphtheria
255. What is the mechanism of myocardial and neurological complication of faucial diphtheria?
(IMDC)
 Exotoxin*
 Endotoxin
 Bacterial endocarditis
 Rheumatic carditis
 Autoimmune carditis
256. What is the space between the tonsil capsule and tonsillar bed called? (IMDC)
 Parapharyngeal space
 Retropharyngeal space
 Submandibular space
 Peritonsillar space*
 Submental space
257. Another name for a peritonsillar space abscess is: (IMDC)
 Retrotonsillar abscess
 Post-tonsillar abscess
 Quinsy*
 Ludwig’s angina
 Vincent’s angina
258. Which cranial nerve supply sensory fibers to the palatine tonsils? (IMDC)
 Vagus nerve (CN-X)
 Glossopharyngear nerve (CN-IX)*
 Maxillary division of trigeminal nerve (CN-V2)
 Hypoglossal nerve (CN-XII)
 Facial nerve (CN-VII)
259. Which of the following is the absolute indication for tonsillectomy? (IMDC)
 Chronic tonsillitis
 Acute tonsillitis
 Suspicion of malignancy*
 Diphtheria
 Glossopharyngeal neurectomy
260. A post-tonsillectomy patient complains of pain in the ear after surgery. What is the most
appropriate explanation? (IMDC)
 Tympanic membrane perforation
 Trauma to the temporomandibular joint
 Acute otitis media
 Referred pain from glossopharyngeal nerve*
 Trauma to maxillary teeth
261. The organisms involved in Vincent’s angina are a fusiform bacillus and a spyrochaete
named: (IMDC)
 Borrelia Bergdorfery
 Borrelia Vincentii*
 Chlamydia Psitasii
 Mycoplasma pneumonia
 Chlamydia vincentii
262. What is the etiology or oral thrush? (IMDC)
 Herpes zoster virus
 Herpes simplex virus
 Fusiform bacillus
 Candida albican fungus*
 Staphyllococcus aureus
263. A child suffers from sore throat with odynophagia, fever and malaise. On examination
there are clusters of vesicles and ulcers on the hard palate. What is the most likely
diagnosis? (IMDC)
 Herpes Simplex infection*
 Herpes zoster infection
 Oral candidiasis
 Aphthous ulcers
 Pemphigus vulgaris
264. What is ranula? (IMDC)
 Benign neoplasm of sublingual salivary gland
 Malignant neoplasm of sublingual salivary gland
 Calculus od submandibular salivary gland
 Retention cyst of sublingual salivary gland*
 Dermoid cyst of floor of the mouth
265. A 35 years old trumpet blower complains of hoarseness, cough and occasional respiratory
obstruction. On examination there is a reducible swelling on the left side of the neck at the
level of thyroid cartilage which increases in size with cough and Valsalva. What is the most
likely diagnosis? (IMDC)
 Saccular cyst
 Laryngocele*
 Metastatic lymph node
 Solitary thyroid nodule
 Cricothyroid bursitis
266. Which of the following is the commonest site of involvement in squamous cell carcinoma
of the larynx? (IMDC)
 Epiglottis
 Supraglottis
 Glottis*
 Subglottis
 Aryepiglottic folds
267. Which of the following factor is not incriminated in the causation of laryngeal carcinoma?
(IMDC)
 Cigarette smoking
 Alcohol
 Asbestos
 Radiations exposure
 Professional voice users*
268. A malignant tumor with vocal cord fixation is staged as: (IMDC)
 Stage T1
 Stage T2
 Stage T3*
 Stage N2
 Stage T4
269. Lymphatic metastasis of a squamous cell carcinoma of larynx primarily involves which
group of lymph nodes? (IMDC)
 Pretracheal and paratracheal nodes*
 Submental and Submandibular nodes
 Mediastinal group of lymph nodes
 Posterior triangle lympoh nodes
 Parotid and preauricular nodes
270. Which of the following carcinoma of larynx does not metastasize? (IMDC)
 Squamous cell carcinoma
 Verrucus carcinoma*
 Transitional cells carcinoma
 Large cells tumor
 Small cells tumor
271. Which of the following glands is predominantly involved in the formation of salivary
stones? (IMDC)
 Parotid glands
 Submandibular glands*
 Minor salivary glands
 Weber glands
 Palatine glands
272. What is the most common etiology of acute supporative parotitis? (IMDC)
 Paramyxovirus (Mumps virus)
 Staphylococcus aureus*
 Mycobacterium tuberculosis
 Actinomyccosis
 Aspergillus nigra
273. Which of the following is the most common benign tumor of salivary glands? (IMDC)
 Warthin tumor
 Oncocytoma
 Pleomorphic adenoma*
 Cystic hygroma
 Haemangioma
274. A patient undergoing parotidectomy for a pleomorphic adenoma complained of severe
distress due to sweating and flushing of the preauricular skin while eating food. What is the
diagnosis? (IMDC)
 Crocodile tears
 Frey’s syndrome*
 Drugs induced sweating
 Hyperthyroidism
 Facial nerve impairment
275. A child was choked by a large foreign body stuck in the throat, was unable to cry and
asphyxiating. What immediate measure you need to take? (IMDC)
 Pass an Endotracheal tube.
 Perform emergency tracheostomy
 Perform Heimlich maneuver*
 Perform cricothyrotomy
 Perform Laryngo-tracheo-bronchoscopy
276. What is the surgical treatment of a squamous cell carcinoma of palatine tonsil? (IMDC)
 Tonsillectomy
 Tonsillectomy with excision of uvula and anterior pillar
 Tonsillectomy with excision of base of tongue
 Commando operation*
 Laser evaporation
277. What is the indication of Dohlman’s procedure? (IMDC)
 Pharyngeal pouch*
 Post-cricoid tumor
 Tumor of pyriform sinus
 Laryngocele
 Branchial fistula
278. Which of the following conditions is an important etiological factor in post-cricoid
carcinoma? (IMDC)
 Corrosive burns
 Chronic gastro-esophageal reflux
 Iron deficiency anemia*
 Chronic obstructive pulmonary disease
 Cystic fibrosis
279. Subepithelial space of the membranous vocal cords is called? (IMDC)
 Rima Glottis
 Glottic space
 Paraglolttic space
 Reinke’s space*
 Pre-tracheal space
280. Which age group is most commonly effected by Acute layngo-tracheo-bronchitis? (IMDC)
 6 month to 3 years*
 3 years to 5 years
 5 to 10 years
 10 years and above
281. Greisinger’s sign means:
 Pain over the tempromandibular joint
 Pain in the eye
 Pain and tenderness over the mastoid*
 Pain and tenderness over the auricle
282. An adult presented with acute onset dysphagia, edema of the uvula and palate, medially
pushed tonsil and trismus is most likely to have:
 Parapharyngeal abscess
 Acute retropharyngeal
 Quinsy*
 Chronic retropharyngeal abscess
283. Absent laryngeal click may suspect:
 Supraglottic carcinoma*
 Pyriform fossa tumor
 Subglottic cancer
 Post-cricoid carcinoma
284. Quinsy is suppuration in:
 Parapharyngeal space
 Peritonsillar space*
 Retropharyngeal space
 Submandibular space
285. Griesinger’s sign is positive in:
 Acute petrositis
 Acute sinusitis
 Acute labyrinthitis
 Lateral sinus thrombophlebitis*
286. Kernig’s sign is present in:
 Otitic meningitis*
 Otitic brain abscess
 Otitic hydrocephalus
 All of the above
287. What is the origin of antrochoanal polyp? (YMDC)
 Ethmoidal sinus
 Uvulal sinus
 Maxillary sinus*
 Sphenoid sinus
 All of the above
288. What is the most common site for origin of nasopharyngeal angiofibroma? (YMDC)
 foramen ovate
 posterior wall of the nasopharynx
 roof of the nasopharynx
 sphenopalatine foramen*
 pterygopalatine fossa
289. In which of the following demographic groups nasopharyngeal angiofibroma is most
commonly seen? (YMDC)
 middle age female
 middle age male
 teenage female
 teenage male*
 female child
290. What is the level for the lower limit of the pharynx in adults? (YMDC)
 5th cervical vertebra
 6th cervical vertebra*
 1st thoracic vertebra
 2nd thoracic vertebra
 3rd thoracic vertebra.
291. Which of the following lymph nodes receives lymphatic drainage from the palatine
tonsils? (YMDC)
 jugulo-omohyoid lymph node
 jugulodigastric lymph node*
 supra-digastric lymph node
 supra-clavicular lymph node
 sub-omohyoid lymph node
292. Some surgery was performed on a patient and post-operatively he complained of loss of
taste sensation on the anterior two third of the tongue. Which of the following surgery was
most likely done on this patient? (YMDC)
 adenoidectomy
 mastoidectomy *
 parotidectomy
 tonsillectomy
 rhinoplasty
293. 'Hot potato voice' is typically seen in cases of (YMDC)
 cleft palate
 hypertrophied adenoids
 vocal nodules
 vocal cord paralysis
 peritonsillar abscess*
294. On examination of n 35-year-old lady, laryngeal crepitus was found to be present. What is
the most likely possibility in this patient? (YMDC)
 foreign body in the hypopharynx
 fracture of the laryngeal cartilages
 normal larynx and hypopharynx*
 post-cricoid tumor
 laryngeal carcinoma
295. A 4-year-old girl came in ER and was clinically suspected to be suffering from pharyngeal
diphtheria. What complications can occur if diphtheria antitoxin is not given immediately?
(YMDC)
 myocarditis and muscle paralysis*
 nephritis and electrolyte imbalance
 peripheral neuronitis is and deafness
 pleural effusion and pneumonia
 all oldie above
296. A 17-year-old female patient presented with acute tonsillitis for last 3 days. What is the
most common bacterial organism for this condition? (YMDC)
 pneumococcus
 haemophilus influenza
 Moraxella catarrhalis
 haemolytic streptococcus*
 staphylococcus
297. Characteristic spotted appearance of the tonsil is seen in (YMDC)
 acute catarrhal tonsillitis
 acute parenchymatous tonsillitis
 acute follicular tonsillitis *
 acute membranous tonsillitis
 tuberculous tonsillitis
298. A 39-year-old male patient was planned for tonsillectomy to approach a cranial nerve in
the tonsillar bed. Which of the following nerves can be approached by tonsillectomy
operation? (YMDC)
 lingual nerve
 hypoglossal nerve
 vagus nerve
 glossopharyngeal nerve*
 accessory nerve
299. A 9-year old boy presented with grossly enlarged adenoids. On examination he also had
adenoid facies. Which of the following is a typical feature of such condition? (YMDC)
 open mouth *
 pooling of saliva
 prominent molars
 wide open nostrils
 frontal bossing
300. Plain X-ray nasopharynx lateral view was done in a 8-year-old girl which showed grossly
enlarged adenoids. What will be the typical voice of this patient? (YMDC)
 buccal voice*
 nasal voice
 hoarse voice
 oesophageal voice
 hot potato voice
301. The treatment of chronic retropharyngeal abscess is (YMDC)
 incision and drainage through oral cavity
 incision and drainage through neck *
 intra-venous antibiotics
 abscess tonsillectomy
 adenoidectomy
302. The most common complication of Ludwig's angina Is (YMDC)
 lower respiratory tract infection
 retropharyngeal abscess
 laryngeal oedema
 parapharyngeal abscess *
 mediastinitis
303. A 20-year-old girl presented with a progressively increasing swelling in the flour of the
mouth for last 6 months. On examination there was a large, smooth, rounded and bluish
swelling seen on the right side of the floor of the mouth. What is the treatment of choice in
this patient? (YMDC)
 medical or conservative
 incision and drainage
 partial glossectomy
 excision of submandibular gland
 marsupialization *
304. A 40-year-old male patient presented with a lesion in the oral cavity for which surgery was
advised. Patient refused surgery but later presented with malignant change in the lesion.
What is the most likely possibility for the primary lesion? (YMDC)
 leukoplakia
 oral sub-mucous fibrosis
 erythroplakia*
 lichen planus
 ranula
305. A 52-year-old male patient presented with a malignant tumor of the oral cavity. What is
the most likely pathology at this site? (YMDC)
 adenoid cystic carcinoma
 lymphoma
 melanoma
 muco-epidermoid carcinoma
 squamous cell carcinoma*
306. what is the most common site for origin of squamous cell carcinoma of the tongue?
(YMDC)
 tip of the tongue
 dorsum of the tongue
 lateral margin of the tongue*
 anterior half of the tongue
 posterior one third of the tongue
307. A 38-year-old male patient was diagnosed as a case of salivary calculus. What is the most
common site of stone formation in salivary gland? (YMDC)
 deep lobe of parotid
 superficial lobe of parotid and its duct
 submandibular gland and its duct*
 sublingual gland
 minor salivary glands
308. A 30-year-old female patient presented with xerophthalmia, xerostomia and connective
tissue disorder. What is the most likely diagnosis? (YMDC)
 primary Sjogren's syndrome
 secondary Sjogren's syndrome *
 viral parotitis
 all of the above
 none of the above
309. Which of the following antibiotic is secreted in salivary secretions? (YMDC)
 ofloxacin
 erythromycin
 clindamycin
 amoxicillin
 cephradine *
310. The commonest benign tumour of salivary gland is (YMDC)
 oncocytoma
 Warthin stumour
 lymphangioma
 neurofibroma
 pleomorphic adenoma*
311. Squamous cell carcinoma of larynx can spread in regional lymph nodes causing nodal
metastasis. In which of the following carcinoma, nodal metastasis is usually late? (YMDC)
 supra-glottic carcinoma
 glottic carcinoma*
 sub-glottic carcinoma
 trans glottic carcinoma
 hypopharyngeal carcinoma
312. Total laryngectomy was performed in n 55-year-old patient for extensive carcinoma of
larynx. What are the activities during which this patient can experience problem because of
removal of larynx? (YMDC)
 deep breathing, weight lifting and micturition
 expiration, defecation and deglutition
 inspiration, deep breathing and micturition
 micturition, defecation and weight lilting *
 deglutition, defecation and quick breathing
313. A female primary teacher of 35-year age, comes with complaint or hoarseness for last few
months which is usually worst after returning from the school with no other significant
complaint. What is the most probable diagnosis in this case? (YMDC)
 laryngeal diphtheria
 vocal nodules *
 laryngomalacia
 vocal cord paralysis
 intubation granuloma
314. A 18-year-old male patient had a road traffic accident and presented with supraglottic
haematoma. He has noisy breathing. What is the likely character of stridor in this patient?
(YMDC)
 biphasic
 inspiratory *
 expiratory
 sometimes inspiratory ans sometimes expiratory
 on deep expiration only
315. A 42-year-old male patient came with the complaint of hoarseness of voice. What is the
best method for assessing movements of the vocal cords among the following available
tools? (YMDC)
 micro-laryngoscopy
 flexible laryngoscopy *
 laryngoscopy with anterior commissure type laryngoscope
 laryngoscopy with sliding panel laryngoscope
 indirect laryngoscopy
316. A father brought his 4-year-old son with the complaint that he had ingested a coin two
hours back and after the incident he is complaining of difficulty in swallowing. What is the
most likely site of impaction of foreign body in this case? (YMDC)
 larynx
 cricopharyngeus*
 trachea
 oropharynx
 middle third of the esophagus
317. An 8-year-old boy was brought with the complaint that he had ingested some foreign
body which has impacted in his throat. Which of the following sign on clinical examination
is most important for diagnosis of impacted foreign body? (YMDC)
 severe trismus
 pooling of saliva on indirect laryngoscopy*
 congested larynx and pharynx *
 immobility of the tongue
 increased pulse rate
318. To dislodge foreign body from the larynx in emergency, following method is used (YMDC)
 Valsalva man oeuvre
 Troller man oeuvre
 Heimhch man oeuvre*
 Hallpike mano oeuvre
319. Laryngoscopy of a 45-year-otil patient shows bilateral nodular thickenings on the true
vocal cords. What is the most likely location of vocal nodules on true vocal cords? (YMDC)
 Junction of anterior one third and posterior two third *
 junction of anterior two third and posterior one third
 junction of middle one third and posterior one third
 close to anterior commissure
 close to posterior commissure
320. Thumb signor plain X-ray lateral view is diagnostic of (YMDC)
 laryngeal diphtheria
 laryngeal tuberculosis
 acute epiglottitis *
 chronic laryngitis
 early laryngeal carcinoma
321. The commonest organism responsible for acute laryngo-tracheo-bronchitis is (YMDC)
 Staphylococcus aureus
 Streptococcus pyogenes
 Haemophilus inflenzae
 parainfluenza virus*
 Coxsackie Virus
322. A 4-year-old boy presented with gradually progressing hoarseness of voice and dyspnea.
Direct laryngoscopy was done Which showed multiple papillomata in the larynx. Which of
the following virus is considered as aetilogical factor of this condition? (YMDC)
 HIV
 EBV
 HPV *
 ECHO virus
 Influenza virus
323. CO2 laser therapy was advised in a 54-year-old male patient who was diagnosed as a case
of squamous cell carcinoma of the larynx. In which of the following tumour this modality of
treatment is most effective? (YMDC)
 early glottis carcinoma *
 all stages of sub-glottic carcinoma
 late supra-glottic carcinoma
 trans-glottic carcinoma
 all advanced stage carcinoma
324. A 60-year-old male patient was diagnosed as a case of bronchogenic carcinoma. He
presented with vocal cord paralysis. Which of the following nerve is most likely involved in
this case? (YMDC)
 left recurrent laryngeal nerve *
 left superior laryngeal nerve
 right recurrent laryngeal nerve
 right superior laryngeal nerve
 right main trunk or vagus nerve
325. After tracheostomy operation, it 35-year-old female patient developed perichondritis of
the cricoid cartilage. What could be the most common reason for this? (YMDC)
 Anti-inflammatory drug is not prescribed alter surgery
 antibiotic is not prescribed after surgery
 high tracheostomy is performed*
 regular suction cleaning or Ilk tube is not done
 the tube is displaced accidently
326. A 40-year-old male patient is planned for a major oral cancer surgery and a preoperative
elective tracheostomy. Which of these following incision is made in this patient for
tracheostomy? (YMDC)
 horizontal incision two fingers above the suprasternal notch
 horizonal incision just at suprasternal notch
 horizontal incision at the level of cricothyroid membrane
 vertical incision from cricoid cartilage to suprasternal notch *
 vertical incision from hyoid bone to first tracheal ring
327. Which of the following type of laser is most commonly used in ENT practice? (YMDC)
 argon
 Nd-YAG
 CO2*
 KIP
 Erb-YAG
328. The importance of the Fossa of Rosen muller is (YMDC)
 angiofibroma arises from it
 carcinoma of nasopharynx usually wises from it*
 antrochoanal polyps arise from ii
 nasopharyngeal cysts arise from it
 adenoid arises from it
329. Following is true regarding Peritonsillar abscess (YMDC)
 is usually bilateral
 collection of pus is near upper pole of tonsil *
 collection of pus is near lower pole of tonsil
 is due to mycobacterium tuberculous
 is due to Pseudomonas infection
330. A 55-year-old male was diagnosed as a case of carcinoma nasopharynx. The treatment of
choice is (YMDC)
 surgery
 Radiotherapy*
 Chemotherapy
 Reassurance
 Medical treatment
331. Most associated injuries in cases of maxillofacial trauma are to the: (FMDC)
 Brain*
 cervical spine
 chest
 abdomen
332. All of the following are true about children with maxillo facial trauma except (FMDC)
 greater risk of lower cervical spine injury*
 intracrainial injury is higher
 mid-face fracture higher as childgrows
 non-accidental trauma should be Considered
333. Followings are the features of Nasopharyngeal Angiofibroma except (IMDC/FMDC)
 Usually age less than 20years
 Recurrent epistaxis
 Nasal obstruction
 Spread to surrounding areas
 Enlarged regional lymph nodes *
334. A 20-year-old male is diagnosed as having nasopharyngeal angiofibroma. The surgical
excision is planned under GA. The best method to reduce per operative bleeding is: (FMDC)
 Pre-operative radiotherapy
 Preoperative hormonal therapy
 Preoperative chemotherapy
 Preoperative Angiography to identify reeding vessels
 Preoperative Angiography with embolization *
335. Absolute indication for thyroid scan is (FMDC)
 Solitary thyroid nodule*
 Infrahyoid thyroglossal duct cyst
 Thyroiditis
 Toxic multinodular goiter
 Suspicion of malignancy
336. In the head and neck rhabdomyosarcomas are found most commonly in the (FMDC)
 Orbit*
 The ear
 Oral cavity
 Larynx
 Neck
337. Peritonsillar abscess is also known as (FMDC)
 Retropharyngeal absce
 Tonsillar abscess
 Quinsy*
 Thornwaldt abscess
 Parapharyngeal Abscess
338. Infrastapedial lesion of facial nerve causes (FMDC)
 Loss of lacrimation
 Loss of stapedial reflex
 Loss of taste*
 None of the above
339. Most important indication for tonsillectomy is (FMDC)
 Eagle's syndrome
 Halitosis
 Obstructive sleep apnea*
 Peritonsillar abscess
340. Bed of the tonsil is formed by (FMDC)
 Superior constrictor*
 Superior oblique
 Stylopharyngeus
 Palatopharyngeus
341. Commonest site for origin of nasopharyngeal carcinoma (FMDC)
 Adenoids
 Torus tubaris
 Fossa of Rosenmiuller*
 Rathke's pouch
342. Post-cricoid area is the subsite of (FMDC)
 Subglottis
 Hypopharynx*
 Nasopharynx
 Supraglottic
343. Membrane over the tonsil is seen in (FMDC)
 Leukemia
 Agranulocytosis
 Diphtheria
 All of the above*
344. All of the following conditions are associated with snoring except (FMDC)
 Angiofibroma
 Laryngeal papillomatosis*
 Antrochoanal polyp
 Adenoids
345. Laryngeal crepitus is seen in (FMDC)
 Normal persons*
 Fracture of thyroid cartilage
 Post-cricoid carcinoma
 Prevertebral abscess
346. Cricothyroid is supplied by (FMDC)
 Recurrent laryngeal nerve
 External laryngeal nerve*
 Glossopharyngeal nerve
 Hypoglossal nerve
347. All of the following laryngeal muscles are adductors of vocal cords except (FMDC)
 Lateral cricoarytenoid
 Posterior cricoarytenoid *
 Thyroarytenoid,
 Oblique arytenoid
348. The most common and earliest manifestation of carcinoma of glottis is (FMDC)
 Hemoptysis
 Stridor
 Cervical lymphadenopathy
 Hoarseness*
349. Xerostomia can be caused by al of the following except (FMDC)
 Antihistamines
 Sjogren's syndrome
 Cerebral palsy*
 Uraemia
350. Most important cause of SCC of buccal mucosa (FMDC)
 Dust pollution
 Arsenic poisoning
 Naswar addiction
 Cigarette smoking*
351. IDL shows left vocal cord immobile, the cause is (FMDC)
 Bronchogenic carcinoma *
 Vocal nodule
 Vocal polyp
 Laryngocele
352. Length of Oesophagus in adults is (FMDC)
 40cm
 25cm*
 50cm
353. Newborn having stridor which increased on crying but subsides on placing the child in
prone position, the possible diagnosis is (FMDC)
 Laryngeal web
 Laryngomalacia*
 Congenital subglottic stenosis
 Congenital vocal cord paralysis
354. Child of 4 year presented with hoarseness and croupy c Fever was 103 F. Thumb sign seen
on X-Ray neck later (FMDC)
 Whooping cough
 Diphtheria
 Acute LTB
 Acute epiglottitis*
355. Retromolartrigone is a subsite of (FMDC)
 Supraglottic
 Нуроpharyпх
 Oral cavity*
 Glottis
356. A 2Yrs old child presented in ER with Respiratory distress/stridor, change of voice, low
grade fever & barking cough. Child's parents also giving history of Upper respiratory tract
infection. What is ur diagnosis? (FMDC)
 Acute epiglottitis
 Croup *
 Acute laryngitis
 Acute tonsillitis
 both a & c
357. Which of the following is not the pan of oral cavity (FMDC)
 Lip
 Buccal mucosa
 Anterior 2/3 of tongue
 Retromolar trigone
 Posterior 1/3 of tongue*
358. 30 yrs old female patient who is a teacher by profession presented in ENT OPD is
complaining of change of voice for the last 06 months. She is also complaining of vocal
fatigue & pain in neck on prolonged phonation. O/E Her tonsils & posterior pharyngeal wall
are congested. IDL shows reddish, edemntous Ewelling at the junction of ant 1/3rd & post
2/3rd of right vocal cord. What is ur diagnosis? (FMDC)
 Vocal cord polyp *
 Carcinoma larynx
 Contact ulcer
359. Which of the following is most suitable regarding laryngocele (FMDC)
 It is the most common benign neoplasm of larynx
 It presents in neck as hard fixed non reducible swelling
 It is a retention cyst due to blockage of mucinous glands in larynx mucosa
 It commonly affects children
 It is an air filled cystic swelling due to dilatation of saccule *
360. The primary & single abductor of vocal cords is? (HBS/FMDC)
 Lateral cricoarytenoid
 Posterior cricoarytenoid*
 Interarytenoid
 Thyroarytenoid
 Cricothyroid
361. A 4 yrs old child presented with sore throat, dysphagia, odynophagia, high grade fever &
respiratory distress for the last 04 hours. O/E Child is toxic looking, leaning forward with
stridor & drooling of saliva from mouth. What is your diagnosis & what is the causative
organism? (FMDC)
 Acute epiglottitis & para influenza virus respectively*
 Acute larynx tracheobronchitis & C. diphtheria respectively
362. Which is not true about Acoustic Neuroma (FMDC/YMDC)
 ls commonest tumor of cerebellopontine angle
 in pure tone audiometry there is conductive hearing loss*
 unilateral tinnitus
 may cause facial paralysis
 may involve other cranial nerves
363. A 16 Yrs old male patient with insignificant history of immunization presented in ER with
sore throat, odynophagia, dysphonia & breathing difficulty with Rt sided neck swelling. OE
there is whitesh dirty membrane on Rt tonsil& soft palate what is your diagnosis? (FMDC)
 Acute membranous tonsillitis
 Acute epiglottitis
364. Which one of the following is incorrect regarding facial nerve (FMDC/YMDC)
 Labyrinthine portion is the shortest portion of facial nerve
 Vertical part of nerve runs in facial canal
 It gives of greater petrosal nerve in middle ear cavity
 All terminal branches arise from extra temporal/ extra cranial part*
365. Which of the following statement is not true regarding juvenile laryngeal papillomatosis?
(FMDC)
 Most common benign neoplasm of larynx in children
 Caused by Human papilloma virus 6 & 11
 Mostly affect supraglottic & glottic region
 Treatment is MLS & CO2 laser
 Malignant transformation rate is very high *
366. Taste sensation from Anterior 2/3rd of tongue is carried by (FMDC)
 Glossopharyngeal nerve
 Vagus nerve
 Hypoglossal nerve
 Greater auricular nerve
 Chorda tympani*
367. 45 Yrs old female patient presented in ENT OPD with history of progressive dysphagia,
Right sided neck swelling, weight loss & pain in right ear. O.E patient is anemic &
emaciated. Right sided neck node about 3cm in size is palpable, Flexible endoscopy shows
excessive pooling of saliva in both pyriform fossae & an exophytic growth visualized in right
pyriform sinus. What is your probable diagnosis? (FMDC)
 CA larynx
 CA Esophagus
 CA Hypopharynx *
 Laryngeal TB
 Plummer Vinson syndrome
368. In Bilateral Abductor paralysis vocal cords lie m which position? (FMDC)
 Median
 Paramedian
 Slight Abduction
 Full abduction
 Both a & b*
369. Where does Stenson’s duct opens in the oral cavity? (FMDC)
 Opposite upper 2 molar*
 Opposite lower 2 molar
 Opposite upper 2 Premolar
370. Which of the following is not the subsite of supraglottic? (FMDC)
 Epiglottis
 Aryepiglottic folds
 False cords
 True vocal cords *
 Arytenoids
371. All the muscles of the larynx are supplied by Recurrent laryngeal nerve except (FMDC)
 Vocalis muscle
 Posterior cricoarytenoid muscle
 Lateral cricoarytenoid muscle
 Cricothyroid muscle*
372. Which is supplied by external branch of superior laryngeal nerve. (FMDC)
 Posterior cricoarytenoid
 Lateral cricoarytenoid
373. 40 Yrs old male after history of RTA got blunt trauma to the neck by hitting against the
steering wheel presented in ER with Respiratory distress &neck-swelling. O/E patient is
vitally stable having Pulse 78 BPM, BP 110/70 mmHg, his neck size girth is increasing &
patient is in stridor what immediate step Will you take in emergency? (FMDC)
 Pass endotracheal tube
 Emergency tracheostomy & neck exploration *
 Transfuse blood
 Start on IV fluids
 Wait & monitor vitals
374. Neutral position of vocal cords from which adduction & abduction take place? (FMDC)
 Median
 Paramedian
 Cadaveric *
 Slight abducted position
 Both a & b
375. A child 5 years of age presented with difficulty in speech O/E The child cannot protrude his
tongue past his lower incisors. What is your diagnosis? (FMDC)
 Ranula
 Tongue tie *
 Erythroplakia
 Leukoplakia
 Tonsillitis
376. Preferred tracheal rings at which tracheostomy is done? (FMDC)
 1st tracheal ring
 2nd & 3rd tracheal ring *
 4th & 5th tracheal ring
 6th tracheal ring
 Both b & c
377. Stone formation is common in which of the following salivary gland duct? (FMDC)
 Stenson duct
 Sublingual duct
 Submandibular gland duct*
 Minor salivary gland duct
 All of the above
378. 25 yrs old female presented in ENT OPD with progressive dysphagia for solids for last 06
months on examination signs & symptoms of iron deficiency anemia are also present, what
is your probable diagnosis? (FMDC)
 Pharyngeal pouch
 Plummer Vinson syndrome*
 Achalasia cardia
 Carcinoma esophagus
 GERD
379. 70 yrs old male who is a known smoker & alcohol addict for the last 13 years presented in
ENT opd with Respiratory distress stridor for the last 2 days &change of voice for the last 03
Months There are no neck nodes palpable On indirect laryngoscopy left vocal cord is
immobile & has fungating growth at the junction of ant 1/34 & post 2/34d what is probable
diagnosis? (FMDC)
 Vocal cord nodule
 Vocal cord polyp*
 Carcinoma Larynx
 Carcinoma hypopharynx
 Laryngeal haemengionma
380. Regarding Herpangina which is true (FMDC)
 It is a coxsackie viral infection mostly affecting children *
 It is a benign neoplasm of oral cavity
 Malignant transformation rate is very high
 It is caused by candida albicans
381. Most common cause of congenital stridor is? (FMDC)
 Congenital vocal cord paralysis
 Subglottic haemangioma
 Congenital subglottic stenosis
 Laryngeal web
 Laryngomalacia*
382. Which Histopathological variety of malignancy is most common in oral cavity? (FMDC)
 Adenocarcinoma
 Pleomorphic adenoma
 Basal cell carcinoma
 Squamous cell carcinoma*
 Lymphoma
383. 45 yrs old male patient presented in ENT OPD with Relative dysphagia both for solids &
liquids, halitosis & regurgitation of undigested food at nighttime. Patient is also giving
history of recurrent chest infection what is your diagnosis? (FMDC)
 Zenker's diverticulum *
 Plummer vinson syndrome
 Achalasia cardia
 Carcinoma hypopharynx
 Carcinoma esopliagus
384. A 50-year-old male who presented with hoarseness of voice. He has taken medicines for
more than 1-month durations. He is smoker. What will be the best option? (RIHS)
 Continue medical treatment & follow up
 Endoscopic laryngeal examination*
 Voice rest & vocal rehabilitation
 Self-limiting requiring No treatment
 Total laryngectomy.is the treatment of choice
385. A 12 years girl presents with a mass in the midline of neck just below the hyoid bone. It
moves with protrusion of tongue. The most probable diagnosis is (RIHS)
 Cystic Hygroma
 Thyroglossal duct cyst*
 Bronchial cleft cyst
 Dermoid cyst
386. A 3 years old male child is suffering from low grade fever and sore throat. On examination,
he has toxic look, has membrane adherent to tonsils and adjoining pharynx which is
difficult to remove. His pulse is fast and thready. Multiple lymph nodes in neck are also
enlarged. What is the most probable clinical diagnosis? (RIHS)
 Acute follicular tonsilitis
 Pharyngeal diphtheria*
 Ludwig's angina
 infectious mononucleosis
 Acute epiglottitis
387. A 65 years old male presented with dysphagia and regurgitation of food at night for the
last few months. He also co plains of weight loss and bad mouth breath. On examination,
there is a soft swelling under the left sternomastoid Muscle which gurgles on palpation.
What is your probable diagnosis? (RIHS)
 Laryngocele
 Zenker diverticulum*
 Esophageal web
 Hiatus hernia
 Esophageal strictures
388. A 25-year-old schoolteacher present with a 3 month history of hoarseness. On
laryngoscopy well circumscribed lesions are seen at the junction of the anterior third and
posterior two third of both vocal cords. What is the diagnosis? (RIHS)
 Carcinoma of larynx
 Intubation granuloma
 Thyrotoxicosis
 Vocal cord nodules*
 Vocal cord polyp
389. A 25 years male presented with corneal, oral & genital ulcers refractory to routine
treatment. The likely cause is (RIHS)
 Leukemia
 Agranulocytosis
 Behecet’s disease*
 Iron deficiency
 Aphthus ulcers
390. The best way of management of living insect in external auditory canal is (RIHS/FMDC)
 Kill insect by instilling ether and chloroform in ear
 Removal under general anesthesia with microscopic control
 Removal by syringing in OPD
 Kill the insect by instilling water or oil in the ear & then remove it*
 Removal by crocodile forceps
391. A picnic party is going on a riverbank about 35kms away from the town. Suddenly a 35
years old male became severely dyspneic due to impaction of bolus of food in throat. All
non-provocative procedures have failed to relieve dyspnea. The person is becoming
increasingly cyanosed. Which one is the only choice to overcome this problem? (RIHS)
 Elective Tracheostomy
 Cricothyrotomy*
 Endotracheal Intubation
 Direct Laryngoscopy ant removal of foreign body
 Laryngeal mask
392. Regarding blow out fracture all are true except (YMDC)
 There is always exophthalmos.*
 Diplopia.
 Tear drop sign on radiograph of maxillary sinuses
 Require Caldwell luc's operation tor reduction
 Open and close reductions are the options of treatment
393. In malignant tumor of tongue, which is true: (YMDC)
 Pan and betel nuts are common etiological factors*
 Adenoid cystic carcinoma is the most common
 Dorsum of tongue is common site
 More common in female
394. Patients with Peritonsillar abscess usually presents with (YMDC)
 Odynophagia*
 Neck swelling
 Unilateral earache
 Stridor
395. Sialolithiasis is common in (YMDC)
 Submandibular Gland*
 Sublingual Gland
 Paranoid Gland
 Minor Salivary Gland
396. Main Stay of treatment in Ludwig's angima is (YMDC)
 Supportive therapy
 Antibiotics*
 Incision and drainage
 Radiation
397. The most common cause for stridor in a newborn infant is (YMDC)
 Laryngeal web
 Subglottic Hemangioma
 Laryngomalacia*
 Congenital Subglottic stenosis
398. Following nerves can frequently be damaged in thyroid surgery (YMDC)
 Vagus nerve and superior laryngeal nerve
 External laryngeal nerve and internal laryngeal nerve
 Recurrent laryngeal nerve and superior laryngeal nerves*
 Superior laryngeal nerve
399. Regarding vocal cord polyp, which of the following is not correct? (YMDC)
 Forms due to sub mucosal edema
 Usually bilateral*
 Hoarseness, choking. stridor and dyspnea are common Symptoms
 Treatment is surgical remove
400. Treatment of chronic simple laryngitis include all except (YMDC)
 Steam inhalation and avoidance of allergies
 Voice rest and speech therapy
 Treatment of URTI
 Stripping of vocal cords*
401. Regarding aphthous ulcers which one is incorrect (YMDC)
 Highly painful
 Usually present on hard palate*
 Treatment include local steroid
 Related to tension or unknown etiology
402. which one is not a cause of stridor? (YMDC)
 Vocal nodule*
 Glottic cancer
 supraglottic cancer
 Laryngomalacia
 FB airway
403. Which one of the following is not part of Oropharynx? (YMDC)
 Tonsils
 Posterior Pharyngeal wall
 upper surface of soft palate*
 Base of tongue
404. Regarding tracheostomy which option is incorrect (YMDC)
 indicated in stridor
 Indicated in advanced stage of carcinoma larynx
 Hole is made in the 1st ring of trachea*
 Laryngeal stenosis is an important complication
405. Which one is causative organism in 3 years child with acute epiglottitis? (YMDC)
 H. Influenza type B*
 Paramyxo virus
 Pneumococeus
 Pseudomonas
406. 25 years old female, Hb 8.5, presented with progressive dysphagia for solids for last one
year and stomatitis for 6 months Probable diagnosis is: (YMDC)
 Cancer esophagus
 Pharyngeal pouch
 Plumer Vinson syndrome *
 Cancer Larynx
407. Arnold Nerve is a branch of (YMDC)
 Vagus nerve*
 Glossopharyngeal nerve
 Auriculotemporal nerve
 Facial nerve
408. inferior Salivary Nucleus is related to (YMDC)
 Hypoglossal nerve
 Glossopharyngeal nerve*
 facial nerve
 Trigeminal nerve
409. Multiple matted cervical lymphadenopathies is feature of (YMDC)
 Tuberculosis*
 Lymphoma
 Squamous cell carcinoma
 Infectious mononuclcosis
410. 14 years old girl has developed hoarseness of voice after hearing the news of her failure in
the examination. Her cough is normal. On indirect Laryngoscopy, both vocal cords look
normal. What is one best line of treatment?
 Voice rest and steam inhalation
 Psychotherapy*
 Speech therapy
 Medializations of one cord
 Antibiotic and antipyretics
411. A 4 years old child is suffering from dyspnea, stridor & dysphagia. Clinical diagnosis is
Acute Epiglottitis. Which is first and most important step to be taken in this patient?
 Start broad spectrum I/V antibiotics
 Carry out clinical ENT examination to confirm the diagnosis
 Ask for throat swab C/S
 Secure airway by intubation/tracheostomy*
 Lateral neck soft tissue x ray
412. What is the common microorganism of the Acute Laryngotracheobronchitis?
 Streptococcus
 E coli
 Haemophilus influenza
 Staphylococcus
 Para influenza*
413. A 60 years old male presented with a growth on true vocal cord on right side which on
histopathology showed squamous cell carcinoma. What is the most common site of origin
of this type of tumor? Anterior commissure
 Anterior half of the vocal cord
 At the middle part of vocal cord*
 Posterior commissure
 Posterior half of the vocal cord
414. A 32 years old female schoolteacher presented in ENT OPD with complaint of gradually
progressive hoarseness of voice for last 2 years. Direct laryngoscopy was performed which
showed nodular lesion on the true vocal cord. What is the most likely appearance of this
nodular lesion?
 Unilateral and purplish red in color
 Bilateral and purplish red in color
 Unilateral and grayish white in color
 Bilateral and grayish white in color*
 Bilateral and bluish in color
415. A 50-year-old man who smokes 20 cigarettes a day presents with a 2 month history of
hoarseness and dyspnea, a mass is palpable in neck at right level III
 Carcinoma of larynx*
 Intubation granuloma
 Thyrotoxicosis
 Vocal cord nodules
 Vocal cord polyp
416. A 25-year-old schoolteacher presents with a 3 month history of hoarseness on
laryngoscopy well circumscribed lesions are seen at the junction of the anterior third and
posterior two thirds of both vocal cord (RIHS)
 Carcinoma of larynx
 Intubation granuloma
 Thyrotoxicosis
 Vocal cord nodules*
 Vocal cord polyp
417. A 40-year-old man is recovering from a life-threatening illness which required intubation
and ventilation for 3 weeks. He developed hoarseness of voice after a month. What is the
diagnosis? (RIHS)
 Carcinoma of larynx
 Intubation granuloma*
 Thyrotoxicosis
 Vocal cord nodules
 Vocal cord polyp
418. A tracheostomized patient, with portex tracheostomy tube in the ward, developed sudden
Blockage of the tube. Which of the following is best next step in the management? (RIHS)
 Immediate removal of the tracheostomy tube
 Suction of tube with sodium bicarbonate
 Suction of tube with saline
 Jet ventilation
 Positive pressure oxygen inhalation*
419. A 40 years old smoker, one-year HX of hoarseness, dysphagia for 2 months
 Vocal Nodule
 Cancer larynx*
 Recurrent Laryngeal nerve palsy
 Acute laryngitis
 Vocal Polyp
420. A teacher presents with permanent hoarseness for 2 weeks.IDL showed nodules
 Vocal Nodule*
 Cancer larynx
 Recurrent Laryngeal nerve palsy
 Acute laryngitis
 Vocal polyp
421. Hoarseness following thyroidectomy.
 Vocal Nodule
 Cancer larynx
 Recurrent Laryngeal nerve palsy*
 Acute laryngitis
 Vocal polyp
422. Post thyroidectomy patient presents with hoarseness of voice and severe respiratory
distress on IDL
 Recurrent laryngeal nerve
 Superior laryngeal nerve
 Vagus nerve
 Bilateral recurrent laryngeal nerve palsy*
 Vocal polyp
423. Vocal cords are lined by
 Squamous epithelium*
 Transitional epithelium
 Ciliated columnar epithelium
 Cuboidal epithelium
 columnar epithelium and
424. Larynx develops during
 4th week of intrauterine life*
 6th week of intrauterine life
 8th week of intrauterine life
 14th week of intrauterine life
 16th week of intrauterine life
425. Only muscle that opens the glottis is
 Vocalis
 Intarytenoid
 Posterior Cricoarytenoid*
 Mylolyoid
 Throarytenoid
426. The most common congenital laryngeal Anomaly
 Laryngeal web
 Subglottic stenosis
 Laryngomalacia*
 Vocal cord paralysis
 Sub glottis hemangioma
427. A 50 years old male who presented with hoarseness of voice more than 1-month duration,
have taken medications. He is smoker what will be the best option?
 Continue medical treatment follow up
 Endoscopic laryngeal examination*
 Voice rest & vocal rehabilitation
 Self-limited No treatment required
 Total laryngectomy is the treatment of choice.
428. A 3-year-old boy complaint of sudden acute respiratory distress, with spasmodic cough,
cyanosis & acting necessary respiratory muscles is most probably due to
 Acute epiglottitis
 Vocal cord paralysis
 Acute follicular tonsillitis
 Laryngeal web
 Foreign body Inhalation*
429. Early glottic carcinoma is best treated with
 Radiotherapy*
 Antibiotics
 Total laryngectomy
 Cryosurgery
 Chemotherapy
430. Commonly used laser in ENT
 Carbon dioxide*
 ARGONYA
 KAPT
 CRYO
431. The part of larynx lined by Squamous epithelium is
 Epiglottis
 Vestibular fold
 Vocal cords*
 Subglottis
 False vocal cord
432. Potentially serious clinical feature of acute laryngitis includes
 Aphonia*
 Dyshonia
 Cough
 Stridor
 Wheezing
433. In tracheotomy, patient breaths through the tracheostomy hole and which of the disability
is due to loss of laryngeal function
 Air is not humidified during inspiration
 Air is not warmed during inspiration
 Air is not cleansed during respiration
 Hoarseness of voice
 Inability to speak*
434. Voice is not good without dyspnoae
 Bilateral vocal cord paralysis with vocal cords near the midline
 Bilateral vocal cord paralysis with vocal cords away from the midline in cadaveric position
 Left unilateral vocal cord paralysis *
 Right vocal cord paralysis
 Left unilateral vocal cord paralysis in cadaveric position
435. Voice may be good with dyspnoae.
 Bilateral vocal cord paralysis with vocal cords near the midline*
 Bilateral vocal cord paralysis with vocal cords away from the midline in cadaveric
 Laryngomalacia
 Left unilateral vocal cord paralysis
 Left unilateral vocal cord paralysis in cadaveric position
436. Tracheostomy will help the patient in all except
 Bilateral vocal cord paralysis with vocal cords near the midline
 Bilateral vocal cord paralysis with vocal cords away from the midline in cadaveric position
 Laryngomalacia*
 Left Unilateral vocal cord paralysis
 Stroke
437. Voice is hoarse in the beginning but recovers with passage of time
 Bilateral vocal cord paralysis with vocal cords near the midline
 Bilateral vocal cord paralysis with vocal cords away from the midline in cadaveric position
 Laryngomalacia
 Left unilateral vocal cord paralysis, in cadaveric position
 Left unilateral vocal cord paralysis, in paramedian position*
438. Vocal cord medialization will help in
 Bilateral vocal cord paralysis with vocal cords near the midline
 Bilateral vocal cord paralysis with vocal cord away from the midline in cadaveric position
 Laryngomalacia
 Left unilateral vocal cord paralysis*
 Bilateral recurrent nerve paralysis
439. The most common cause of unilateral vocal cord paralysis is
 Thyroid surgery*
 Thoracic surgery
 Stroke
 Idiopathic
 Diabetes
440. A 32-year-old shopkeeper has 5 year history of hoarseness that sometimes improves.
There is no dysphagia & had never smoked. IDL Shows small nodules involving both vocal
cords in their anterior third. Treatment includes none of the following except
 Voice rest & steam inhalation alone*
 Coagulation diathermy of nodules
 Microlaryngoscopic excision and voice rehabilitation
 Radiotherapy
 Antibiotics & corticosteroids orally
441. Which of the following is true regarding vocal cord nodules?
 They are often seen in men
 They are mostly bilateral*
 They are most treated surgically
 They are rarely associated with vocal abuse
 Smoking is main cause
442. A 35 years old female patient complains of persistent lump in throat and sticking of food
during swallowing for the last 2 years, X-rays lateral view soft tissue neck, barium swallow,
endoscopy, blood picture, electrolytes, PH and manometry studies have been normal.
What is the possible cause?
 Severe esophagitis
 Gastroesophageal reflux
 Globus hystericuspharyngeous*
 Hiatus hernia
 Plummer Vinson syndrome
443. A 28 years old male presents with recurrent pain in the throat and swelling submandibular
region of left side which exaggerates on chewing and especially with soar food for the last 2
years. Swelling in submandibular region has become persistent for the last 6 weeks. On
bimanual examination, left submandibular gland is enlarged and tender. What is the single
most probable diagnosis? (IMDC/RIHS)
 Malignant growth left submandibular gland
 Benign growth left submandibular gland
 Calculus left submandibular gland*
 Acute submandibular abscess
 Left sided mumps
444. A 1 year’ female presents in ENT OPD with complaint of a swelling in front of her neck. On
examination, there was a single, non-tender swelling which was moving on swallowing and
on tongue protrusion. What is the most likely diagnosis?
 Branchial cyst
 Dermoid cyst
 Thyroglossal cyst*
 Thyroid nodule
 Cystic Hygroma
445. 25 years old lady is suffering from dysphagia of one-year duration. On examination she
looks anemic has koilonychia and complete blood picture reveals hypochromic, microcytic
anaemia neck examination NAD. One most probable diagnosis
 Reflux stricture
 Malignant growth oesophagus
 Plummer Vinson syndrome*
 Achalasia cardia
 CA hypopharynx
446. A 45 years thin lady presents with progressive increasing difficulty in swallowing. She is
markedly anemic and having angular stomatitis koilonychia& weight loss. What is your
diagnosis?
 Growth esophagus
 Plummer Vinson syndrome (Patterson prawn kelling syndrome)*
 Growth larynx
 Growth larynx
 Growth oropharynx
447. A 68 years old male has developed Dysphagia of gradual onset and soft swelling on the left
side of neck which can be reduced with gurgling noise. He also regurgitates undigested
food. What is the first single best diagnostic method?
 Endoscopy rigid
 Plain X-rays neck
 Barium swallow *
 CT scan
 MRI
448. 8 years old boy presented with dysphagia for solids for one year, stricture was seen on
barium swallow in third of esophagus. The most common cause upper one of esophageal
stricture in children is
 Diphtheria
 Achalasia cardia
 Corrosive intake *
 Coin ingestion
 carcinoma
449. A 68 years old male has developed Dysphagia of gradual onset and soft swelling on the left
side of neck which can be reduced with gurgling noise. He also regurgitates undigested
food. Diagnosis is
 Pharyngeal pouch*
 CA hypopharynx
 Laryngocele
 CA esophagus
 Cardiac achalasia
450. A 2 years old patient has swallowed coin. Which one of the following is most common site
for impaction of such foreign body?
 Lower end of oesophagus
 Cricopharyngeal (Upperend)*
 Left bronchus crossing oesophagus
 Arch of aorta crossing oesophagus
 At level of diaphragm
451. A 40 years old lady has dysphagia for the last 6 months. Dysphagia has been progressive
and initially associated with burning sensation in chest and frequently hoarseness of voice,
she has been under treatment of gastroenterologist There is no significant weight loss.
Barium swallow shows a smooth stricture at lower part and dilatation of oesophagus in
upper2/3rd. What is one best possible diagnosis?
 Carcinoma lower end of oesophagus
 Achalasia cardia *
 Stricture due to reflux oesophagitis
 Congenital stricture lower end of oesophagus
 Stricture due to corrosive ingestion
452. In a case of dysphagia suspected due to carcinoma esophagus, which one procedure is
most important to confirm diagnosis?
 Barium swallow
 CT scan neck and chest
 PH and manometry studies
 Esophagoscopy and biopsy *
453. In cadaveric dissection total length of esophagus was measured, it was
 30cm
 20cm
 25cm*
 40cm
 15cm
454. 8 years old boy presented with high grade fever. trismus, bilateral swelling over parotid
region most probable diagnosis?
 Tumour of parotid gland
 Mumps *
 Sjogren syndrome
 Bacterial parotitis
 Peri tonsillar abscess
455. 5 years old male presented with severe pain in chest during meal and dysphagia for last 5
years . On barium swallow rat tail appearance was seen most probable diagnosis. (IMDC)
 Carcinoma lower end of esophagus
 Plummer Vinson syndrome
 Gastro esophageal reflux
 Cardiac achalasia*
 Foreign body esophagus
456. 30 years old male presented with fever, severe pain during eating on right parotid region
after having appendicectomy pain was more severe while taking spicy & sour food. On
examination opening of right parotid duct in oral cavity was swollen Common diagnosis
 Mumps
 Parotitis*
 TM joint dislocation
 Sjogren syndrome
 Tumour of parotid gland
457. Adult male having painless, gradually increasing swelling in left Submandibular region
with no association of food intake for the last 2 years. On examination swelling is globular
and non tender. What is the single most probable diagnosis?
 Malignant growth left Submandibular gland
 Benign growth left Submandibular gland
 Stone in left Submandibular duct*
 Acute Submandibular abscess
 Acute sialadenitis
458. 22 years old male presented with bluish swelling in the floor of mouth, soft and cystic in
consistency All are true except:
 Swelling of sublingual gland
 Retention cyst
 Blue in color
 Punjent type seen in neck
 Is a tumour *
459. 22 years old male presented with bluish swelling in the floor of mouth, soft and cystic in
consistency. Diagnosis is
 Calculus in submandibular gland duct
 CA floor of mouth
 Rannula *
 Benign tumour of submandibular gland
 Haemangioma
460. 10 years old boy presented with swelling of right parotid region, right facial nerve
paralysis. Most common tumour at this age
 Pleomorphic adenoma
 Muco epidermoid *
 Monomorphic adenoma
 Squamous cell carcinoma
 Adeno carcinoma
461. F.B in esophagus can be impacted at anatomical constrictions in esophagus, these are all
except
 At the level of diaphragm
 At the level of Aortic arch
 Left main bronchus
 Left subclavian artery*
 Crico pharyngeus
462. ENT surgeon is doing esophagoscopy, Length of esophagoscope is
 30cm
 25cm
 15cm
 40cm
 45cm *
463. 40 years old female, overweight, presented with recurrent hoarseness of voice, burning
sensation in chest, recurrent sore throat and dysphagia for 2 years. On examination throat
was congested, inlet of larynx & vocal cords was congested. most probable diagnosis
 CA Hypo pharynx
 Gastroesophageal reflux *
 CA larynx
 Cardiac achalasia
 Chronic laryngitis
464. patient with 60 years of age presented with pain in right ear, T-M is normal, with history
of mild Dysphagia. First important investigation is
 Blood CP
 C.T scan
 X-ray soft tissue neck *
 Barium swallow
 Endoscopy
465. 8 years old boy presented with sinus along anterior border of R sternocleidomastoid just
above the level of hyoid bone. Most probable diagnosis
 Thyroglossal cyst
 Sinus in metastatic lymph node
 Branchial sinus *
 Collarstud fistula
 Histo cytosis multiplex
466. On Barium swallow of 60 years old male pharyngeal pouch is seen, most common site is
 Oropharynx
 Nasopharynx
 Hypopharynx *
 Vocal cords
 Nasopharynx
467. Killian's dehiscence, is seen in
 Oropharynx
 Nasopharynx
 Cricopharynx
 Hypopharynx *
 Vocal cords
468. One of the following is Not a feature of chronic pharyngitis:
 Pain in throat
 Cough
 Tiredness of voice
 Foreign body sensation in throat
 Fever*
469. A middle age anemic lady is having progressive dysphagia for solids and brittle nails for 2
years. Her neck examination is normal. The likely diagnosis is:
 Carcinoma larynx
 Carcinoma pharynx
 Globus pharyngeal
 Plummer Vinson syndrome*
 Pharyngeal pouch
470. A middle age lady Is having anemia, difficulty swallowing and spoon shaped nails. The
diagnosis Is (RIHS)
 Leukemia
 Carcinoma larynx
 Carcinoma pharynx
 Pharyngeal pouch
 Plummer Vinson syndrome*
471. Patient with high grade fever, headache, and nasal obstruction is case of (FMDC)
 Bilateral ethmoidal polyps
 Bilateral inferior turbinate hypertrophy
 Acute sinusitis*
 Atrophic sinusitis
472. Most common organism involved in acute Epiglottitis (FMDC)
 Staphylococcus aureus
 Streptococcus pneumonia
 H. Influenzas*
 Pseudomonas aeruginosa
473. All are characteristics of geographical tongue except (FMDC)
 Congenital*
 Adenoid of papillae
 Migratory glossitis
 irregular keratotic white outlines
474. The name of the cartilage formed in the aryepiglottic fold is (FMDC)
 Cuneiform*
 Corniculate
 Arytenoid
 None of the above
475. Investigation of laryngeal cancer all are correct except (FMDC)
 Sputum for AFB*
 Direct laryngoscopy
 Biopsy
 C.T scan
476. A 6 yr old child with recurrent URTI with mouth breathing and failure to grow with high
arched palate and impaired hearing, treatment option is: (HBS)
 Tonsillectomy
 grommet insertion
 myringotomy with grommet insertion
 adenoidectomy with grommet insertion*
477. Abductors of vocal cords (HBS)
 Cricothyroid.
 Posterior cricoarytenoid*
 lateral cricothyroid.
 Inter arytenoid
 Non of the above
478. Bowing of vocal cords occur in (HBS)
 Presbylarynx
 Reinke's oedema
 Vocal cords haemorrhage
 Denervation of vocal cords*
 Non of the above
479. All of the following may cause tonsillar ulceration except: (HBS)
 Diphtheria.
 Behchet's syndrome
 Vincent's angina.
 Agranulocytosis.
 Papilloma. *
480. Thumb sign is seen in (HBS)
 Acute laryngitis
 Acute epiglottitis*
 Acute laryngotracheo bronchitis
 Acute tonsillophayngitis
 Laryngeal diphtheria
481. Which statement is CORRECT regarding vocal nodules? (HBS)
 Are benign lesions*
 Vocal nodules tends to be larger than laryngeal polyps.
 Develop at the middle of vocal cords
 Seldom develop bilaterally.
 Can be prevented by immunization.
482. Laryngomalacia: (HBS)
 The larynx is of an exaggerated adult type.
 The epiglottis is long and wide and folded backward at each lateral edge*
 The epiglottis is converted into a delta shaped incomplete cylinder
 The glossopharyngeal folds are approximated.
 None of the above
483. All are causes of congenital strider except: (HBS)
 Acute epiglottitis*
 Laryngeal web.
 Subglottic stenosis.
 Laryngomalacia.
 Vascular anomaly.
484. Which of the following is part of laryngopharynx? (HBS)
 Epiglottis.
 Cricoids cartilage.
 Thyroid cartilage.
 Postericoid*
 Arytenoids.
485. Absolute indication of tonsillectomy is: (HBS)
 Chronic tonsillitis
 Diphtheria carrier.
 Obstructive sleep apnoea *
 Glossopharyngeal neurectomy.
 Rheumatic fever.
486. Which of the following is not a complication of acute tonsillitis? (HBS)
 Peritonsillar abscess.
 Bezold's abscess*
 Acute rheumatism
 Acute nephritis.
 Acute infection of middle ear cleft
487. Pyriform fossa lies: (HBS)
 Medial to aryepiglottic fold
 Lateral to aryepiglottic fold *
 Inferior to aryepiglottic fold.
 Superior to aryepiglottic fold.
 None of the above.
488. A newborn baby presented with weak cry and hoarseness, the most probable diagnosis:
(HBS)
 Laryngomalacia.
 Subglotic stenosis.
 Subglotic heamangioma.
 Vocal cord palsy*
 Tracheomalacia
489. Which of the following is not a feature of pharyngeal diphtheria?
 Enlarged tender cervical lymph nodes.
 Ругехiа.
 True membrane*
 Toxaemia.
 Bull's neck.
490. A 20- year-old man presents to the ENT clinic with sudden onset high fever of one day
duration, on examination: both tonsils are congested with whitish membrane over the
right tonsil with petichae over the palate and palpable both jugulodigastric lymph nodes.
What is the most possible diagnosis? (HBS)
 Acute membranous tonsillitis
 Infectious mononucleosis*
 Diphtheria
 Quinsy
 Leukaemia.
491. All are functions of the larynx except: (HBS)
 To protect the lungs.
 To control air flow
 Phonation.
 Resonator of speech*
 To build positive intrathoracic pressure.
492. Stridor: (HBS)
 It is an auditory manifestation of disordered respiratory function due to air
 flow changes within the larynx, trachea, or bronchi.
 It needs investigations in every case
 It is due to turbulence of air flow within a partially obstructed respiratory tract
 All the above*
493. A three year old mongol boy get high fever then associated with difficult swallowing and
strider, the child was sitting , cannot sleep and drooling saliva, the resident pediatrician
sent for lateral cervical X-ray then he asked your opinion about what was looking as a
thumb in front of the hypopharynx, your diagnosis was: (HBS)
 Acute pharyngitis.
 Acute simple laryngitis.
 Laryngo-tracheo-bronchitis.
 Acute epiglottitis*
 Croup.
494. What is not correct in acute epiglottitis? (HBS)
 Constant supervision in hospital is mandatory.
 Dyspnea may be progressing and alarming.
 It is a special form of acute laryngitis, in which the inflammatory changes affect mainly
the loosely attached mucosa of the epiglottis.
 Systemic antibiotics is not a must to be started immediately*
 Age incidence is between 2-6-year-old
495. What is true about lymphatic drainage of the vocal cords? (HBS)
 It has poor lymphatic drainage.
 It has rich lymphatic drainage
 It has no lymphatic drainage*
 It has a lymphatic drainage only in the anterior half.
 None of the above
496. Open bite may be secondary to all except: (FMDC)
 LeFort Fracture
 tripod fracture
 mandibular fracture
 NEO fracture*
497. Cribriform plate is a part of (FMDC)
 Sphenoid bone
 Ethmoid bone*
 Maxilla
 Frontal bone
498. Opening of nasolacrimal duct is situated in (FMDC)
 Superior meatus
 Middle meatus
 Inferior meatus*
 Ethmoid infundibulum
499. Which of the following is standard surgical treatment for Ranula? (FMDC)
 Excision
 Cautrization
 Laser ablation
 Excision & marsupialization*
500. Arnold Nerve & Jacobson nerve are branches of (FMDC)
 Vagus nerve & Glossopharyngeal nerve respectively*
 Glossopharyngeal nerve & vagus nerve respectively
 Auriculotymporal nerve & facial nerve respectively
 Facial nerve and vagus nerve
 Auriculotemporal nerve & vestibulocochlear nerve respectively
501. Regarding Heimlich manoeuvre which is true? (FMDC)
 Manoeuvre for laryngeal foreign body removal *
 Manoeuvre for tracheal & bronchial foreign body removal
 Manocuvre for Eustachian tube dysfunction
 Manoeuvre for diagnosis of vertigo
 Both a & b
502. The sensory nerve supply of the larynx below the vocal cords is through? (FMDC)
 Recurrent laryngeal nerve *
 Internal laryngeal nerve
 External laryngeal nerve
 Superior laryngeal nerve
 Both b & c
503. Immediate mastoid exploration is required in (FMDC/YMDC)
 Facial nerve paralysis
 Labyrinthitis
 Meningitis
 Brain abscess
 Mastoiditis*
504. In a patient suffering, from purulent discharge & attic perforation. Which one is correct?
(RIHS)
 Treatment is essentially surgical*
 Medical treatment & regular follow up
 Self-limited & no treatment required
 Myringoplasty is the only needed treatment
 Topical car drops & follow up is sufficient
505. Trismus accompanying Peritonsillar abscess is due to spasm of which muscle? (RIHS)
 Masseter Pharyngeal constrictors
 Medical Pterygoid*
 Temporalis
506. A 25-year-old gentleman underwent Intranasal polypectomy. You received a surgical
specimen showing classic dumb bell appearance. What may be the possible disease?
(FMDC)
 Antrochoanal polyp
 Bilateral nasal polyp*
 hypertrophied interior turbinate
 Malignant growth
507. Juvenile angiofibroma is characterized by (FMDC)
 Nasal obstruction
 Epistaxis
 Incidence in males
 All of the above*
508. Cystic hygromas are found most commonly in (FMDC)
 The anterior triangle
 The posterior triangle*
 Occipital region
 Extremities
509. Arteries which takes part in "Kiesselbach's plexus" include all except
 Ant Ethmoidal artery
 greater palatine artery
 Post. Ethmoidal artery *
 sup. Labial artery
 sphenopalatine artery
510. McEwen's triangle is the surface landmark of (HBS)
 The tympanic part of the facial nerve
 Mastoid antrum*
 Dome of the lateral semicircular canal
 Ieudo-stapedial joint
511. Most important investigation for Carcinoma nasopharynx is
 X-Ray lateral view neck
 CT scan
 Angiography
 Nasoendoscopic Biopsy*
 MRI skull
512. The treatment of choice in Carcinoma nasopharynx is
 Surgical excision
 Chemotherapy
 LASER ablation
 Cryosurgery
 Radiotherapy*
513. Nasopharynx extends from base of skull to
 Hyoid bone
 Base of tongue
 soft palate
 Hard palate*
 Cricoid cartilage
514. Treatment of choice for Juvenile nasopharyngeal angiofibroma is
 Reassurance
 Surgical excision*
 Chemotherapy
 Radiotherapy
 LASER
515. The most common site of origin of Juvenile nasopharyngeal angiofibroma is
 Nasopharynx
 Posterior end of inferior turbinate
 Foramen ovale
 Sphenopalatine foramen*
 Roof of the nose
516. Surgery was planned in a patient with nasopharyngeal angiofibroma Which procedure is
the most appropriate one to perform before surgery
 CT Scan
 MRI
 Endoscopy
 Biopsy
 Angiography & Embolization*
517. Which characteristic is Not associated with Juvenile nasopharyngeal angiofibroma
 Benign
 Noninvasive*
 Adolscent male patient
 Nasal obstruction
 Severe epistaxis
518. What is the corresponding vertebra for lower limit of the pharynx
 5th cervical vertebra
 6th cervical vertebra*
 1st thoracic vertebra
 1st cervical vertebra
 3rd cervical vertebra
519. Which wall of the nasopharynx Eustachian tube opens
 Roof of nasopharynx
 Anterior wall of nasopharynx
 Lateral wall of nasopharynx*
 Medial wall of nasopharynx
 Posterior wall of nasopharynx
520. Juvenile nasopharyngeal angiofibroma is characterized by the following except:
 Affects teenagers
 Causes nasal obstructions
 Very vascular tumor
 Affects only females*
521. Early sign of nasopharyngeal carcinoma is:
 Unilateral secretory otitis media*
 Trotter’s triad
 Petrositis
 Nasal obstruction
522. Nasopharyngeal carcinoma is treated by:
 Antibiotics
 Surgery
 Radiotherapy*
 Non of the above
523. Nasopharyngeal angiofibroma is treated by:
 Antibiotics
 Nasal packing
 Surgical excision*
 None of the above
524. Which of the following viruses is associated with nasopharyngeal carcinoma? (IMDC)
 Herpes simplex virus
 Human papilloma virus
 Epstein-Barr virus*
 Respiratory syncytial virus
 Human immunodeficiency virus (HIV)
525. Which of the following is the best treatment modality for nasopharyngeal carcinoma?
(IMDC)
 Nasopharyngectomy
 Radiotherapy alone
 Chemotherapy alone
 Combined chemoradiotherapy*
 Nasopharyngectomy with post-operative radiotherapy
526. What is the earliest and most common clinical feature of nasopharyngeal carcinoma?
(IMDC)
 Epistaxis
 Hemoptysis
 Nasal obstruction
 Cervical lymph node metastasis*
 Otitis media with effusion
527. Tuberculosis of larynx is almost always secondary to: (IMDC)
 Pulmonary tuberculosis*
 Extra-pulmonary tuberculosis
 Caries spine
 Cervical Lymph Nodes
 Oral Cavity
528. Pneumothorax may develop following tracheostomy due to accidental injury to: (IMDC)
 Injury to esophagus
 Air trapped sucked from the skin incision
 Injury to the apex of pleura*
 Injury to recurrent laryngeal nerve
 Injury to the lung parenchyma
529. A 17 years old male is suffering from pain in mouth and throat with high grade fever. On
examination. there are multiple small superficial ulcers in oral cavity and oropharynx. Neck
lymph nodes are also enlarged. Blood picture shows neutrophil count which is 1000/cm.
what is the one most probable diagnosis? (RIHS)
 Oral stomatitis
 Leukemia*
 Agranulocytosis
 Dengue fever
 Candidiasis
530. A 30 years old male is having severe pain right side of throat and high grade fever. He is
unable to open his mouth Saliva is dribbling from right angle of mouth. He complains of
pain in the right eat He also gives history of sore throat for the last 3 days. Jugulodiagastric
lymph node on right side is palpale and tender Which is the single best diagnosis (RIHS)
 Parapharyngeal abscess
 Acute retropharyngeal abscess
 Chronic retropharyngeal abscess
 Cervical abscess(right)
 Peritonsillar abscess*
531. A 21 years old female has recurrent painful ulcers on the buccal mucosa and tongue for
last few months. These ulcers subside spontaneously after few days to week. Throat swab
for smear examination and culture is negative. Blood CP is within normal limits. What is the
most probable diagnosis? (RIHS)
 Vincent's angina
 Aphthous ulcer*
 Malignant oral ulcer
 Infectious mononucleosis
 Leukemia
532. A child was brought with 8 hours history of severe sore throat, saliva dribbling and
difficulty in lying down. He also had high grade fever. What is your diagnosis? (RIHS)
 Acute Epiglottitis*
 Laryngotracheal bronchitis
 Peritonsillar abscess
 Para pharyngeal abscess
 foreign body airway
533. A 25 year old woman presents with weight loss, cough, hoarseness and swelling in front of
neck. There are several non-tender swellings on both sides of the neck. She has history of
evening rise of temperature and frequent palpitations. The probable diagnosis is? (RIHS)
 Carcinoma of larynx
 Intubation granuloma
 Thyrotoxicosis*
 Vocal cord nodules
 Vocal cord polyp
534. A 10 year old girl underwent tonsillectomy in the morning Suddenly vomits blood in
afternoon She looks quite pale &her pulse is 110/minute. Her BP is 100/60mmHg. She is
most likely (RIHS)
 Primary hemorrhage
 Secondary hemorrhage
 Tertiary hemorrhage
 Reactionary hemorrhage*
 Hematemesis
535. A 12 year old boy is currently suffering from acute follicular tonsillitis He has history of
recurrent tonsillitis every 2-3 months for the last 3-4 years. He has no mouth breathing or
significant symptoms. What can be done in this case? (RIHS)
 Tonsillectomy is not indicated
 Tonsillectomy can be done straightaway under heavy antibiotic cover
 He should be managed conservatively & tonsillectomy done 4-6 weeks later*
 He needs both tonsillectomy & adenoidectomy
 Only broad spectrum antibiotics for 5 days
536. A 3 year old girl presented with mouth breathing recurrent upper respiratory infections
and nasal discharge, X-ray lateral view neck showed a soft tissue in the Nasopharynx. The
clinical diagnosis based on the data is (RIHS)
 Chronic maxillary sinusitis
 allergic rhinitis
 Adenoids*
 DNS
 Recurrent tonsillitis
537. A 9 year old child is suffering from pain in throat and is unable to swallow due to pain he
has high grade fever accompanied by rigors for the last two days On throat examination.
palatine arch uvula. pharyngeal walls are hyperemic and swollen, tonsils are enlarged,
swollen, congested and angry looking. Medial surface of tonsils are covered by grayish
white membrane which is easily visible. Jugulodigastric lymph nodes are enlarged and
tender. Temperature is 103 F. what is the most likely diagnosis? (RIHS)
 Pharyngeal diphtheria*
 Acute tonsillitis
 Candidiasis
 Infectious mononucleosis
 Retropharyngeal abscess
538. A 19 years old male is suffering from pain in throat, odynophagia, fever, lymphadenitis
neck of 2 weeks duration Tonsils are swollen, congested red. lymph node neck enlarged.
Liver and spleen are also enlarged. What is single best investigation to confirm diagnosis?
(RIHS)
 Blood complete picture and ESR
 LFT's and RFT's
 Throat swab
 Monospot test *
 FNAC cervical lymph nodes
539. A 30 year old female develops dysphagia, trismus and ulceration on oral mucosa She gives
history of pan chewing for last 10 years. Diagnosis is (RIHS)
 Aphthous ulcer
 Candidiasis
 Herpetic gingivostomatitis
 Submucosal fibrosis*
 Leukoplakia
540. 20 years old male presented with submental and submandibular swelling after tooth
extraction What is diagnosis? (RIHS)
 Ludwig's angina*
 Sialadenitis
 Parapharyngeal abscess
 Peritonsillar abscess
 Leukemia
541. Which statement is CORRECT regarding vocal nodules
 Are benign lesions*
 Vocal nodules tends to be larger than laryngeal polyps
 Develop at the middle of vocal cords
 Seldom develop bilaterally.
 Can be prevented by immunization
542. Peritonsillar abscess is also known as:
 Retropharyngeal abscess
 Tonsillar abscess
 Quinsy*
 Thornwaldts abscess
 Ludwig angina
543. Third molar caries with extension of the lesion towards tonsillar fossa and shift of tonsil
reveals which of the following complication?
 Parapharyngeal abscess*
 Retropharyngeal abscess
 Tonsillar abscess
 Dental abscess
 Ludwig angina
544. Peritonsillar abscess can extend posteriorly into:
 Anterior triangle of neck
 Parapharyngeal space*
 Posterior triangle on neck
 Submaxillary space
 Mastoid space
545. Swelling between tonsillar area and superior constrictor muscle is know as:
 Quinsy*
 Dental abscess
 Parapharyngeal abscess
 Retropharyngeal abscess
 bezold abbess
546. Killiance dehiscence is seen in
 Oropharynx
 Nasopharynx
 Cricopharynx*
 Vocal cords
 Esophagus
547. Early post-tonsillectomy complications include all of the following except:
 Oedema of uvula
 Secondary hemorrhage*
 Pneumonia
 Referred otalgia
 Anaesthesia complications
548. Indications of tracheostomy:
 In all cases of acute laryngitis
 any incubated patient within 3 days*
 Unilateral choanal atresia
 Ludwig's angina
 If there is suspicion of laryngomalacia
549. What is wrong about the pharynx?
 Extends from base of the skull to the sixth cervical vertebra at upper border of the cricoid
cartilage
 It is the upper part of the respiratory tract
 It is the upper part of the digestive tract
 About 10 cm in length in adult
 The pharyngeal cavity opens in front into the nose, mouth, and larynx*
550. Odynophagia is
 Pain during swallowing*
 Difficulty in swallowing
 Bad odour from mouth
 Psychiatric disease
 difficulty in opening mouth
551. Indications for tracheostomy are all EXCEPT:
 Acute epiglottitis
 Maxillofacial trauma
 Laryngeal malignancy
 Extensive consolidation of lung*
 upper airway obstruction
552. Most common complication of Tracheostomy is:
 Stenosis
 Infection*
 Pneumonia
 Respiratory failure
 Granulation tissue formation
553. Collar stud abscess is seen in:
 Pyogenic cervical abscess
 Peritonsillar abscess
 Retropharyngeal abscess
 TB lymphadenitis*
 Malignancy
554. Steeple sign is seen in:
 Croup*
 Acute epiglottitis
 Laryngomalacia
 Quinsy
 laryngeal diphtheria
555. The antibiotic of choice in acute epiglottitis pending culture sensitivity report is:
 Erythromycin
 Tetracycline
 Doxycycline
 Ampicillin*
 ciprofloxacin
556. Which of the following statement Is true for Ludwig's angina?
 It is an ischemic, painful condition of Pectoralis minor muscle
 It is diffuse cellulitis affecting the floor of the mouth*
 Glycerin nitrate, local application is quite helpful
 it is contagious disease
 None of the above
557. Constrictions normally present in esophagus are all EXCEPT:
 10 cm from the incisor teeth*
 15 cm from the incisor teeth
 25 cm from the incisor teeth
 40 cm from the incisor teeth
 27cm from the incisor teeth
558. The hypopharynx includes all the following EXCEPT
 Pyriform fossa
 Epiglottis*
 Post cricoid region
 Valeculae
 part of posterior pharyngeal wall
559. A lesion causing compression of the facial nerve at the stylomastoid foramen will cause
ipsilateral
 Paralysis of the facial muscles*
 Paralysis of the facial muscles and loss of taste
 Paralysis of the facial muscles, loss of taste and lacrimation.
 Paralysis of the facial muscles, loss of taste, lacrimation and decreased salivation
 none of the above
560. Paralysis of all facial muscles without hyperacusis and with intact taste sensation in
anterior two third of the tongue, the site of lesion in facial nerve is
 cortical lesion
 Lesion in the internal capsule
 Lesion in stylomastoid foramen *
 Lesion in fallopian canal
 lesion in parotid gland
561. All of the following muscles are supplied by facial nerve except
 Orbicularis oculi
 Levator Palpebrae Superioris *
 Posterior belly of digastric*
 Orbicularis oris
 Buccinator
562. Omega shaped epiglottis seen in
 Laryngeal tuberculosis
 Laryngomalacia*
 Tracheal stenosis
 Carcinoma larynx
 None of the above
563. 25 year old man who is on chemotherapy has developed pain in throat and odynophagia,
Throat examination shows curd like raised patches surrounded by zone of congestion on
tonsils, pillar's and buccal mucosa. Blood picture is normal. What is one probable diagnosis.
(RIHS)
 Agranulocytosis
 Vincent's angina
 Candiasis*
 Leucoplakia
 Sub mucous fibrosis
564. Following structures a part of oral cavity (RIHS)
 Uvula
 Soft palate
 Retromolar tigon*
 Epiglottis
 Tonsils
565. Adenoids hypertrophy can lead Into all the following problems Except. (RIHS)
 Nasal obstruction
 Mouth breathing
 Decreased hearing
 Poor performance at school
 Ear discharge*
566. Nasopharyngeal angiofibroma mostly occurs in. (RIHS)
 Young males*
 Young female
 Elderly male
 Elderly female
 Any gender
567. A patient presented with dysphagia and fever. He had some history of tooth Extraction,
On Examination he had hard swelling in upper part of the neck, Diagnosis is (RIHS)
 Plummer Vinton disease
 Pharyngeal pouch
 Peritonsillar abscess
 Ludwig’s angina*
 Parapharyngeal abscess
568. Reactionary hemorrhage regarding tonsillectomy occurs (RIHS)
 At the time of surgery
 Within 24 hour after surgery*
 Alter 24 hours of surgery
 At 5th day of surgery
 Any time after surgery
569. A child presented with sore throat and low grade fever for 5 days Throat examination
showed membrane over the tonsils and soft palate. Diagnosis is (RIHS)
 Acute membranes tonsillitis
 Acute follicular tonsillitis
 Faucial diphtheria *
570. Following is not the complication of Adenoidectomy (RIHS)
 Trauma to Eustachian tube
 Griesel syndrome
 Rhinolalia clausa *
 Rhinolalia Sparta
571. The most common complication of Tonsillectomy Is (RIHS)
 Primary haemorrhage*
 Reactionary haemorrhage
 Secondary Haemorrhage
 Tonsillar remnant
 Decreased immunity
572. Posterior faucial pillar is formed by (RIHS)
 Palatopharyngeal*
 Superior constrictor
 Palatoglossus
 Glossopharyngeus
 Middle constrictor
573. Frontal sinus opens into (RIHS)
 Sphenoethmoidal recess
 Superior meatus
 Middle meatus*
 Inferior meatus
 Uncinate process
574. Regarding complications of tonsillectomy, Secondary henmorrhage occur (RIHS)
 During the operation
 During 24 hours alter operation
 Alter 24 hours of operation*
 On 2 day after operation
 Any time after operation
575. One of the following is not included in Signs of a patient with chronic tonsillitis (RIHS)
 Prominent tonsillar crypts*
 Enlarge tonsils
 Enlarge lymph nodes
 Enlarge of tonsillar pillar
 Choesy material in crypts
576. Nasopharyngeal carcinoma ls ideally treated with (RIHS)
 Surgery
 Radiotherapy*
 Steroids
 Chemotherapy
 System Antibiotics
577. The following condition is not a causing factor for chronic pharyngitis (RIHS)
 Asthma*
 Smoking
 GERD
 Mouth breathing
 DNS
578. The presence of sore-throat, fever and presence of grey membrane over the tonsil and
uvula is a child are features of (RIHS)
 Quinsy.
 Chronic tonsilitis
 Diphtheria*
 Membranous tonsillitis
 follicular tonsillitis
579. A four months old male child is brought by his mother to ENT department for
consultation. Child has inspiratory stridor since birth, which exaggerates on crying and child
also becomes dyspneic but during sleep stridor is absent. What is your one possible clinical
diagnosis (RIHS)
 Congenital web
 Congenital vocal cord paralysis
 Laryngomalacia *
 Recurrent Juvenile Papilloma
 Subglottic stenosis
580. A Baja Blower of six years of age who has been in profession for last 40 years. Patient
complains of swelling appears in the upper part of neck on right side on blowing Baja and
reduces on compressing with hissing noise He also feels slight hoarseness and dyspnea
What one most probable clinical diagnosis when there is no other symptoms or past history
of surgery or trauma. (RIHS)
 Cold abscess
 Pseudo aneurysm
 Pharyngeal pouch
 Laryngocele *
581. 27 year old male who had toothache in right lower tooth for last 07 days has developed
pain in throat/odynophagia change in voice, slight dyspnea, inability to open the mouth.
Early on examination it was tender, non-fluctuating, indurated swelling in submandibular
region, swelling of floor of mouth and tongue pushed back ward and fever. What is one
possible clinical diagnosis: (RIHS)
 Peritonsillar abscess
 Retropharyngeal abscess
 Ludwig's angina *
 Alveolar abscess
 Parapharyngeal abscess
582. A 60 years old male who is heavy smoker presented with history of hoarseness presenting
for last 7 weeks. Indirect laryngoscopy revealed small exophytic growth of maize grain size
at the middle of night vocal cord. Both cords are freely Mobile. No neck glands. Direct
laryngoscopy and biopsy shows squamous cell carcinoma. What is one best choice of
treatment (RIHS)
 Radio therapy alone *
 Cordectomy
 Vertical Hemi laryngectomy
 Total laryngectomy
 Chemotherapy
583. You are called to see at child in emergency at mid night. The child is sitting up in bed, looks
toxic is dyspneic, stridulous, has dysphagia and drooling of saliva and high grade lever.
History reveals that the child had mild cold this morning Which has dramatically progressed
to this state by mid night. What is one possible diagnosis (RIHS)
 Acute laryngitis
 Acute laryngo tracheo bronchitis
 Acute epiglottitis *
 Acute retropharyngeal abscess.
584. A 45 years old female has presented to ENT department with history of dysphagia of 1 1/2
years which was gradual in onset. There is no significant weight loss/ no lump in neck.
There is koilonychia of fingers, tongue is smooth and there are trismus at angle of mouth.
Barium swallow show a smooth stricture at the upper end of esophagus. Give one best slip
you will take in the case. (RIHS)
 Endoscopy and dilatation
 Endoscopy and biopsy *
 MRI of Neck
 X-ray chest
 Complete blood picture & BSR
585. A 35 year old male who drinks alcohol, is chain smoker and nonprofessional singer is
Suffering from hoarseness for the last four month. IDL shows white particles on vocal
cords. On biopsy and histopathology report reveal carcinoma in situ. What is choice of
management (RIHS)
 Total laryngectomy
 Radiotherapy
 Follow up for observation after complete peeling *
 Nothing to be done after peeling removal
 Chemotherapy
586. A 20 years male has presented with pain in throat difficulty in swallowing and fever of 5
days duration. Examination reveals multiple small, shallow ulcers on Gum's buccal mucosa,
tongue and pillars covered with dirty white membrane. Which can be easily removed.
Patient has characteristic fruity odour breath. Give one best investigation for diagnosis.
(RIHS)
 Complete blood picture sod ESR
 Throat swab for culture
 Throat swat for staining & smear examination *
 Biopsy of the ulcer
 Xray chest
587. A 3 years old boy was playing with other small children that he suddenly developing
choking which settled after few minutes. Ever since he is unable to swallow anything, All
vital signs and chest auscultation is normal throat NAD. Tick one most important step to
like to take: (RIHS)
 Laryngoscopy & bronchoscope under GA
 Esophagoscopy under GA *
 Antibiotics and steroids course
 Blood ESR & CP
 Plan X-ray soft tissue neck/ chest and abdomen
588. 40 years old man who underwent difficult esophagoscopy is received in the ward from
operation theatre. You suspect perforation in esophagus. Tick the best one earliest clinical
feature: (RIHS)
 Surgical emphysema
 Shock
 Pain chest & back *
 Dysphagia
 Raised rate of pulse and respiration
589. Eighteen years old girl has developed hoarseness following an arranged engagement by
her parents against her liking. Hoarseness is of sudden & dramatic onset. On indirect
laryngoscopy vocal cords look normal, but adduction IS incomplete on phonation. What is
likely diagnosis (RIHS)
 Chronic laryngitis
 Myasthenia gravis
 Functional dysphonia *
 Bilateral Adduction paralysis
590. A 2 years old patient has swallowed coin. Which one of the following is most common site
for impaction of such foreign body. (RIHS)
 Lower end of esophagus
 Cricopharynaious (upper end) *
 Left bronchus crosses oesophagus
 Arch of aorta cross oesophagus
 Oropharynx
591. 12 year old male child has painless, non tender cystic swelling in mid line just below the
hyoid. It moves with swallowing and is present since early childhood What may be possible
clinical diagnosis (RIHS)
 Hyoid burse
 Nodular Goiter of isthmus
 Thyroglossal cyst *
 Metastatic lymph node
 Laryngocele
592. 40 year old lady who was suffering from reflux for the last fifteen years has developed
dysphagia over the last two months, which is rapidly increasing and she is loosing weight
Which of the following investigation is mandatory in confirming diagnosis (RIHS)
 Barium swallow
 Blood ESR &CP
 Esophagoscopy & biopsy *
 CT Scan abdomen
 Chemotherapy
593. A young lady presented to ENT OPD with complaints of fever, difficulty opening mouth
and neck swelling. The swelling was hard and tender. She also gave history of check-up to a
Dentist a week ago. What is your clinical diagnosis? (RIHS)
 Quinsy
 Retropharyngeal abscess
 Parapharyngeal abscess
 Ludwig's angina*
 Goiter
594. One of the following is Not a feature of chronic pharyngitis: (RIHS)
 Pain in throat
 Cough
 Tiredness of voice
 Foreign body sensation in throat
 Fever *
595. A middle age anemic lady is having progressive dysphagia for solids and brittle nails for 2
years. Her neck examination is normal. The likely diagnosis is: (RIHS)
 Carcinoma larynx
 Carcinoma pharynx
 Globus pharyngeus
 Plummer-vinson syndrome*
 Pharyngeal pouch
596. Regarding tonsillectomy, reactionary haemorrhage occurs: (RIHS)
 During surgery
 After surgery for up-to 24 hours*
 After 24 hours of surgery
 On 5 post-operative day
 Any time after operation
597. A 5 years old child is having sore-throat, low-grade fever and cervical lymphadenopathy.
On examination he is having grayish membrane over the tonsils and soft palate. What is
your provisional diagnosis? (RIHS)
 Diphtheria*
 Follicular tonsillitis
 Membranous tonsillitis
 Leukemia
 Aphthous ulcer
598. Which of the following muscle of larynx is not supplied by recurrent laryngeal nerve?
(RIHS)
 Lateral cricoarytenoid
 Posterior cricoarytenoid
 Cricothyroid*
 Aryepiglottic
 Thyroepigiottic
599. A 17 years old patient has developed pain in throat & fever for last 3 days. On
examination there are multiple oral ulcers covered by yellow-white membrane present on
gums, tongue, tonsil and pillars. Suspected diagnosis is Vincent angina. Indicate one best
test for diagnosis. (RIHS)
 Blood ESR and complete picture
 Throat swab for culture
 Smear examination under microscope*
 Biopsy of the ulcer.
 Serum IgE
600. Treatment of laryngeal carcinoma stage IV-b is: (RIHS)
 Partial laryngectomy
 Chemotherapy
 Palliative treatment*
 Chemotherapy and partial laryngectomy
 Supragiottic laryngectomy
601. A mother brought her young college going son that he's not having normal masculine
voice. The boy is shy but having no any other physical problem. Your diagnosis is: (RIHS)
 Vocal cord palsy
 Vocal nodule
 Vocal polyp
 Puberphonia*
 Functional aphonia
602. Which of the following structure is Not seen on X-ray lateral view skull and neck? (RIHS)
 Trachea
 Maxillary sinus*
 Esophagus
 Cervical vertebrae
 Mandible
603. A Young female is having severe sore throat, difficulty swallowing and fever. On
examination tonsil is enlarged and pushing the ovule to opposite side. The likely diagnosis
is: (RIHS)
 Diphtheria
 Tumor of the tonsil
 Acute tonsillitis
 Quinsy *
 Tonsillar cyst
604. Soon after thyroidectomy, the patient developed shortness of breath and strider after
examination. What immediately should be done? (RIHS)
 Oxygen inhalation through mouth
 Lateralization of vocal cords
 Portable CT scan to knew about problem
 Tracheostomy *
 I/V steroids
605. A child was brought to ENT OPD with complaints of fever, enuresis end bilateral tenders
swelling in parotid region. Which of the following organism is most involved in this case?
(RIHS)
 Staphylococcus aureus
 Streptococcus viridians
 Herpes zoster virus
 Paramyxovirus *
 Pentadomeuas
606. Vocal polyp is present mostly in: (RIHS)
 Smokers *
 Singers
 Trumpet players
 Teachers
 Weight lifter
607. Vocal nodules are present at (RIHS)
 Center of false vocal cords
 Junction of anterior 1/3rd and posterior 2/3rd of true vocal cord *
 Junction of anterior 2/3rd and posterior 1/3rd of true vocal cord
 At the center of true vocal cord
 Any area of true vocal cords
 The most common site for incidence of laryngeal carcinoma is (RIHS)
 Glottis *
 Subglottis
 Supraglottis
 Epiglottis
 Arytrnoids
608. Thorwald’s cyst is seen in. (YMDC)
 Floor of mouth
 Oropharynx.
 Nasopharynx*
 Hypopharynx.
 Larynx
609. After a Road traffic accident 21 years old, was admitted in ICU, anticipating his prolonged
stay in ICU his tracheostomy was performed. Care of tracheotomy Patient include: (YMDC)
 Humidification by wet gauze
 Changing the tube
 Care of the inflatable cuff
 All the above*
 None of the above
610. A 7 year old male presents to the ER with a two day history of worsening ear pain. and
drainage. On the day prior to presentation, his parents noted redness, behind his right ear
and that his right ear appeared to be sticking out. He had been well until 10 days ago when
he starred complaining of Cough and runny nose that progressed to include right ear pain
and fever. He was evaluated in the clinic 5 days ago and diagnosed with an acute right otitis
media. He was placed on amoxicillin and he initially appeared to improve until two days
ago when his ear pain recurred and this is How accompanied by ear drainage. redness
behind his right ear, and a prominent right pinna which is pointing up and out, The most
likely diagnosis? (YMDC)
 Otitis externa
 Otitis media with effusion
 Mastoiditis*
 Necrotizing otitis externa
611. On endoscopy of nose a patient was found fossa of Rosenmullaar. Which is a common site
for: (FMDC)
 Angiofibroma
 Lipoma
 Adenoid
 Nasopharyngeal carcinoma*
 Thorwald’s cyst
612. Rhinophyma is associated with: (YMDC)
 Hypertrophy of the sebaceous glands*
 Hypertrophy of sweat glands
 Hypertrophy of endothelial cells
 Hypertrophy of epithelial cell
 Hypertrophy of follicies
613. Treatment of choice in a 6 year old child with recurrent URTI with mouth breathing and
failure to grow with high arched palate and impaired hearing is: (YMDC)
 Tonsillectomy
 Grommet insertion
 Myringotomy with grommet insertion
 Adenoidectomy with grommet insertion*
 Medical management
614. A patient presented to you complaint of left submandibular pain and swelling whe ning.
O/E, there enlarged submandibular gland firm What is the o likely Diagnosis? (YMDC)
 Mumps
 Sjogren’s syndrome
 Hodgkin's lymphoma
 Salivary gland calculi
 Ludwig's Angina*

You might also like